1. Trang chủ
  2. » Luận Văn - Báo Cáo

Các chuyên đề hình học lớp 9

278 46 0

Đang tải... (xem toàn văn)

Tài liệu hạn chế xem trước, để xem đầy đủ mời bạn chọn Tải xuống

THÔNG TIN TÀI LIỆU

Nội dung

Ta có hình chữ nhật và hình thang cân đều có tổng hai góc đối diện bù nhau nên chúng nội tiếp trong một đường tròn. Chứng minh tứ giác ABCD nội tiếp được. Từ B kẻ tiếp tuyến Bx với đ[r]

(1)



Tài liệu sưu tầm

CÁC CHUYÊN ĐỀ BỒI DƯỠNG

HÌNH HỌC LỚP

(2)

1

Mục Lục

Trang Lời nói đầu

Chủ đề Hệ thức lượng tam giác vuông Chủ đề Tứ giác nội tiếp

Chủ đề Hai tam giác nhau Chủ đề Hai tam giác đồng dạng Chủ đề Đoạn thẳng nhau Chủ đề Đường thẳng song song Chủ đề Đường thẳng vng góc Chủ đề Các điểm thẳng hàng Chủ đề Các đường thẳng đồng quy Chủ đề 10 Tiếp tuyến đường tròn

Chủ đề 11 Độ dài cạnh- độ lớn góc- diện tích Chủ đề 12 Đẳng thức hình học

Chủ đề 13 Cực trị hình học

Chủ đề 14 Quan hệ góc đường tròn Chủ đề 15 Tứ giác nội tiếp phần 2

(3)

CHƯƠNG 1- HỆ THỨC LƯỢNG TRONG TAM GIÁC VUÔNG Hệ thức cạnh đường cao

KIẾN THỨC CƠ BẢN

Khi giải toán liên quan đến cạnh đường cao tam giác vng, ngồi việc nắm vững kiến thức vềđịnh lý Talet, vềcác trường hợp đồng dạng tam giác, cần phải nắm vững kiến thức sau:

Tam giác ABC vuông A, đường cao AH , ta có: 1) a2 b2 c2

2) b2 a b c '; a c ' 3) h2 b c' '

4) a hb c 5) 12 12 12

hbc 6) b' b22

aa Chú ý: Diện tích tam giác vng:

2 Sab

Ví dụ Cho tam giác ABC vuông A, đường cao AH Biết AB AC: 3 : ABAC 21cm

a) Tính cạnh tam giác ABC b) Tính độdài đoạn AH BH CH, ,

b' c'

h c

b

a

H C

B

(4)

Giải:

a) Theo giả thiết: AB AC: 3 : 4,

suy

3 4

ABACABAC

Do AB 3.39

 

cm ;

 

3.4 12

AC   cm

Tam giác ABC vuông A, theo định lý Pythagore ta có: 2 92 122 225

BCABAC    , suy BC 15cm b) Tam giác ABC vng A, ta có AH BCAB AC , suy

 

9.12 7,2

15 AB AC

AH cm

BC

  

2 .

AHBH HC Đặt BHx

0 x 9

HC 15x, ta có:

 

2

2

7,2 x 15xx 15x 51, 84 0 x x5, 9,6 x5, 

x 5, 4



x 9,6

x 5,

      x 9,6 (loại) Vậy BH 5, 4cm Từđó HCBCBH 9,6

 

cm

Chú ý: Có thể tính BH sau:

2 .

ABBH BC suy 92 5, 4

 

15

AB

BH cm

BC

  

A

(5)

Ví dụ 2: Cho tam giác cân ABC có đáy BC 2a, cạnh bên

b ba

a) Tính diện tích tam giác ABC b) Dựng BKAC Tính tỷ số AK

AC Giải:

a) Gọi H trung điểm BC Theo định lý Pitago ta có:

2 2 2

AHACHCba

Suy . 2

2

ABC

SBC AHa ba 2

AH b a

  

b) Ta có

2BC AH  2BK ACSABC

Suy BK BC AH 2a b2 a2

AC b

   Áp dụng định lý Pitago tam giác vng

AKB ta có:

 

2 2

2 2 2

2

2

4a b a

AK AB BK b b a

b b

      Suy

2 2 b a AK

b

2

2 b a AK

AC b

Ví dụ 3: Cho tam giác ABC với đỉnh A B C, , cạnh đối diện với đỉnh tương ứng là: a b c, ,

a) Tính diện tích tam giác ABC theo a b) Chứng minh: a2 b2 c2 4 3S

K

H C

B

(6)

Giải:

a) Ta giả sử góc A góc lớn tam giác ,

ABCB C góc nhọn Suy chân đường cao hạ từ A lên BC điểm

H thuộc cạnh BC

Ta có: BCBHHC Áp dụng định lý Pi ta go cho tam giác vuông

,

AHB AHC ta có:AB2 AH2 HB AC2, AH2 HC2 Trừhai đẳng thức ta có:



2 2 .

cbHBHCHBHC HBHCa HBHC

2 c b HB HC

a

   ta có: 2

2 a c b HB HC a BH

a

 

   

Áp dụng định lý Pitago cho tam giác vuông

2 2 2 2 2

2

2 2

a c b a c b a c b

AHB AH c c c

a a a

          

    

        

  

    

2 2 2

2







2

2

a c b b a c a b c a c b b a c b c a

a a a

                                     

Đặt 2p   a b c









2 16

p p a p b p c p p a p b p c

AH AH a a         

Từđó tính





2

SBC AHp p a p b p c  

H

C B

(7)

b) Từ câu a) ta có: Sp p a p b p c





Áp dụng bất đẳng thức Cơ si ta có:

p a p b p c





p a     p b3 p c3  27p3

 Suy

3

27 3 3

p p

Sp

Hay

2

12 a b c

S    Mặt khác ta dễ chứng minh được:

2

2 2 2

3

a  b cabc suy

2

2 2

4 12

a b c

S    abcS Dấu xảy hki tam giác ABC

Ví dụ Cho tam giác nhọn ABC đường cao CK ; H trực tâm tam giác Gọi M điểm CK cho AMB 900

1 , ,

S S S theo thứ tự diện tích tam giác ,

AMB ABC ABH Chứng minh SS S1 2 Giải:

Tam giác AMB vng MMKAB nên MK2 AK BK. (1)

AHK CBK

  có   900

AKHCKB  ; KAH KCB

(cùng phụ với ABC) Suy AK HK

CKBK , AK KBCK KH (2) Từ (1) (2) suy MK2 CK HK. nên MKCK HK. ;

1

1 1

2 2

AMB

SAB MKAB CK HKAB CK AB HKS S

Vậy SS S1 2

D

K M

H

C B

(8)

Ví dụ Cho hình thang ABCDA D 90 ,0 B 60 ,0 CD 30cm CA, CB Tính diện tích hình thang

Giải:

Ta có CAD ABC 600 (cùng phụ với CAB), thế tam giác vng ACD ta có AC 2AD

Theo định lý Pythagore thì: AC2 AD2 DC2 hay

2AD

2 AD2 302 Suy 3AD2 900AD2 300 nên AD 10 3

 

cm

KẻCHAB Tứ giác AHCD hình chữ nhật có A D H 900, suy

 

30 ; 10

AHCDcm CHADcm

Tam giác ACB vuông C , ta có: CH2 HA HB. , suy

 

2

 

2 10 300 10

30 30

CH

HB cm

HA

    ,

 

30 10 40

ABAHHB    cm

 

2

1 1.10 40 30 350 3

2

ABCD

SCH ABCD   cm

Vậy diện tích hình thang ABCD 350 3cm2

Tỉ số lượng giác góc nhọn

KIẾN THỨC CƠ BẢN

1 Các tỉ sốlượng giác góc nhọn (hình) định nghĩa sau: sin AB;cos AC ; tan AB;cot AC

BC BC AC AB

(9)

+ Nếu góc nhọn 0sin1;0cos1; tan0;cot0

2 Với hai góc  , mà   900,

ta có: sincos ;cos sin ; tan cot ;cot tan

Nếu hai góc nhọn có sinsin coscos sin2cos21;tg.cotg1

4 Với sốgóc đặc biệt ta có:

0 0

sin 30 cos 60 ;sin 45 cos 45

2

   

0 0

cos 30 sin 60 ;cot60 tan 30

2 3

   

0 0

tan 45 cot 45 1;cot 30 tan 60 

Ví dụ Biết sin 13

 Tính cos , tan cot

Giải:

Cách Xét ABC vuông A Đặt B  Ta có: sin

13 AC BC

  suy

5 13

AC BC k

  ,

5 , 13

ACk BCk Tam giác ABC vuông A nên:

   

2

2 2 13 5 144

ABBCACkkk , suy AB 12k

α

Cạnh đối Cạnh huyền

Cạnh kề C

B

A

α B C

(10)

Vậy cos 12 12

13 13

AB k BC k

   ; tan 5 ;

12 12

AC k AB k

   cot 12 12

5

AB k AC k    Cách Ta có sin

13

 suy sin2 25 169

 , mà sin2cos21,

2 25 144

cos sin

169 169

     , suy cos 12 13

sin 12 13

tan :

cos 13 13 13 12 12

    ; cot cos 12 5: 12 13 12

sin 13 13 13 5

   

Ở cách giải thứ ta biểu thịđộ dài cạnh tam giác ABC theo đại lượng k sử dụng định nghĩa tỉ sốlượng giác góc nhọn để tính cos , tan , cot Ở cách giải thứ hai, ta sử dụng giả thiết sin

13

 để tính sin2 rồi tính cos từ

2

sin cos 1 Sau ta tính tan cot qua sin cos

Ví dụ Cho tam giác nhọn ABC hai đường cao AD BE cắt H Biết : :

HD HA Chứng minh tgB tgC 3 Giải:

Ta có: tgB AD;tgC AD

BD CD

 

Suy tan tan AD

B C

BD CD  (1)

 

HBDCAD (cùng phụ với ACB); HDB ADC 900

Do BDH ADC (g.g), suy DH BD

DCAD , BD DCDH AD (2) Từ (1) (2) suy tan tan

AD AD

B C

DH AD DH

  (3) Theo giả thiết HD

AH  suy

H E

D C

B

(11)

ra HD

AHHD   hay

1 HD

AD  , suy AD 3HD Thay vào (3) ta được:

tan tanB C HD

DH

 

Ví dụ Biết sin cos 12 25

 Tính sin , cos

Giải:

Biết sin cos 12 25

 Để tính sin , cos ta cần tính sincos giải phương trình với ẩn sin cos

Ta có:

sin cos

2 sin2 cos2 2 sin cos 1 2.12 49

25 25

   Suy

sin cos

5

 nên sin cos

  Từđó ta có:

7 12 12

cos cos cos cos

5 25 25

   

 

 

2

25 cos 35 cos 12 cos cos cos

        

5 cos cos



3

    Suy cos

 cos  + Nếu cos

5

 sin 12 4:

25 5

  + Nếu cos

5

 sin 12 3:

25 5

  Vậy sin

5

 , cos

 sin 4, cos

5

(12)

1 Trong tam giác vng, cạnh góc vng bằng:

a) Cạnh huyền nhân với sin góc đối hay nhân với cosin góc kề

b) Cạnh góc vng nhân với tan góc đối hay nhân với cot góc kề

.sin cos ; sin cos ; cot ;

ba Ba C ca Ca B bc tgBc gC

.cot

cb tgCb gC

2 Giải tam giác vng tìm tất cạnh góc chưa biết tam giác vng

Ví dụ Cho tam giác ABCAB 16,AC 14 B 600 a) Tính độ dài cạnh BC

b) Tính diện tích tam giác ABC Giải:

a) Kẻđường cao AH

Xét tam giác vng ABH, ta có:

0

.cos cos 60 16

2 BHAB BAB  

0

.sin sin 60 16

2

AHAB BAB   Áp dụng định lý Pythagore vào tam giác vuông AHC ta có:

 

2

2 2 142 8 3 196 192 4

HCACAH      Suy HC 2 Vậy 10

BCCHHB   

b) Cách 1 1.10.8 40

2

ABC

SBC AH   (đvdt)

Cách sin 1.10.16 40

2 2

ABC

SBC BA B   (đvdt)

A

B 60 C

(13)

Ví dụ 2: Tính diện tích tam giác ABC biết ABC 45 ,0 ACB 600 bán kính đường trịn ngoại tiếp tam giác ABC R

Giải:

Giả thiết có góc có sốđo đặc biệt , tam giác ABClà tam giác thường nên ta tạo tam giác vuông cách Dựng đường

thẳng qua C B, vng góc với ,

AC AB Gọi D giao điểm hai đường

thẳng Khi tam giác ABD ACD tam giác

vuông điểm A B C D, , , nằm đường trịn đường kính AD 2R Ta có: .sin 600 . 3

2

ABADADR Kẻđường cao AH suy HBC.Tức là: BCBHCH Tam giác AHB vng góc H nên

0

.sin 45

2 2

AB R

AHBHAB  AD  Mặt khác tam giác ACH vuông H nên 2

2 R

ACAHCHCH

1 2

2 R

BC

 

Từđó tính diện tích

3 3

4 R

S  

H

D

600 450

C B

(14)

Ví dụ 3: Cho tam giác ABC với đỉnh A B C, , cạnh đối diện với đỉnh tương ứng là: a b c, , Chứng minh rằng:

a) a2 b2 c2 2 cosbc A

b) Gọi D chân đường phân giác góc A Chứng minh: cos

2 A bc

AD

b c           

Giải:

a) Dựng đường cao BH tam giác ABC ta có:

Cách 1: Giả sử H thuộc cạnh AC Ta có: ACAHHC

Áp dụng định lý

Pi ta go cho tam giác vuông ,

AHB BHC ta có:AB2 AH2 HB BC2, BH2 HC2 Trừhai đẳng thức ta có:



2 2 .

caHAHCHAHC HA HC b HA HC

2

c a

HA HC

b

   ta có: 2

2

b c a

HA HC b AH

b

 

    Xét tam giác vuông AHB ta có: cos 2 2 2 2 cos

2

AH b c a

A a b c bc A

AB bc

 

     

Cách 2: Xét tam giác vng CHB ta có: c

b a

A

B

(15)

2

2 2 2 2 2 .

BCBHHCBHACAHBHAHACAC AH Ta có: AHCB.cosA suy BC2 BH2 AH2 AC2 2AC CB. .cosA hay

2 2 2 . .cos

BC BA AC AC CB A

     a2 b2 c22 cosbc A

b) Để chứng minh toán ta cần kết sau: + sin 22 sin cos

+ sin

Sab C

*) Thật xét tam giác vuông ABC A, 900, gọi M là trung điểm của BC, dựng đường cao AH Đặt ACB AMB 2

Ta có sin sinC AH h AC b   

cos cosC AC b

BC a   

sin sin

2

AH h h

AMH

AM a a

    Từđó ta suy ra: sin 22 sin cos

*) Xét tam giác ABC Dựng đường cao BE ta có:

1 . .

2

ABC

SBE ACBE b (1) Mặt khác tam giác vuông AEB ta có:sinA BE BE c.sinA

AB

  

thay vào (1)

2α α

h

b

H M C

B

A

E

C B

(16)

Ta có: sin

Sab C Trở lại tốn:

Ta có sin 1 sin

2 2

ABD

A SAD AB AAD c     

 

1 . sin .sin

2 2

ACD

A SAD AC AAD b     

  Suy SABCSACDSABD

1 sin

2

A

AD   c b      

  Mặt khác

1 sin

2 ABC

Sbc A

cos sin 2 sin sin sin A bc

A bc A

AD c b bc A AD

c b A b c                               

Chú ý rằng: Ta chứng minh kết sau: cos 22 cos2  1 1 sin2 Thật xét tam giác vuông ABC A, 900, gọi M là trung điểm của BC , dựng đường cao AH Đặt ACB AMB 2

Ta có : cos cosC AC b BC a

   sin sinC AB c

BC a

   ,

 2

cos cos

2

AM MB AB

(17)

2

2 2 2

2 2

2

2

4 1 2 1 2. 2 1

2 2

a a

c a c c a b b

a a a a a a

       

 

   

           

    Từ suy

2

cos 22 cos   1 sin

Áp dụng 2 2 cos 2 2 2 cos2 1

2 A abcbc Aabcbc  

 

2 2

2 2

2

2 cos cos

2 2

b c a

A b c a A

bc bc

 

 

     Thay vào công thức đường phân giác ta có:



2 2 cos

b c a A bc

bc bc b c a b c a

bc AD

c b b c b c

 

   

  

   Áp dụng bất

đẳng thức Cơ si ta có:



( )

2

b c a b c a

b c

bc   AD      p p a

với 2p   a b c

Áp dụng công thức: a2 b2 c22 cosbc A Ta chứng minh được hệ thức rất quan trọng hình học phẳng ( Định lý Stewart) là:

‘’Cho điểm D nằm cạnh BC tam giác ABC ta có:

2. 2. .

AB CDAC BDBC ABBD DC ’’ + Thật :Ta giả kẻ AHBC

không tính tổng quát, ta giả sử D nằm đoạn

HC Khi ta có:

2 2 2 . .cos 2 2 .

ABADBDAD BD ADBADBDDB DH (1) D

H

C B

(18)

Tương tự ta có: AC2 AD2 DC2 2DH DC. (2) Nhân đẳng thức (1) với DC đẳng thức (2) với BD cộng lại theo vế ta có:

2. 2. .

AB CDAC BDBC ABBD DC

Ví dụ Khơng dùng máy tính bảng số chứng minh

sin 75

4 

Giải:

Vẽ tam giác ABC vuông A với BC 2a (a độ dài tùy ý) , C 150, suy B 750

Gọi I trung điểm BC , ta có

IAIBICaAIB góc ngồi đỉnh I tam giác cân IAC nên  2 300

AIBC  Kẻ AHBC .cos 300 a IHAI  ;

.cos 30 a

AHAI  ;

2 3

2

a a

CHCIIH  a   Tam giác AHC vuông H , theo định lý Pythagore, ta có:

2

2

2

2 2 4 3

4 4

a a a

ACCHAH       

2

4

4 a

 a2

2 3

, suy ACa 2 3 I

H C

(19)

0 3

sin 75 sin

2 2 2

AC a B

BC a

  

    

2

3 3 1 6 2

4

2 2 2 2

   

   

Vậy sin 750 

(20)

Chủ đề 1: CHỨNG MINH TỨ GIÁC NỘI TIẾP A KIẾN THỨC CƠ BẢN

Tứ giác nội tiếp đường tròn tứ giác có bốn đỉnh nằm đường trịn Đường trịn đó gọi đường trịn ngoại tiếp tứ giác

I Phương pháp chứng minh: Chứng minh bốn đỉnh tứ giác cách điểm

CÁC VÍ DỤ Mức độ 1: NB

Câu 1: Cho hình thang ABCD (AB/ / ,CD AB <CD) có   60

C=D= ,CD=2AD Chứng minh bốn điểm A B C D, , , thuộc đường tròn

Hướng dẫn giải

Gọi I trung điểm CD, ta có / / IC AB

ICBA IC AB

= 

⇒ 

 hình hành⇒BC= AI (1) Tương tự AD=BI (2)

ABCDlà hình thang có C =D=600 nên ABCDlà hình thang cân(3); mà

Từ (1), (2), (3) ta có hai tam giác ICB IAD; hayIA=IB=IC=ID hay bốn điểm , , ,

A B C D thuộc đường tròn

(21)

Hướng dẫn giải

Do ABCD hình thoi nên O trung điểm AC, BD; AC, BD phân giác góc , , ,

A B C D nên ∆MAO= ∆SAO= ∆NCO= ∆PDOOM =ON =OP=OS hay bốn điểm , ,

M N R S thuộc đường tròn Câu 3: Cho tam giác ABC có đường cao BHCK

Chứng minh , , , B K H C nằm đường trịn Xác định tâm đường trịn

Hướng dẫn giải

Gọi I trung điểm CB, ∆CHB;∆CKB vuông H K, nên IC=IB=IK =IH hay , , , B K H C nằm đường tròn tâm I

Mức độ 2: TH

Câu 4: Cho đường trịn tâm O đường kínhAB Vẽ dây cung CD vng góc với AB I (I nằm A O ) Lấy điểm E cung nhỏ BC (E khác B C ),AE cắt CD F Chứng minh: BEFI tứ giác nội tiếp đường tròn

Hướng dẫn giải

F

E

I O

D C

B A

Tứ giác BEFI có: BIF=900(gt)

 

BEF=BEA=90 (góc nội tiếp chắn nửa đường tròn) Suy tứ giác BEFI nội tiếp đường tròn đường kính BF

(22)

a) Chứng minh: AIMK tứ giác nội tiếp đường tròn

b) VẽMPBC

(

PBC

)

Chứng minh: CPMK tứ giác nội tiếp Hướng dẫn giải

H

O P

K I M

C B

A

a) Ta có:AIM =AKM=900(gt), suy tứ giác AIMK nội tiếp đường trịn đường kính AM b) Tứ giác CPMK có  

MPC=MKC=90 (gt) Do CPMK tứ giác nội tiếp

Câu 6: Cho hình vng ABCD có hai đường chéo cắt tạiE Lấy I thuộc cạnh AB, M thuộc cạnh BC cho: IEM=900( I M khơng trùng với đỉnh hình vuông )

a) Chứng minh BIEM tứ giác nội tiếp đường trịn b) Tính số đo góc IME 

c) Gọi N giao điểm tia AM tia DC; K giao điểm BN tia EM Chứng minBKCE tứ giác nội tiếp

Hướng dẫn giải

I

E M

N

B C

A D

K

a)Tứ giác BIEM :IBM =IEM=900(gt);hay tứ giác BIEM nội tiếp đường trịn đường kính IM

(23)

c) ∆EBIECMBE=CE, BEI =CEM(   IEM=BEC=90 ) ⇒ ∆EBI =∆ECM (g-c-g)⇒MC=IBMB=IA

CN/ / BA nên theo định lí Thalet, ta có: MA MB MN =MC=

IA

IB Suy IM song song với BN (định lí Thalet đảo)

 

BKE IME 45

⇒ = = (2) Lại có 

BCE=45 (do ABCD hình vng) Suy BKE =BCE⇒ BKCE tứ giác nội tiếp

Mức độ 3: VDT

Câu 7: Cho nửa đường tròn tâm O đường kính AB=2R tia tiếp tuyến Ax phía với nửa đường tròn AB Từ điểm M Ax kẻ tiếp tuyến thứ hai MC với nửa đường tròn (C tiếp điểm).AC cắt OM E; MB cắt nửa đường tròn

( )

O D (D khác B )

Chứng minh: AMCO AMDE tứ giác nội tiếp đường tròn Hướng dẫn giải

x N

I H E D M

C

O B

A

MA MC, tiếp tuyến nên: MAO =MCO=900 ⇒ AMCO tứ giác nội tiếp đường trịn đường kính MO

ADB=90 (góc nội tiếp chắn nửa đường tròn)  ADM 90

⇒ = (1)

Lại có: OA=OC=R; MA=MC (tính chất tiếp tuyến) Suy OM đường trung trực AC

AEM 90 ⇒ = (2)

Từ (1) (2) suy AMDE tứ giác nội tiếp đường trịn đường kính MA

Câu 8: Cho hai đường tròn

( )

O (O )′ cắt A B Vẽ AC, AD thứ tự đường kính hai đường trịn

( )

O (O )′

a) Chứng minh ba điểm C B D, , thẳng hàng

b) Đường thẳng AC cắt đường tròn(O )′ E; đường thẳng ADcắt đường tròn

( )

O F (E F, khác A) Chứng minh bốn điểm C D E F, , , nằm đường tròn

(24)

d

K I

N M

F E

O/ O

C

D B

A

a) ABC  ABDlần lượt góc nội tiếp chắn nửa đường tròn

( )

O (O )′ ⇒ABC =ABD=900

Suy C B D, , thẳng hàng b) Xét tứ giác CDEF có:

 

CFD=CFA=90 (góc nội tiếp chắn nửa đường trịn (O))

 

CED=AED=90 (góc nội tiếp chắn nửa đường tròn (O/)

 

CFD CED 90

⇒ = = suy CDEF tứ giác nội tiếp

Câu 9: Cho đường tròn

( )

O (O )′ cắt hai điểm A B phân biệt Đường thẳng OAcắt

( )

O , (O )′ điểm thứ hai C D Đường thẳng O A′ cắt

( )

O , (O )′ điểm thứ hai E E, F

1 Chứng minh đường thẳng AB, CE DF đồng quy điểm I Chứng minh tứ giác BEIF nội tiếp đường tròn Hướng dẫn giải:

I

Q

O O'

F H

P E

D

C B

A

Ta có: ABC=90o(góc nội tiếp chắn nửa đường trịn)

 o

ABF=90 (góc nội tiếp chắn nửa đường trịn) nên B, C, F thẳng hàng AB, CE DF đường cao tam giác ACF nên chúng đồng quy

(25)

Câu 10: Cho nửa đường trịn tâm O đường kính AB Lấy điểm M thuộc đoạn thẳng OA, điểm N thuộc nửa đường tròn

( )

O Từ A B vẽ tiếp tuyến Ax By Đường thẳng qua V vng góc với NM cắt , Ax By thứ tự C D

a) Chứng minh ACNM BDNM tứ giác nội tiếp đường tròn

b) Chứng minh ∆ANB đồng dạng với ∆CMD từ suy IMKN tứ giác nội tiếp Hướng dẫn giải

K I

y x

D C N

M O B

A

a)Ta có tứ giác ACNM có: MNC=900(gt) MAC=900( tínhchất tiếp tuyến)

ACNM tứ giác nội tiếp đường trịn đường kínhMC Tương tự tứ giác BDNM nội tiếp đường trịn đường kính.MD

b) ∆ANBCMD có:  

ABN=CDM (do tứ giác BDNM nội tiếp)  

BAN=DCM (do tứ giác ACNM nội tiếp ) nên ∆ANB∆CMD (g.g)

c) ∆ANB∆CMD ⇒CMD =ANB=90o(do ANB  góc nội tiếp chắn nửa đường tròn

( )

O ) Suy IMK =INK=900⇒ IMKN tứ giác nội tiếp đường tròn đường kính IK

BÀI TẬP TỰ LUYỆN

Mức độ 1: NB

Bài Cho tứ giác ABCD Gọi M N, hình chiếu B đường thẳng ,

AC AD Chứng minh bốn điểm A B M N, , , nằm đường tròn HD: Chứng minh bốn điểm A B M N, , , cùng nằm đường trịn đường kính AB Bài Cho tam giác ABC có hai đường cao BD CE cắt tạiH

Chứng minh bốn điểm A D H E, , , nằm đường tròn (gọi tâm O) HD Chứng minh bốn điểm A D H E, , , cùng nằm đường tròn đường kính AB

Bài Cho tam giác ABC có ba góc nhọn nội tiếp đường trịn

(

O R;

)

Các đường cao BE CF cắt tạiH

Chứng minh: AEHF BCEFlà tứ giác nội tiếp đường tròn Hướng dẫn giải:

(26)

I

E

x M O

C B

A

II Phương pháp chứng minh “Chứng minh tứ giác có hai góc đối diện bù ( tổng hai góc đối

diện 180 ) CÁC VÍ DỤ Mức độ 1: NB

Câu 11: Hình chữ nhật; Hình thang cân; Hình bình hành Hình nội tiếp đường trịn? Chứng minh

Hướng dẫn giải

Ta có hình chữ nhật hình thang cân có tổng hai góc đối diện bù nên chúng nội tiếp đường tròn

Câu 12: Cho tứ giác ABCD cho: AD cắt BC M MA MD =MB MC Chứng minh tứ giác ABCD nội tiếp

Hướng dẫn giải

Xét hai tam giác MAB, MCD

Có  AMB=CMD MA MD MB MC MA MC

MB MD

= ⇒ = hay ∆MAB∆MCD hay

    o

180

MCD=MABDAB+BCD= hay tứ giác ABCD nội tiếp

Câu 13: Cho đường trịn

(

O R;

)

,đường kính AB DâyBC=R Từ B kẻ tiếp tuyến Bx với đường tròn Tia AC cắt Bxtại M Gọi E trung điểm AC

Chứng minh tứ giác OBME nội tiếp đường trịn Hướng dẫn giải

Ta có E trung điểm ACOEAC

BxAB ⇒ ABx=90onên tứ giác OBME nội tiếp Mức độ 2: TH

Câu 14: Cho đường tròn tâm O đường kínhAB Vẽ dây cung CD vng góc với AB I (I nằm A O ) Lấy điểm E cung nhỏ BC (E khác B C ),AE cắt CD F Chứng minh: BEFI tứ giác nội tiếp đường tròn

(27)

F

E

I O

D C

B A

Tứ giác BEFIcó: BIF=900(gt) BEF =BEA=900(góc nội tiếp chắn nửa đường tròn) Suy tứ giác BEFInội tiếp đường trịn đường kính BF

Câu 15: Cho đường trịn tâm O đường kínhAB, điểm M nửa đường tròn (M khác A, B ) Trên nửa mặt phẳng bờ AB chứa nửa đường tròn kẻ tiếp tuyến Ax Tia BM cắt Ax I ; tia phân giác góc IAM cắt nửa đường trịn E; cắt tia BM F tia BE cắt Ax tạiH, cắt AM K Chứng minh rằng: EFMK tứ giác nội tiếp

Hướng dẫn giải

X

2 1 2

1

E K I

H

F M

B O

A

Ta có: AMB=90o ( nội tiếp chắn nửa đường trịn ) ⇒KMF=90o (vì hai góc kề bù)  90o

AEB= ( nội tiếp chắn nửa đường trịn ) ⇒KEF =90o (vì hai góc kề bù)

  180o

KEF KMF

⇒ + = EFMKlà tứ giác nội tiếp

Câu 16: Cho đường trịn tâm O đường kínhAB, Kẻ tiếp tuyến Bx lấy hai điểm C D thuộc nửa đường tròn Các tia AC AD cắt Bx E, F (F B E)

1 Chứng minh:  ABD=DFB

2 Chứng minh CEFD tứ giác nội tiếp

D C

A O B

F E X

Hướng dẫn giải:

1) ∆ADB có ADB=90o ( nội tiếp chắn nửa đường trịn ) ⇒ABD +BAD=90o (vì tổng ba góc tam giác o

(28)

ABF

∆ có ABF =90o ( BF tiếp tuyến ).⇒ AFB+BAF =90o(vì tổng ba góc tam giác o

180 ) (2)

Từ (1) (2) ⇒ ABD=DFB

2) Tứ giác ACDB nội tiếp

( )

O ⇒ABD +ACD = 180o

  o

180 ECD ACD

⇒ + = ∠( Vì hai góc kề bù) ⇒ECD =DBA

Theo ABD =DFB, ECD=DBAECD =DFB Mà EFD +DFB = 180o ( Vì hai góc kề bù) nên   o

180 ECD AEFD

⇒ + = , tứ giác CEFD tứ giác nội tiếp Mức độ 3: VDT

Câu 17: Cho đường tròn

(

O R;

)

; AB CD hai đường kính khác đường trịn Tiếp tuyến B đường tròn

(

O R;

)

cắt đường thẳng AC, AD thứ tự E F

a) Chứng minh tứ giác ACBD hình chữ nhật b) Chứng minh ∆ACD∆CBE

c) Chứng minh tứ giác CDFE nội tiếp đường tròn Hướng dẫn giải

F E

O D

C

B A

a) Tứ giác ACBD có hai đường chéo AB CD cắt trung điểm đường, suy ACBD hình chữ nhật

b) Tứ giác ACBD hình chữ nhật suy CAD =BCE=900(1) Lại có CBE

2

= sđBC (góc tạo tiếp tuyến dây cung); ACD

= sđAD(góc nội tiếp), mà  

BC=AD(do BC= AD ) ⇒CBE =ACD(2) Từ (1) (2) suy ∆ACD∆CBE

c) Vì ACBDlà hình chữ nhật nên CB song song vớiAF, suy ra: CBE =DFE(3) Từ (2) (3) suy ACD =DFE tứ giác CDFE nội tiếp đường trịn

Câu 18: Cho nửa đường trịn đường kính BC=2R Từ điểm A nửa đường tròn vẽ AHBC Nửa đường trịn đường kínhBH , CH có tâm O1; O2 cắt AB CA thứ tự D E a) Chứng minh tứ giác ADHE hình chữ nhật, từ tính DE biết R=25 BH =10 b) Chứng minh tứ giác BDEC nội tiếp đường tròn

Hướng dẫn giải

a) Ta có BAC=90o(vì góc nội tiếpchắn nửa đường trịn) Tương tự có   o

BDH=CEH=90 D

A

(29)

Xét tứ giác ADHE có A  =ADH=AEH=90ohay ADHE hình chữ nhật Từ DE= AHAH2=BH CH (Hệ thức lượng tam giác vuông) hay AH2 =10.40=202

(

BH =10;CH =2.25 10− =40

)

DE=20

b) Ta có:BAH =  C  (góc có cạnh tương ứng vng góc) mà DAH =ADE (1)

(Vì ADHE hình chữ nhật) => C =ADE C BDE + =180o nên tứ giác BDEC nội tiếp đường tròn

Câu 19: Cho đường tròn

(

O R,

)

đường kính AB Các tia AC, AD cắt Bx E F (Fnằm B E)

Chứng minh CEFD tứ giác nội tiếp Hướng dẫn giải

D C

A O B

F E

X

thật vậy. ABD=BFD(1) (cùng phụ với DBF )

Mặt khác A B C D, , , nằm đường tròn nên  ECD=ABD(2) Từ (1) (2)     180o

ECD=BFDECD+EFD= hay CEFD tứ giác nội tiếp Mức độ 4: VDC

Câu 20: Cho ∆ABC cân A, I tâm đường tròn nội tiếp, K tâm đường trịn bàng tiếp góc A, O trung điểm củaIK Chứng minh bốn điểm B I C K, , , thuộc đường tròn tâm O

2

2

4

1

3

K I H

B C

A

O

Hướng dẫn giải:

Theo giả thiết ta có:B = B , B = B Mà    1    

0

1

(30)

Tương tự  

C + C = 90

Xét tứ giác BICK có  

B + C = 180 ⇒ bốn điểm B I C K, , , thuộc đường trịn tâm O đường kính IK

Câu 21: Cho tam giác ∆ABCvuông A

(

AB> AC

)

, đường cao AH Trên nửa mặt phẳng bờ BC chứa điểm A, vẽ nửa đường trịn đường kính BH cắt AB E, nửa đường trịn đường kính HC cắt

AC F Chứng minh:

1) Tứ giác AFHE hình chữ nhật

2) Tứ giác BEFC tứ giác nội tiếp đường tròn Hướng dẫn giải

o2 o1

o e

f

h

c b

a

Từ giả thiết suy

 

CFH = 90 , HEB = 90 (góc nội tiếp chắn nửa đường trịn) Trong tứ giác AFHE có: A=F=E= 90   o ⇒ AFHE hình chữ nhật

2) Vì AFHE hình chữ nhật ⇒ AFHEnội tiếp ⇒ AFE = AHE  (góc nội tiếp chắn AE ) (1)  Ta lại có AHE = ABH   (góc có cạnh tương ứng ⊥) (2)

Từ (1) (2)

⇒AFE = ABH mà   CFE + AFE = 180  0⇒ CFE + ABH = 180   Vậy tứ giác BEFC nội tiếp Câu 22: Cho nửa đường trịn tâm O đường kính AB C điểm nằm O A Đường thẳng vng góc với AB C cắt nửa đường tròn I K điểm nằm đoạn thẳng CI (K khác C I ), tia AK cắt nửa đường tròn

( )

O M, tia BM cắt tia CI D Chứng minh:

1) ACMD tứ giác nội tiếp đường tròn 2) ∆ABD ~∆MBC

3) AKDE tứ giác nội tiếp Hướng dẫn giải

E

D

M I

C K

O B

(31)

1) Ta có: 

AMB=90 (góc nội tiếp chắn nửa đường trịn)⇒AMD=900 Tứ giác ACMD

 

AMD=ACD=90 , suy ACMD nội tiếp đường trịn đường kính AD

2) ∆ABDMBC có: B chung BAD =BMC (do ACMDlà tứ giác nội tiếp) Suy ra: ∆ABD ~∆MBC (g – g)

3) Lấy E đối xứng với B qua C E cố định EDC =BDC, lại có: BDC =CAK (cùng phụ với B), suy ra: EDC =CAK Do AKDE tứ giác nội tiếp

III Phương pháp chứng minh: “Chứng minh hai đỉnh nhìn đoạn thẳng tạo hai điểm cịn lại hai góc nhau”

CÁC VÍ DỤ Mức độ 1: NB

Câu 23: Cho tam giác ABC,lấy điểm Dthay đổinằm cạnh BC (D không trùng với B )

C Trên tia AD lấy điểm P cho D nằm A P đồng thời

DA DPDB DC Đường tròn

 

T qua hai điểm A D, cắt cạnh AB AC, F E Chứng minh rằng: Tứ giác ABPC nội tiếp

1

1

1 1 1

2

P

H K

F

E

D C

B

A

Hướng dẫn giải:

Ta có DA DP DB DC DA DC DB DP

= ⇒ = mà  ADB=CDP nên hai tam giác ADB CDP, đồng dạng Suy ra, DAB DCP  Tứ giác ABPC nội tiếp

(32)

Hướng dẫn giải

H

O P

K I

M

C B

A

Ta có:AIM =AKM=900(gt), suy tứ giác AIMK nội tiếp đường tròn đường kính AM Câu 25: Cho đường trịn

( )

O có đường kính AB Lấy điểm M thuộc đoạn thẳng OA, điểm N thuộc

nửa đường tròn

( )

O Từ A B vẽ tiếp tuyến Ax By Đường thẳng qua N vng góc với MN cắt Ax By thứ tự C D Chứng minh ACNM BDNM tứ giác nội tiếp đường tròn

Hướng dẫn giải:

K I

y x

D C N

M O B

A

Tứ giác ACNM có: MNC =90o(gt) MAC =90o( tínhchất tiếp tuyến)

ACNM tứ giác nội tiếp đường trịn đường kínhMC Tương tự tứ giác BDNM nội tiếp đường trịn đường kính MD

Mức độ 2: TH

Câu 26: Từ điểm A nằm ngồi đường trịn

(

O R;

)

ta vẽ hai tiếp tuyếnAB, AC với đường tròn ( B, C tiếp điểm) Trên cung nhỏ BC lấy điểm M , vẽMIAB, MKAC (IAB K, ∈AC )

(33)

H

O P

K I

M

C B

A

a) Ta có:AIM =AKM=900(gt), suy tứ giác AIMK nội tiếp đường trịn đường kính AM b) Tứ giác CPMK có MPC =MKC=900(gt) Do CPMKlà tứ giác nội tiếp⇒MPK =MCK(1)

KC tiếp tuyến

( )

O nên ta có: MCK =MBC (cùng chắn MC ) (2)  Từ (1) (2) suy MPK =MBC(3)

Chứng minh tương tự câu b ta có BPMI tứ giác nội tiếp

Câu 27: Cho đường tròn

(

O R;

)

có đường kính AB Vẽ dây cung CD vng góc với AB (CD khơng qua tâm O ) Trên tia đối tia BA lấy điểm S; SC cắt

(

O R;

)

điểm thứ hai M Gọi H giao điểm MA BC; K giao điểm MD AB Chứng minh BMHK tứ giác nội tiếp

Hướng dẫn giải:

ABCD nên AC =AD

Suy MHB =MKB (vì 1(sdAD sdMB) 

2 + ⇒ tứ giác BMHKnội tiếp đường tròn

Câu 28: Cho đường trịn

( )

O có đường kính AB Lấy điểm M thuộc đoạn thẳng OA, điểm N thuộc nửa đường tròn

( )

O Từ A B vẽ tiếp tuyến Ax By Đường thẳng qua N vng góc với MN cắt Ax By thứ tự C D

(34)

c) Gọi I giao điểm AN CM, K giao điểm BN DM Chứng minh IMKN tứ giác nội tiếp

Hướng dẫn giải:

K I

y x

D C N

M O B

A

Tứ giác ACNM có: MNC =90o(gt) MAC =90o( tínhchất tiếp tuyến)

ACNM tứ giác nội tiếp đường trịn đường kínhMC Tương tự tứ giác BDNM nội tiếp đường trịn đường kính MD

b) ∆ANB ∆CMD có:  

ABN=CDM(do tứ giác BDNM nội tiếp)  

BAN=DCM(do tứ giác ACNM nội tiếp) ⇒ ∆ANB ∆CMD (g.g)

c) ∆ANB ∆CMD ⇒CMD =ANB=90o (do ANBlà góc nội tiếp chắn nửa đường trịn (O)) Suy IMK =INK=90o ⇒ IMKN tứ giác nội tiếp đường trịn đường kính IK

Mức độ 3: VDT

Câu 29: Cho hình vng ABCD có hai đường chéo cắt E Lấy I thuộc cạnh AB, M thuộc cạnh BC cho: IEM=900( I M khơng trùng với đỉnh hình vng )

a) Chứng minh BIEM tứ giác nội tiếp đường trịn b) Tính số đo góc IME 

c) Gọi N giao điểm tia AM tia DC; K giao điểm BN tia EM Chứng minBKCE tứ giác nội tiếp

Hướng dẫn giải

I

E M

N

B C

A D

K

(35)

b) Tứ giác BIEM nội tiếp suy ra: IME =IBE=450(do ABCD hình vng) c) ∆EBIECMBE=CE, BEI =CEM( IEM =BEC=900)

⇒ ∆EBI =∆ECM (g-c-g) ⇒ MC=IBMB=IACN/ / BA nên theo định lí Thalet, ta có: MA MB

MN = MC= IA

IB Suy IM / /BN (định lí Thalet đảo)

 

BKE IME 45

⇒ = = (2) Lại có 

BCE=45 (do ABCD hình vng) Suy BKE =BCE⇒ BKCE tứ giác nội tiếp

Câu 30: Cho đường tròn

( )

O với dây BC cố định điểm A thay đổi cung lớn BC cho AC> AB AC>BC Gọi D điểm cung nhỏ BC Các tiếp tuyến

( )

O D C cắt E Gọi P, Q giao điểm cặp đường thẳng AB với

CD; AD với CE

1) Chứng minh rằng: DE/ /BC

2) Chứng minh tứ giác PACQ nội tiếp đường tròn Hướng dẫn giải

q o

p

e d

c b

a

1) CDE DC BD = BCD 

1

2 =

= ⇒DE/ /BC

2) APC (AC - DC) = AQC   2

=

PACQ nội tiếp đường trịn (vì APC = AQC )  

Câu 31: Cho tam giác ABC có   90

C< <B , đường cao AH trung tuyến AM a) Chứng minh 

90

BAC= BAH =MAC

(36)

M H

B

A C

N

Ta có: BAH =BCA (cùng phụ với ABC)  

MCA=MAC(Tam giác MAC cân M theo tính chất trung tuyến tam giác vuông) Suy BAH =MAC

b) Giả sử tam giác ABC tam giác vuông Kẻ đường cao CN tam giác ABC

Ta có MAC =BAH (giả thiết)  

BAH =BCN (cùng phụ với ABC)  

MCN =MNC (Tam giác MNC cân N )

Suy MAC =MNC Do ACMN tứ giác nội tiếp mà

 

90 90

ANC = ⇒ AMC= ⇒HM

Suy tam giác ABC cân (mâu thuẫn giả thiết)

Vậy BAH =MAC tam giác ABC tam giác vuông Mức độ 4: VDC

Câu 32: Cho tứ giác ABCD có hai đỉnh B C nửa đường tròn đường kính AD, tâm O Hai đường chéo AC BD cắt E Gọi H hình chiếu vng góc E xuống AD I trung điểm DE Chứng minh rằng:

1) Các tứ giác ABEH, DCEH nội tiếp đường tròn 2) E tâm đường tròn nội tiếp tam giác BCH

3) Năm điểm B C I O H, , , , thuộc đường tròn Hướng dẫn giải

I O H E

D C

B

A

(37)

Tương tự, tứ giác DCEHcó C = H = 90  o, nên nội tiếp

2) Trong tứ giác nội tiếpABEH , ta có: EBH = EAH   (cùng chắn cung EH )  Trong

( )

O ta có: EAH = CAD = CBD    (cùng chắn cung CD ) 

Suy ra: EBH = EBC , nên   BE tia phân giác góc HBC 

Tương tự, ta có: ECH = BDA = BCE , nên    CE tia phân giác góc BCH  Vậy E tâm đường tròn nội tiếp tam giác BCH

3) Ta có I tâm đường trịn ngoại tiếp tam giác vng ECD, nên BIC = 2EDC   (góc nội tiếp góc tâm chắn cung EC ) Mà  EDC = EHC , suy   BIC = BHC  

+ Trong

( )

O , BOC = 2BDC = BHC    (góc nội tiếp góc tâm chắn cung BC )  Hay năm điểm B C I O H, , , , thuộc đường tròn

Câu 33: Cho hình vng ABCD có hai đường chéo cắt tạiE Lấy I thuộc cạnh AB, M thuộc cạnh BC cho: 

IEM=90 (I M khơng trùng với đỉnh hình vng ) a) Chứng minh BIEM tứ giác nội tiếp đường trịn

b) Tính số đo góc IME 

c) Gọi N giao điểm tia AM tia DC; K giao điểm BN tia EM Chứng minh BKCElà tứ giác nội tiếp, từ suy : CK ⊥ BN

Hướng dẫn giải

I

E M

N

B C

A D

K

a) Tứ giác BIEM có: 

IBM=IEM=90 (gt); suy tứ giác BIEM nội tiếp đường trịn đường kính IM

b) Tứ giác BIEM nội tiếp suy ra: IME =IBE=450(do ABCDlà hình vng) c) ∆EBIECM có: 

(38)

(

)

EBI ECM g c g MC IB MB IA

∆ = ∆ ⇒ = ⇒ =

⇒ Vì CN / /BA nên theo định lí Thalet, ta có: MA MB

MN = MC= IA

IB Suy MI/ /BN (định lí Thalet đảo)

 

BKE IME 45

⇒ = = (2) Lại có 

BCE=45 (do ABCD hình vng) Suy BKE =BCE⇒ BKCE tứ giác nội tiếp

Suy ra: BKC BEC 180 + = 0mà BEC =900; suy BKC =900; hay CK ⊥ BN

Câu 34: Cho tam giác nhọn ABC nội tiếp

( )

O , đường cao BD, CE cắt H

(

DAC E; ∈AB

)

Kẽ đường kính BK, Kẽ CPBK

(

PBK

)

a) Chứng minh BECD tứ giác nội tiếp

b) Chứng minh EDPC tứ giác nội tiếp, từ suy ED=CP ( trích HK2-Sở bắc ninh 2016-2017)

Hướng dẫn giải

Do E D P, , nhìn BC góc vng nên B E D P C, , , , nằm đường trịn đường kính BC

(39)

Chủ đề 2: CHỨNG MINH HAI TAM GIÁC BẰNG NHAU 1 PHƯƠNG PHÁP CHỨNG MINH

a) Khái niệm:

ABC

A 'B'C' khi

A

A ';

B

B';

C

C'

AB

A 'B'; BC

B'C'; AC

A 'C'

∠ = ∠

∠ = ∠

∠ = ∠

= ∆

=

=

=

b) Các trường hợp hai tam giỏc: c.c.c; c.g.c; g.c.g

c) Các trường hợp hai tam giác vng: hai cạnh góc vng; cạnh huyền cạnh góc vng; cạnh huyền gúc nhọn

d) Hệ quả: Hai tam giác đường cao; đường phân giác; đường trung tuyến tương ứng

2 CÁC VÍ DỤ Mức độ 1: NB

Câu 1: Trong hình sau tam giác (Các cạnh đánh dấu kí hiệu giống nhau) Kể tên đỉnh tương ứng tam giác Viết kí hiệu tam giác

300 300

800 800

B

A C

I N

M

600

600 40

0

400 80

0

800

Q

H

P

R

Hướng dẫn giải

• Xem hình a) ta có:    80 , 30 A= =IC=N = ?  

(

0

)

180 80 30 70

B=M = − + =

AB=MI AC, =IN BC, =MN nên ∆ABC= ∆IMN

• Xem hình b) ta có: Q 2 =R2 =800 ( vị trí so le trong) nên QH/ /RP Nên   1 60 R =Q = (so le trong)  P=H =400 QH =RP HR, =PQ, QR chung nên ∆HQR= ∆PRQ Câu 2: Cho ∆ABC=∆HIK

(40)

Hướng dẫn giải

• Ta có ∆ABC= ∆HIK, nên cạnh tương ứng với BC cạnh IK.Góc tương ứng với góc H góc A

• ∆ABC= ∆HIK Suy AB=HI AC, =HK BC, =IK   A=H B; =I C; =K

Câu 3: Cho tam giác ABC nội tiếp đường tròn (O) Trên cạnh AB lấy điểm N (N khác A B), cạnh AC lấy điểm M cho BN = AM Gọi P giao điểm BM CN Chứng minh ∆BNC= ∆AMB

Hướng dẫn giải

A

B C

M

N

Gợi ý:∆BNC ∆AMB có: BN=AM (gt) GócNBC= gócMAB

BC=AB (vì ∆ABC tam giác đều) ⇒ ∆BNC= ∆AMB

Mức độ 2: TH

Câu 4: Cho đường trịn (O) đờng kính AB = 2R C điểm thuộc đường tròn (CA;CB ) Trên nửa mặt phẳng bờ AB có chứa điểm C , kẻ tia Ax tiếp xúc với đờng tròn (O), gọi M điểm cung nhỏ AC Tia BC cắt Ax Q , tia AM cắt BC N

a) Chứng minh tam giác BAN MCN cân b) Chứng minh ∆ MCB = ∆ MNQ

Hướng dẫn giải

Q

N

M

O C

(41)

a) Xét ∆ABMNBM

Ta có: AB đờng kính đờng trịn (O) nên :AMB = NMB = 90o

M điểm cung nhỏ AC nên ABM = MBN => BAM = BNM => ∆BAN cân đỉnh B

Tứ giác AMCB nội tiếp

=> BAM = MCN ( bù với góc MCB) => MCN = MNC ( góc BAM) => Tam giác MCN cân đỉnh M

b) Xét ∆ MCBMNQ có :

MC = MN (theo cm MNC cân ) ; MB = MQ ( theo gt)

∠ BMC =∠ MNQ ( : ∠MCB = ∠MNC ; ∠MBC = ∠MQN ) => ∆ MCB =∆ MNQ (c.g.c)

Câu 5:Cho đường tròn (O;R) điểm A cho OA = R Vẽ tiếp tuyến AB, AC với đường trịn Một góc ∠xOy = 450 cắt đoạn thẳng AB AC D E

Chứng minh rằng: DE tiếp tuyến đường tròn ( O )

Hướng dẫn giải

B

M A

O

C D

E Áp dụng định lí Pitago tính AB = AC = R ⇒ ABOC hình vng

Kẻ bán kính OM cho ∠BOD = ∠MOD⇒

∠MOE = ∠EOC

Chứng minh ∆BOD = ∆MOD

⇒ ∠OMD = ∠OBD = 900 Tương tự: ∠OME = 900

⇒D, M, E thẳng hàng Do DE tiếp tuyến đường tròn (O)

Câu 6: Chứng minh rằng: Nếu hai cạnh trung tuyến thuộc cạnh thứ ba tam giác hai cạnh trung tuyến thuộc cạnh thứ ba tam giác hai tam giác Giải

GT ABC, A'B'C': AB = A'B', AC= A'C' M∈BC: MB=MC M'∈B'C': M'B'=M'C' AM=A'M'

(42)

A

B C

D

A'

C'

D' M'

M B'

2

1

Hướng dẫn giải

Lấy DAM MD: =MA

Lấy D A M' ' ' : 'M D'=M A' ' Xét ABM DMC có: MB=MC(gt)

AMB=CMD(đối dỉnh) ⇒ABM DMC(c.g.c) AM = MD(cách lấy điểm D)

⇒CD= AB( hai cạnh tương ứng) Và A2 =D1(1)( hai góc tương ứng) C/m tương tự ; C'D'=A'B'; A'2=D'1(2) Xét ACD A'C'D' có:

AC = A'C'(gt)

AD=A'D'(vì AM=A'M') ⇒ACD = A'C'D'(c.g.c) CD=C'D'(=AB)

⇒ A1=A'1vàD1=D'1(3)

Từ (1), (2),(3) ⇒ A2=A'2mà A1=A'1 ⇒BAC=B A C' ' ' Vậy ABC=A'B'C'(c.g.c)

* cách 2:

AMC A'M'C' có: AM=A'M'(gt)

A =A'1(cmt) ⇒AMC = A'M'C'(c.g.c) AC= A'C'(gt)

⇒MC = M'C'( hai cạnh tương ứng) Mà MC =

2BC; M'C' =

2B'C'(gt) Do đó: BC=B'C' Vậy ABC=A'B'C'(c.c.c)

Mức độ 3: VD

Câu 7: Cho tam giác ABC cân A,  20

A= Trên cạnh AB lấy điểm D choAD=BC Chứng minh góc  

2 DCA= A

Hướng dẫn giải

Giải: Tam giác ABC cân A,  20 A=

suy  

(

)

0

0 180 20

80

B= =C − =

(43)

Suy  

0 20

10

MAB=MAC= =   0

80 60 20 ABM = ACM = − =

Tam giác CAD tam giác ACM có AD=CM (c/m trên)  

20 CAD=ACM =

AC chungVậy tam giác CAD = tam giác ACM (c.g.c) suy   10

DCA=MAC= ^,

 1

2 DCA= BAC

Câu 8: Cho hình thang ABCD (AB // CD, AB<CD) Gọi K, M trung điểm BD, AC Đường thẳng qua K vng góc với AD cắt đường thẳng qua M vng góc với BC Q Chứng minh:

a)KM / /AB b)QD=QC

Hướng dẫn giải

A I B

K

M

D E H R C

Q

Gọi I trung điểm AB, E=IKCD R, =IMCD Xét hai tam giác KIB KED có:  

ABD=BDC, KB=KD (K trung điểm BD), IKB =EKD Suy ∆KIB= ∆KEDIK =KE

Chứng minh tương tự có: ∆MIA= ∆MRC Suy ra: MI = MR

Trong tam giác IER có IK =KE MI =MR nên KM đường trung bình ⇒ KM / /CD Do CD/ /AB (gt) KM // AB (đpcm)

Ta có: IA=IB, KB=KD (gt)⇒ IK đường trung bình ∆ABD ⇒ IK//AD hay IE//AD chứng minh tương tự ∆ABC có IM//BC hay IR//BC

Có: QKAD(gt), IE//AD (CM trên) ⇒QKIE Tương tự có QMIR

Từ có: IK=KE, QKIEQK trung trực ứng với cạnh IE ∆IER Tương tự QM trung trực thứ hai ∆IER

Hạ QHCD suy QH trung trực thứ ba ∆IER hay Q nằm trung trực đoạn CD ⇒ Q cách C D hay QD=QC (đpcm)

Câu 9: Cho đường trịn tâm O đường kính AB CD vng góc với nhau, lấy điểm I đoan CD a) Tìm điểm M tia AD, điểm N tia AC cho I lag trung điểm MN

(44)

Hướng dẫn giải

K O

N

M

I

D C

B A

a) Dựng (I, IA) cắt AD M cắt tia AC N Do MAN = 900 nên MN đường kính Vậy I trung điểm MN

b) Kẻ MK/ /AC ta có : ΔINC=ΔIMK (g.c.g)

CN MK MD

=> = = (vì ΔMKD vng cân)

Vậy AM +AN =AM +CN+CA=AM +MD CA+

AM AN AD AC

(45)

Chủ đề 3: CHỨNG MINH HAI TAM GIÁC ĐỒNG DẠNG

A PHƯƠNG PHÁP CHỨNG MINH Hai tam giác bất kỳ:

1 Dùng định lý: đường thẳng song song với cạnh cắt cạnh lại tam giác Trường hợp: c – c – c

3 Trường hợp: c – g – c Trường hợp: g – g

Hai tam giác vuông:

1 Trường hợp: g – g Trường hợp: c – g – c

3 Trường hợp: cạnh huyền – cạnh góc vng

B VÍ DỤ MINH HỌA

Mức độ 1: Nhận Biết

Câu 1. Cho tam giác ABC vuông A, đường cao AH Chứng minh AH2 =HB HC Lời giải:

Xét hai tam giác ∆AHB ∆CHAta có:

90

AHB= ∠CHA=

ABH = ∠CAH( phụ ∠ACH)

⇒ ∆AHB∽∆CHA(g-g)

AH = HB

CH HA

=

AH HB HC

Câu 2.Cho ∆ABC có ba góc nhọn, đường cao AH

(

HBC

)

Vẽ HD vng góc AB D, HE vng góc AC E Chứng minh ∆ AHB đồng dạng với ∆ ADH

Lời giải:

Ta có AH HD đường cao ∆ABC ∆BHA nên ta có ∆ AHB ∆DHAlà tam giác vuông

Xét hai tam giácvng ∆ AHBADH có ∠HAB= ∠DHA

⇒ ∆ AHB∽∆ ADH

Câu 3.Cho∆ABC có AB = cm, AC = cm, BC = cm Đường phân giác góc A cắt cạnh BC D Qua D vẽ đường thẳng vng góc với BC cắt AC E BA K

Chứng minh ∆EDC đồng dạng ∆BDK Lời giải:

Xét ∆ABC có

2 2

2 2

25, 16 25

= + = + =

⇒ = +

BC AB AC

BC AB AC

A

B

C

H

A

B

C H

D

E

E

(46)

⇒ ∆ABCvuông A

Ta có 9000

90 ∠ + ∠ =  ⇒ ∠ = ∠  ∠ + ∠ =  ECD KBC KBC DEC ECD DEC

Xét hai tam giác vng ∆EDC ∆BDK có∠DBK = ∠DEC ⇒ ∆EDC∽∆BDK ( g-g)

Mức độ 2: Thông Hiểu

Câu Tuyển sinh vào 10 Bình Dương 2010-2011 Một hình vng ABCD nội tiếp đường trịn Tâm O bán kính R Một điểm M di động cung ABC, M không trùng với A, B C, MD cắt AC H Chứng minh tam giác MDC đồng dạng với tam giác

MAH Lời giải:

Xét hai tam giác ∆MDCvà ∆MAH có

MDC = ∠MAH(góc nội tiếp chắn cung MC ) (1)

AMH = ∠DMC(góc nội tiếp chắn hai cung  AD=CD) (2) Từ (1) (2) ⇒ ∆MDC∽∆MAH ( g-g)

Câu Tuyển sinh vào 10 Nghệ An 2011-2012.Cho điểm A nằm ngồi đường trịn (O) Từ A kẻ hai tiếp tuyến AB, AC cát tuyến ADE tới đường tròn (B, C hai tiếp điểm; D nằm A E) Gọi H giao điểm AO BC Chứng minh AH.AO = AD.AE

Lời giải:

Xét tam giác cân ABC có AO phân giác đường cao Áp dụng hệ thức lượng vào tam giácvng ACO ta có

2

=

AC AH AO (1)

Xét hai tam giác ∆ADCvà ∆ACE có

D A

CA = ∠E C

D

DCA= ∠CE (góc tiếp tuyến dây cung số đo góc nội tiếp)

⇒ ∆ADC∽∆ACE ( g-g)

⇒ E

D E D = ⇒ =

AC A

AC A A

A AC (2)

Từ (1) (2) ⇒ AH.AO = AD.AE (đpcm)

Câu 3.Tuyển sinh vào 10 ĐĂK LẮK 2012-2013.Cho tam giác ABC có ba góc nhọn nội tiếp đường trịn tâm O (AB < AC) Hai tiếp tuyến B C cắt M AM cắt đường tròn (O) điểm thứ hai D E trung điểm đoạn AD EC cắt đường tròn (O) điểm thứ hai F Chứng minh

=

MB MA MD Lời giải:

Xét hai tam giác ∆MBAvà ∆MDB có

(47)

D

MB = ∠AMB(góc tiếp tuyến dây cung số đo góc nội tiếp) ⇒ ∆MBA∽∆MDB ( g-g)

D MD = ⇒ =

MB MA

MB MA M

MB (đpcm)

Mức độ 3: Vận Dụng Thấp

Câu 1.Tuyển sinh vào 10 Trà Vinh 2015-2016.Từ điểm M đường tròn O, vẽ hai tiếp tuyến MA, MB đến đường tròn (A, B hai tiếp điểm) Qua A vẽ đường thẳng song song với MB, cắt đường tròn E; đoạn thẳng ME cắt đường tròn F Hai đường thẳng AF MB cắt I.Chứng minh

=

IB IF IA Lời giải:

Xét hai tam giác ∆BIFvà ∆AIB có ∠BIF = ∠AIB

IAB= ∠IBF(góc tiếp tuyến dây cung số đo góc nội tiếp)

⇒ ∆BIF∽∆AIB( g-g)

= ⇒ =

IB IF

IB IA IF

IA IB (đpcm)

Câu Tuyển sinh vào 10 Thái Bình 2015-2016.Cho nửa đường trịn tâm O đường kính AB = 2R Điểm M di chuyển nửa đường tròn (M khác A B) C trung điểm dây cung AM Đường thẳng d tiếp tuyến với nửa đường tròn B Tia AM cắt d điểm N Đường thẳng OC cắt d E.Chứng minh: tứ giác OCNBnội tiếp AC.AN=AO.AB

Lời giải:

+ Ta có OC vng góc MA ( đường kính qua trung điểm dây cung vng góc dây cung)

Xét tứ giác OCNB có

90

OCN = ∠OBN =

Nên tứ giác OCNB nội tiếp (tổng hai góc đối 180 ) + Xét hai tam giác ∆ANOvà ∆ABC có

O

AN = ∠ABC( chắn cung CO)

NAO= ∠BAC

⇒ ∆ANO∽∆ABC ( g-g)

AN = AOAC AN = AO AB

AB AC (đpcm)

Câu Tuyển sinh vào 10 Hà Nội 2015-2016.Cho nửa đường trịn tâm O có đường kính AB Lấy điểm C đoạn thẳng AO (C khác A, C khác O) Đường thẳng

qua C vuông góc với AB cắt nửa đường trịn K Gọi M điểm cung KB (M khác K, M khác B) Đường thẳng CK cắt đường thẳng AM, BM H D Đường thẳng BH cắt nửa đường tròn điểm thứ hai N

1) Chứng minh tứ giác ACMD tứ giác nội tiếp 2) Chứng minh CA.CB=CH.CD

N

D

H

K

M

I

F

E

O

M

A

B

E

N

C

A

O

B

(48)

Lời giải:

1) Xét tứ giác ACMD có

D 90

AC = ∠DMA=

Nên tứ giác ACMD nội tiếp ( đỉnh M, C nhìn AD góc khơng đổi )

2) Xét hai tam giácvng ∆HACvà ∆B CD có D

HAC = ∠B C(cùng phụ ∠ABD) ⇒ ∆HAC∽∆B CD ( g-g)

⇒ D

D = ⇒ =

CA CH

CA CB CH C

C CB (đpcm)

Mức độ 4: Vận Dụng Cao

Câu Cho ∆ABC có ba góc nhọn nội tiếp đường tròn tâm O.Gọi M điểm cung nhỏ AC.Gọi E F chân đường vng góc kẻ từ M đến BC AC.P trung điểm AB, Q trung điểm FE

1) Chứng minh tứ giác MFEC nội tiếp 2) Chứng minh BM EF =BA EM 3) Chứng minh∆AMP∽∆FMQ Lời giải:

1) Chứng minh tứ giác MFECnội tiếp:

E; F nhìn đoạn thẳng CM góc

90 nên tứ giác MFECnội tiếp

2) Chứng minh BM EF =BA EM •Chứng minh ∆EFM ∽∆BAM :

Ta có góc ∠ABM = ∠ACM (vì chắn cung AM) Do MFEC nội tiếpnên góc ∠ACM = ∠FEM (cùng chắn cung FM)

⇒∠ABM = ∠FEM (1)

Ta lại có góc ∠AMB = ∠ACB (Cùng chắn cung AB).Do MFEC nội tiếp nên

FME FCM

∠ = ∠ (Cùng chắn cung FE).⇒Góc ∠AMB = ∠FME (2) Từ (1) và(2) suy ∆EFM ∽∆BAM

BM = EMBM EF =

BA EF BAEM (đpcm)

3) Chứng minh∆AMP ∽∆FMQ

Ta có ∆EFM∽∆ABM (theo c/m trên)⇒ AB = AM

FE MFAB =2AP FE; 2= FQ (gt) ⇒2

2 = ⇒ =

AP AM AP AM

FQ MF FQ FM góc ∠PAM = ∠MFQ (suy từ∆EFM ∽∆BAM ) Vậy∆AMP∽∆FMQ

Câu Cho nửa đường trịn (O) đường kính AB,bán kính OC⊥AB.Gọi M điểm cung BC.Kẻ đường cao CH tam giác ACM

1 Chứng minh AOHC nội tiếp

2 Chứng tỏ ∆CHM vuông cân OH phân giác góc COM

3 Gọi giao điểm OH với BC I.MI cắt (O) D.Cmr:CDBM hình thang cân

4 BM cắt OH N.Chứng minh ∆BNI ∆AMC đồng dạng,từ suy ra: BN.MC=IN.MA

Q

P

F

E

O

A

B

(49)

Lời giải:

1 Chứng minhAOHC nội tiếp:học sinh tự chứng minh

2 •Chứng minh ∆CHM vuông cân:

Do OC⊥AB trại trung điểm O⇒S ACd =S CBd = 90o . Ta lại có:

45

o CMA s AC

∠ = đ = ⇒∆CHM vuông cân M •Chứng minhOH phân giác góc∠COM :

Do ∆CHMvuông cân H⇒CH =HM CO OB; = (bán kính); OH chung⇒∆CHO=∆HOM ⇒∠COH = ∠HOM ⇒đpcm 3.Chứng minh CDBM hình thang cân:

Do ∆OCM cân O có OH phân giác⇒OH đường trung trực CM mà I∈OH⇒∆ICM cân I⇒∠ICM = ∠IMC mà ∠ICM = ∠MDB (cùng chắn cung BM)

⇒∠IMC = ∠IDB hayCM / /DB Do ∆IDB cân I⇒∠IDB = ∠IBDMBC = ∠MDC (cùng chắn cungCM) nên ∠CDB = ∠MBD ⇒CDBM làhìnhthang cân

4 •Chứng minh BNI ∽AMC:

Do OH đường trung trực CM N∈OH ⇒CN=NM Do ∠AMB =900 ⇒ ∠HMB =900 hay NM⊥AM

mà CH⊥AM⇒CH//NM

Có ∠CMH = 45o ⇒ ∠NHM =45o ⇒MNH vng cân M CHMN là hình vng⇒ ∠INB = ∠CMA =45o .

•Do CMBD thang cân⇒CD =BMCD =BM

AC =CB ⇒ AD CM= ∠CAM = ∠CBM(cùng chắn cungCM ) ⇒INB =CMA

Câu Cho hình bình hành ABCD

(

AC< BD

)

Gọi E F hình chiếu B lên đường thằng DA DC Chứng minh rằng:

= +

DB DA DE DC DF

a) Phân tích tìm hình phụ

Giả sử lấy điểm M thuộcđoạn BD cho =

DM DB DA DE

DB DA DE

DM

= , kết hợp với góc D chung để suy ∆MDA∽∆EDB ,

vì 0

D

90 90

= ⇒

BEDAM = hayAMDB

Do điềm M cần tìm hình chiếu A lên DB

b) Bài giải

Qua A kẻ đường thẳng vng góc với BD cắt BD M, AC <BD nên góc∠ADB ∠ABD nhọn nênM nằm D B , BD=DM +MB

N

D

I

H

C

A

O

B

M

E

F

M

C

A

D

(50)

- Dễ chứng minh ∆DAM ∽∆DBE (g-g), suy

DB DA DE

DM

= ⇒ DM DB =DA DE

( )

1 -Lại có ∆DBF∽∆BAM (vì

90

∠ = ∠F M = ∠B FD = ∠ABM (so le trong)) ⇒

DB AB DF BM

= ⇒BM AB = AB DF

( )

2

- Cộng vế theo vế hệ thức (1) (2) ta được:

= +

DB DA DE DC DF ⇒ đpcm

(51)

Chủ đề 4: CHỨNG MINH HAI ĐOẠN THẲNG BẰNG NHAU 1 PHƯƠNG PHÁP CHỨNG MINH

Phương pháp 1:

Chứng minh hai đoạn thẳng hai cạnh tương ứng hai tam giác

Phương pháp 2:

Sử dụng tính chất hai đường chéo hình bình hành, hình chữ nhật, hình vng cắt trung điểm đường

Phương pháp 3:

Vận dụng tính chất hai cạnh bên tam giác cân

Phương pháp 4:

Vận dụng tính chất ba cạnh tam giác

Phương pháp 5:

Vận dụng cạnh đối hình bình hành, hình chữ nhật, hình thoi, hình vng

Phương pháp 6:

Chứng minh hai đoạn thẳng đoạn thẳng thứ ba

Phương pháp 7:

Chứng minh hai đoạn thẳng hai cạnh bên hình thang cân

Phương pháp 8:

Trong đường tròn hai đường tròn nhau, hai dây căng hai cung

Phương pháp 9:

Trong đường tròn hai đường tròn nhau, hai dây cách tâm

Phương pháp 10:

Vận dụng định lí, đường thẳng qua trung điểm cạnh tam giác song song với cạnh thứ hai qua trung điểm cạnh thứ ba

Phương pháp 11:

Vận dụng định nghĩa đường trung trực đoạn thẳng, định nghĩa trung điểm đoạn thẳng, định nghĩa đường trung tuyến tam giác

Phương pháp 12:

Chứng minh hai đoạn thẳng thẳng có số đo

Phương pháp 13:

Chứng minh hai đoạn thẳng đoạn thẳng thứ ba

Phương pháp 14:

Chứng minh hai đoạn thẳng tổng, hiệu, trung bình nhân,…, hai đoạn thẳng đôi

Phương pháp 15:

Sử dụng tính chất trung tuyến ứng với cạnh huyền, tính chất cạnh đối diện với góc

0

(52)

Phương pháp 16:

Sử dụng tính chất đường phân giác góc

Phương pháp 17:

Sử dụng tính chất hai đoạn thẳng song song chắn hai đường thẳng song song

Phương pháp 18:

Chứng minh phản chứng

Phương pháp 19:

Sử dụng đoạn thẳng cho trước biến đổi

Phương pháp 20:

Sử dụng định lí đường trung bình tam giác (thuận đảo)

Phương pháp 21:

Sử dụng tính chất trọng tâm tam giác (tính chất giao điểm ba đường phân giác tam giác), tínhchất giao điểm ba đường trung trực

Phương pháp 22:

Sử dụng bình phương chúng (có thể sử dụng định lí Pitago, tam giác đồng dạng, hệ thức lượng tam giác, đường trịn để đưa bình phương chúng nhau)

2 CÁC VÍ DỤ

Mức độ 1: NB

Câu 1:ABC cân A Vẽ đường tròn

O R;

tiếp xúc với AB AC, B C, Đường thẳng qua

điểm M BC vng góc với OM cắt tia AB AC, D E,

a) Chứng minh điểm O B D M, , , thuộc đường tròn b) Chứng minh MDME

Hướng dấn giải

a) Ta có: DBO DMO 900 (vì gt)

 điểm B M, thuộc đường trịn đường kính DO

 đpcm

b) Chứng minh tương tự có điểm O C E M, , ,

thuộc đường trịn  MEO MCO (vì góc nội tiếp cùngchắn cung MO)

 

MBOMDO (vì góc nội tiếp chắn cung MO)

MBO MCO (vìBOC cân O)

MEO MDO  DOE cân O

MODE nên MD ME (đpcm)

Câu 2: Cho đường trịn tâm O, đường kính AC 2 R Từ điểm E đoạn OA (E

không trùng với A O ) Kẻ dây BD vuông góc với AC Kẻ đường kính DI

đường tròn ( )O

E

D

A

(53)

a) Chứng minh rằng: ABCI

b) Chứng minh rằng: EA2 EB2 EC2 ED2 4 R2 Tính diện tích đa giác ABICD theo R

3

R OEHướng dấn giải

a) Chứng minh rằng: ABCI

Ta có: BDAC (gt)

 900

DBI  ( góc nội tiếp chắn nửa đường trịn)

BD BI

 

Do đó: AC // BI AB CI ABCI

b) Chứng minh rằng: EA2 EB2 EC2 ED2 4 R2 Vì BDACAB AD nên ABAD

Ta có: EA2 EB2 EC2 ED2 AB2 CD2

 

2

2 2 2 4

AD CD AC R R

    

c) Tính diện tích đa giác ABICD theo R

2 .

R OE

1. . 1. .

2

ABICD ABD ABIC

SSSDE ACEB BIAC

*

3

R R

OE  AE

3

R R

EC  R

* . .5 5

3

R R R R

DEAE EC   DE  Do đó:

3

R EB

* 2

3

R R

BIACAER 

Vậy: 5.2 5 16

2 3

ABICD

R R R R R R

SR   R 

  (đvdt)

Câu 3: Cho tam giác ABC có ba góc nhọn ba đường cao AA’ , BB CC’ , ’ cắt H Vẽ hình bình hành BHCD Đường thẳng qua D song song với BC cắt đường

thẳng AH M

1) Chứng minh năm điểm A B C D M, , , , thuộc đường tròn

2) Gọi O tâm đường tròn ngoại tiếp tam giác ABC Chứng minh BMCD

BAM OAC

3) Gọi K trung điểm BC,đường thẳng AK cắt OH G Chứng minh G

là trọng tâm tam giác ABC

Hướng dấn giải HD: HS tự vẽ hình

(54)

1)Chứng minh tứ giác ABMD AMDC, nội tiếp A B C D M, , , , nằm đường trịn

2)Xét ( )O có dây MD // BC  sđ MB  sđ CD  dây MBdây CD hay

BMCD

+ Theo phần 1) BC // MDBAM OAC

3)Chứng minh OK đường trung bình tam giác AHD  OK // AH

1

OKAH hay

 

2 *

OK AH

+ Chứngminh tam giác OGK đồng dạng với tam giác HGA

1 2

2

OK GK AG GK

AH AG

     , từ suy G trọng tâm tam giác ABC

Mức độ 2: TH

Câu 4: Cho hai đường tròn ( )O ( )O tiếp xúc A Kẻ tiếp tuyến chung

, ( )

BC BO , C ( )O Đường thẳng BO cắt ( )O điểm thứ hai D a) Chứng minh tứ giác CO OB’ hình thang vuông

b) Chứng minh ba điểm A C D, , thẳng hàng

c) Từ D kẻ tiếp tuyến DE với đường tròn ( )O (E tiếp điểm) Chứng minh

DBDE

Hướng dấn giải

a)Theo tính chất tiếp tuyến ta có OB O C,  vng góc với BC ⇒tứ giác CO OB

hình thang vng

b) Ta có ABC BDC ⇒ ABC BCA 900 ⇒ BAC 900 Mặt khác, ta có góc BAD 900 (nội tiếp nửa đường trịn) Vậy ta có góc DAC 1800 nên điểm D A C, , thẳng hàng.

c)Theo hệ thức lượng tam giác vng DBC ta có DB2 DA DC.

Mặt khác, theo hệ thức lượng đường tròn (chứng minh tam giác đồng dạng) ta có DE2 DA DC. DBDE..

B

C

E

D

A

(55)

I

Q

O O'

F H

P E

D

C B

A

Câu 5: Cho đường tròn ( )O ( )O cắt hai điểm A B, phân biệt Đường thẳng OA

cắt ( )O , ( )O điểm thứ hai C D, Đường thẳng O′A cắt ( )O , ( )O

điểm thứ hai E F,

a) Chứng minh đường thẳng AB CE, DF đồng quy điểm I

b) Chứng minh tứ giác BEIF nội tiếp đường tròn

c) Cho PQ tiếp tuyến chung ( )O ( )O (P ( )O ,Q( )O )

Chứng minh đường thẳng AB qua trung điểm đoạn thẳng PQ Hướng dấn giải

a) Ta có: ABC 900 (góc nội tiếp chắn nửa đường trịn)

ABF90 (góc nội tiếp chắn nửa đường tròn) nên B C F, , thẳng hàng AB CE,

DF đường cao tam giác ACF nên

chúng đồng quy

b) Do IEF IBF 900 suy ra BEIF nội tiếp đường tròn

c)Gọi H giao điểm AB PQ

Ta chứng minh tam giác AHP PHB đồng dạng

2

HP HA

HB HP HP HA HB

   

Tương tự, HQ2 HA HB Vậy HPHQ hay H là trung điểm PQ..

Câu 6: Cho đường trịn tâm O đường kính AB, nửa đường tròn ( )O lấy

điểm G E (theo thứ tự A G E B, , , ) cho tia EG cắt tia BA D Đường thẳng

vng góc với BD D cắt BE C , đường thẳng CA cắt đường tròn ( )O điểm

thứ hai F

a) Chứng minh tứ giác DFBC nội tiếp

b) Chứng minh: BFBG

c) Chứng minh:

DA DG DE BABE BC

Hướng dấn giải

a) Chứng minh tứ giác DFBC nội tiếp

Ta có: AFB 900 (góc nt chắn nửa đường trịn)

Ta có: CDB CFB 900 tứgiác DFBC nội tiếp đường trịn đường kính BC

b) Chứng minh: BFBG

Ta có: AEB 900 (góc nt chắn nửa đường trịn)

2 1

1

D

O

F

E

G

B A

(56)

 900 AEC

 

Ta có: AEC ADC 1800

⇒Tứ giác ADCE nội tiếp đường trịn đường kính AC

 

1

E C

  (vì nt chắn cung DA)

Ta có: B1 C1 (vì nt chắn cung DF đường trịn đường kính BC )

Do đó: E1 B1 AG AF BF BG BFBG

c) Chứng minh:

DA DG DE

BABE BC

Ta chứng minh được:

DGB

 ∽DAE g

g

 

DG DB DG DE DA DB

DA DE

   

BEA

 ∽BDC g

g

 

BE BA

BE BC BA BD

BD BC

   

Từ

 

1 2

 

suy ra:

(

)

= =

DG DE DA DB DA

BE BC BA BD BA đpcm Mức độ 3: VDT

Câu 7: Cho hai đường tròn

O R,

O R', '

với RR' cắt A B Kẻ tiếp tuyến

chung DE hai đường tròn với D

 

O E

 

O' cho B gần tiếp tuyến so với A

a) Chứng minh DAB BDE

b) Tia AB cắt DE M Chứng minh M trung điểm DE

c) Đường thẳng EB cắt DA P , đường thẳng DB cắt AE Q Chứng minh PQ song song với AB

Hướng dấn giải

a) Ta có  đ

2

D BAs DB (góc nội tiếp)

 đ

2

B EDs DB (góc tiếp tuyến dây cung) Suy DAB BDE b) Xét hai tam giác DMB AMD có:

DMA chung, DAM BDM nên ~

DMB AMD

  ⇒ MD = MA

MB MD

hay

MD =MA MB

(57)

Tương tự ta có: EMB ~AMEME = MA

MB ME hay

ME =MA MB

Từ đó: MDME hay M trung điểm DE

c)Ta có DAB BDM, EAB BEM

        1800

PAQ PBQ DAB EAB PBQ BDM BEM DBE

        

 tứ giác APBQ nội tiếp PQB PAB Kết hợp với PAB BDM suy

 

PQBBDM Hai góc vị trí so le nên PQ song song với AB

Câu 8: Cho đường trịn

 

O R; có đường kính AB cố định Vẽ đường kính MN đường

trịn

O R;

(M khác A M khác B, ) Tiếp tuyến đường tròn

O R;

B cắt

đường thẳng AM AN, điểm Q P, 1) Chứng minh tứ giác AMBN hình chữ nhật

2) Chứng minh bốn điểm M N P Q, , , thuộc đường tròn

3) Gọi E trung điểm BQ Đường thẳng vng góc với OE O cắt PQ

điểm F Chứng minh F trung điểm BP ME //

NF

4) Khi đường kính MN quay quanh tâm O thỏa mãn điều

kiện đề bài, xác định vị trí đường kính MN để tứ giác

MNPQ có diện tích nhỏ

Hướng dấn giải

1) Tứ giác AMBN có góc vng, góc nội tiếp chắn

nửa

đường trịn

2) Ta có ANM ABM (cùng chắn cung AM )

ABM AQB (góc có cạnh thẳng góc) ANM AQB nên MNPQ nối tiếp

3) OE đường trung bình tam giác ABQ

OF // AP nên OF đường trung bình tam giác ABP

Suy F trung điểm BP

AP vng góc với AQ nên OE vng góc OF

Xét tam giác vng NPBF trung điểm cạnh huyền BP

Xét tam giác NOF  OFB c c c

 

nên ONF 900.

Tương tự ta có OME 900nên ME // NF vì vng góc với MN. 4)

2SMNPQ 2SAPQ 2SAMN 2 R PQAM AN 2 (R PBBQ)AM AN

A B

P

Q O

F

E N

(58)

Tam giác ABP đồng dạng tam giác QBA suy AB BP AB2 BP QB.

QBBA  

Nên áp dụng bất đẳng thức Cosi ta có PBBQ 2 PB BQ. 2 (2 )R 4R

Ta có . 2 2

2

AM AN

A MN R

AM N    

Do đó, 2 2 4 2 6

MNPQ

SR RRR Suy 3

MNPQ

SR

Dấu xảy AMAN PQBP hay MN vng góc AB

Câu 9: Cho hai đường tròn

O R;

O R; 

tiếp xúc A Vẽ tiếp tuyến chung

, (

BC B C thứ tự tiếp điểm thuộc

O R;

O R; 

) a)Chứng minh BAC 90 0

b) Tính BC theo R R, .

c) Gọi D giao điểm đường thẳng AC đường tròn

 

O ,(DA) vẽ tiếp tuyến DE với đường tròn

 

O (E

 

O ) Chứng minh BDDE

Hướng dấn giải

a) Qua A vẽ tiếp tuyến chung cắt BC M

Ta có MBMAMC (tính chất tiếp tuyến cắt nhau)

 90 0 A

 

b) Giả sử R R Lấy N trung điểm OO

Ta có MN đường trung bình hình thang

vuông OBCO

(OB // O C ; B C 900) tam giác AMN vuông A.

Có '

2 MNRR ;

2

ANR R Khi MA2 MN2 AN2 RR.  MARR' mà BC 2MA2 RR'

c) Ta có O B D, , thẳng hàng (vì BAD 900; OAOBOD)

BDC

 có DBC 90 ,0 BA CD , ta có: BD2 DA DC 1

 

 

~

ADE EDC g g

  

DE DA DC DE

=  DA DCDE2 2

 

   

1 , BDDE

đpcm

Mức độ 4: VDC

Câu 10: Cho nửa đường trịn tâm O đường kính AB 2R tia tiếp tuyến Ax phía với

nửa đường tròn AB Từ điểm M Ax kẻ tiếp tuyến thứ hai MC với nửa

đường tròn (C tiếp điểm) AC cắt OM E MB; cắt nửa đường tròn

 

O (

D D khác B)

E N A M

O O' B

C

(59)

x N

I H E D M

C

O B

A

a) Chứng minh: AMDE tứ giác nội tiếp đường tròn

b) MA2 MD MB.

c) Vẽ CH vng góc với AB H AB (  ).Chứng minh MB qua trung điểm

CH

Hướng dấn giải

a) ADB 900 (góc nội tiếp chắn nửa đường tròn)

 

1

ADM 90

 

Lại có: OAOCR MA; MC (tính chất tiếp tuyến)

Suy OM đường trung trực AC

 

AEM 90

 

Từ

 

1 2

 

suy MADE tứ giác nội tiếp đường

trịn đường kính MA

b) Xét MAB vng AADMB, suy ra: MA2 MB MD. (hệ thức lượng tam giác vuông)

c) Kéo dài BC cắt Ax N , ta có ACB 900 (góc nội tiếp chắn nửa đường tròn)

ACN 90

  , suy ACN vng C Lại có MCMA nên suy

MCMN , MAMN

 

Mặt khác ta có CH // NA (cùng vng góc với AB) nên theo định lí Ta-lét

 

IC IH BI

MN MA BM

 

 

  

  với I giao điểm CH MB

Từ

 

5 6

 

suy ICIH hay MB qua trung điểm củaCH

Câu 11: Cho nửa đường trịn tâm O đường kính AB 2 , R D điểm tùy ý nửa đường tròn (D khác A D khác B ) Các tiếp tuyến với nửa đường tròn ( )O

A D cắt C BC, cắt nửa đường tròn ( )O điểm thứ hai E Kẻ DF

vuông góc với AB F

a) Chứng minh: Tứ giác OACD nội tiếp

b) Chứng minh: CD2 CE CB.

c) Chứng minh: Đường thẳng BC qua

trung điểm DF

d) Giả sử OC 2 ,R tính diện tích phần tam giác ACD nằm ngồi nửa đường trịn ( )O

theo R

Hướng dấn giải

(60)

 900

CAO  (CA tiếp tuyến )

 900

CDO  (CD tiếp tuyến )

  1800

CAO CDO

  

 Tứ giác OACD nội tiếp

b) + Xét CDECBD có: 

DCE chungvà   

2

CDECBD sdDE 

CDECBD (g.g)

2 .

CD CE CD CE CB

CB CD

   

c) Tia BD cắt Ax A Gọi I giao điểm BC DF

Ta có ADB 900 (góc nội tiếp chắn nửa đường tròn)

'

ADA 90

  , suy ADA vng D

Lại có CDCA ( tính chất2 tiếp tuyến cắt nhau) nên suy CDCA, CAA C

 

1 Mặt khác ta có DF // AA (cùng vng góc với AB)

nên theo định lí Ta-lét ID IF BI

 

CA' CA BC

 

 

  

 

Từ

 

1 2

 

suy IDIF

Vậy BC qua trung điểm DF

d) Tính COD COD

0 60

OD C

cos    

 00

AOD12

 2 120 360 quat R R

S (đvdt)

Tính CDR

2

1

2 2

OCD

S  CD DOR RR (đvdt)

2

OACD OCD

SS  R (đvdt)

Diện tích phần tam giác ACD nằm ngồi nửa đường trịn ( )O

2 2 3 3 OACD quat R

SS R  R

 (đvdt)

Câu 12: Từ điểm A nằm bên ngồi đường trịn ( )O kẻ tiếp tuyến AB AC, với đường

tròn (B C, tiếpđiểm)

1) Chứngminh ABOC tứ giác nội tiếp

2) Cho bán kính đường trịn ( )O 3cm, độ dài đoạn thẳng OA 5cm Tính độ

(61)

3) Gọi

 

K đường tròn qua A tiếp xúc với đường thẳng BC C Đường tròn

 

K đường tròn ( )O cắt điểm thứ hai M Chứng minh đường

thẳng BM qua trung điểm đoạn thẳng AC

Hướng dấn giải

M

K

E

H

A

O

B

C

a) - Có ABOB (tính chất tiếp tuyến) ABO 900

- Có ACOC (tính chất tiếp tuyến) ACO 900

- Xét tứ giác ABOCABO ACO 900 900 1800nên nội tiếp đường tròn

b) - AB AC hai tiếp tuyến đường tròn ( )O nên AO đường trung trực BC Gọi H giao điểm AO BC , ta có BC 2BH

- ABO vng BBH đường cao nên OB2 OH AO.

2 9

5

H OB

O

AO

   cm

- OBH vuông HBH2 OB2 –OH2 12

5 BH

  cm

- Vậy 24

5

BCBH  cm

c) - Gọi E giao điểm BM AC

- EMC vàECBMEC CEB MCE  EBC (Góc nt góc tạo tia tiếp tuyến CA chắn cung MC đường tròn ( )O )

.

 

*

EMC ECB g g EC EM EB

     

- EMAEABMEA AEB a

 

và:

+ Có MAE MCB 3

 

(Góc nt góc tạo tia tiếp tuyến CB chắn cung MC

(62)

+ Có MCB ABE (

 

Góc nt góc tạo tia tiếp tuyến BA chắn cung MB

của đường tròn ( )O)

+ Từ

 

3 4

 

MAE ABE b

 

- Từ

 

a b

 

 EMAEAB g

g

EA2 EM EB. * *

 

- Từ

 

* * *

 

EC2 EA ÞEC2 EA.Vậy BM đi qua trung điểm E của AC.

3 BÀI TẬP TỰ LUYỆN

Mức độ 1: NB

Bài Cho tam giác đềuABC nội tiếp đường tròn

( )

O Trên cung BC không chứa A

ta lấy điểm P (P khác B P khác C) Các đoạn PA BC cắt Q Giả sử D điểm đoạn PA cho PD PB= Chứng minh ∆PDB

Bài Cho tam giác ABC nội tiếp đường tròn ( )O Đường phân giác góc

A cắt đường tròn ngoại tiếp tam giác D Gọi I tâm vòng tròn nội tiếp tam giác

ABC Chứng minh DB DC DI= =

Bài Cho hình vuông ABCD, đường chéo BDlấy điểm I cho BI = BA Đường thẳng qua I vng góc với BD cắt AD E, AI cắt BE H Chứng minh

AE ID=

Mức độ 2: TH

Bài 1 Cho đường tròn (O; R) đường kính AC.Trên đoạn thẳng OC lấy điểm B vẽ đường trịn (O’) có đường kính BC Gọi M trung điểm AB, qua M kẻ dây cung vng góc với AB cắt đường trịn (O) D E Nối CD cắt đường tròn (O’) I

Chứng minh MD = MI

Bài Cho đường tròn (O) dây cung BC cố định Gọi A điểm di động cung lớn BC đường tròn (O) (A khác B, A khác C) Tia phân giác ACB cắt đường

tròn (O) điểm D khác điểm.C Lấy điểm I thuộc đoạn CD cho DI = DB Đường thẳng BI cắt đường tròn (O) điểm K khác điểm.B Chứng minh tam giác KAC cân

Bài Cho đường trịn (O) đường kính AB C điểm cung AB (C khác A B) Vẽ CH AB H AB

(

)

Vẽ đường tròn (C; CH) cắt đường tròn (O) Dvà E DE cắt CH M Chứng minh MH = MC

Mức độ 3: VDT

Bài Giả sử A B hai điểm phân biệt đường tròn (O) Các tiếp tuyến đường tròn (O) Các tiếp tuyến đường tròn (O) A B cắt điểm M Từ A kẻ đường thẳng song song với MBcắt đường tròn (O) tại.C MC cắt đường tròn (O) E Các tia AE MB cắt K Chứng minh MK KB=

(63)

Bài Cho tam giác ABC có ba góc nhọn, nội tiếp đường trịn tâm O AB AC< Vẽ đường kính ADcủa đường trịn (O) Kẻ BE CFvng góc với AD (E, Fthuộc AD) Kẻ AHvng góc với BC(Hthuộc BC) Gọi Mlà trung điểm BC Chứng minh ME

= MF

Mức độ 4: VDC

Bài 1 Cho tam giác ABC có AC = 2AB nội tiếp đường trịn (O; R) Các tiếp tuyến đường tròn (O) A, C cắt M BM cắt đường tròn (O) tại.D Chứng minh rằng: CD BD=

Bài 2. Cho tam giác ABCnội tiếp đường tròn (O) Đường tròn K tiếp xúc với CA, AB E, F tiếp xúc với (O) S SE, SF cắt (O) M, N khác S Đường tròn ngoại tiếp tam giác AEM, AFNcắt P khác.A Gọi EN, FMlần lượt cắt (K) G, H khác E, F Gọi GHcắt MNtại T Chứngminh tam giác AST cân

Bài Cho tam giác ABC vuông A đường cao AH Trên nửa mặt phẳng bờ BC chứa

A vẽ đường trịn (O) đường kính HC. Trên nửa mặt phẳng bờ BC không chứa Avẽ

nửa đường trịn (O’) đường kính BC. Qua điểm E thuộc nửa đường trịn (O) kẻ EI vng góc với BC cắt nửa đường tròn (O’) F Gọi K giao điểm EH BF

(64)

P O

Q D

C B

A

O I

D

C B

A

A B C

D

M O H O'

I E

F H

I

D C

B A

HƯỚNG DẪN GIẢI Mức độ 1: NB

Bài

Trước tiên ta nhận thấy tam giác PBD cân P

Mặt khác, BPD BPA BCA  = = =600 (hai góc nội tiếp

chắn AB đường tròn

( )

O ) Vậy nên tam giác PDB

Bài

Ta ln có DB DC= AD phân giác góc A Ta chứng minh tam giác DIB cân D

Thật ta có: IBD IBC CBD  = + Mặt khác CBD CAD =

(Góc nội tiếp chắn cung CD) mà

 

BAD CAD= , IBC IBA =

(Tính chất phân giác)suy

  

IBD ABI BAI= +

Nhưng BID ABI BAI  = + (Tính chất góc ngồi) Như tam giác BDI cân DDB DI DC= =

Nhận xét: Thơng qua tốn ta có thêm tính chất: Tâm đường tròn ngoại tiếp tam giác IBClà giao điểm phân giác góc A với (O)

Bài

Tam giác ABI cân B nên  

BAI BIA= suy EAI EIA = hay EA = EI (1) Xét ∆DIE vuông cân đỉnh I IE = ID (2) Từ (1) (2) suy AE = ID(đpcm)

Mức độ 2: TH Bài

Đường kính ACvng góc với dây DE M ⇒MD ME= Tứ giác ADBE có MD = ME MA = MB(gt), AB DE

⇒ADBE hình thoi (hình bình

hành có hai đường chéo vng góc nhau)

Ta có BIC=900 (góc nội tiếp chắn nủa đường trịn

(65)

O K E M C B A O H B C A M D N E C K O H M B A

 900

ADC= (góc nội tiếp chắn nửa đường trịn (O))

BI CD

⇒ ⊥ AD DC⊥ nên AD // BI,mà BE // AD ⇒E, B, I thẳng hàng (tiên đề Ơclit)

DIE

∆ có IM đường trung tuyến ứng với cạnh huyền ⇒MI = MD

Bài

Ta có

. 1

 

s s ;

2

DBKđ DAđ AK sđ sđ sđ

2

DIB  BDKC

Vì sđBDsđDA DBI cân D nên sđKCsđAK Suy AKCK hay KAC cân K (đpcm) Bài

Dựng đường kính HN đường trịn (C) cắt đường trịn (O) K ta có CN = CH = HK

MC.MK = MH.MN (=MD.ME)

⇒ MC.MK = (HC −MC) (HC + MC) ⇔ MC.MK =HC2−MC2

2

( )

MC MC MK HC

⇔ + =

Hay⇔MC MC MK( + )=HC2

Nên ⇔MC HC HC.2 = ⇔HC=2MC

là điều phải chứng minh

Mức độ 3: VDT Bài

Do MB AC/ / nên BMC ACM = (1), ta lại có   ACM ACE MAE= = (cùng chắn AE)(2) Từ (1) (2) suy ∆KME∆KAM (g.g)

MK EK AK MK

⇒ = hay MK2 =AK EK. (3).

Ta thấy EAB EBK = (cùng chắn BE) Từ ∆EBK∆BAK (g.g)

BK EK AK BK

⇒ = hay BK2 =AK EK. (4).

Từ (3) (4) suy MK2 =KB2 nghĩa

MK MB= (đpcm)

Bài

(66)

A

B C

D

E M

O

OH = OK (định lý liên hệ dây cung khoảng cách đến tâm) H, K trung điểm AB, CD (định lý đường kính vng góc dây cung)

⇒AH = CK

Xét ∆OHM

(

OHM =900

)

có OM(cạnh chung) OH = OK,

do ∆OHM= ∆OKM (cạnh huyền, cạnh góc vng)

MH MK

⇒ =

Ta có MH AH MK CK− = − ⇒MA MC=

Bài

Theo có  

90

= =

AEB AHB

Suy bốn điểm A, B, H, Ecùng thuộc đường tròn

Tứ giác ABHE nội tiếp đường tròn ⇒  

BAE EHC=

Mặt khác,  BCD BAE= (góc nội tiếp chắn 

BD)

Từ (1) (2) suy  BCD EHC=

suy raHE // CD

Gọi K trung điểm EC, I giao điểm

MKvới ED

Khi MKlà đường trung bình ∆BCE

MK // BE; màBEAD(gt) ⇒MKADhay MKEF (3)

Lại có CFAD(gt) ⇒MK // CFhay KI // CFECFKI // CF, KE= KC nên IE= IF(4)

Từ (3) (4) suy MK là đường trung trực EF

ME= MF Mức độ 4: VDC Bài

Xét ∆MADMBAAMB chung;

 

MAD MBA= (góc nội tiếp, góc tạo tia tiếp tuyến vàdây chắn AD)

#

MAD MBA

⇒ ∆ ∆ (g.g)

MA AD MD MB AB MA

⇒ = =

Ta có MA MC= (tính chất hai tiếp

tuyến cắt đường tròn)

I K

M F E

D H

O

B C

(67)

MD MD MA MC

⇒ =

Lập luận tương tự, ta có MD CD

MC BC=

Suy AD CD AD BC AB CD

AB BC= ⇒ =

Dựng điểm E AC∈ cho EDC ADB =

DAB

∆ ∆DECADB EDC = (cách dựng), ABD ECD = (hai góc nội tiếp chắn AD)

#

DAB DEC

⇒ ∆ ∆ (g.g) AB BD AB DC EC BD

EC DC

⇒ = ⇒ = (1)

Do EDC ADB = ⇒BDC ADE = , nên ∆DAE# ∆DBC (g.g)

AD BC BD AE

⇒ = (2)

Từ (1) (2) ta có AB CD AD BC BD AE EC + =

(

+

)

=BD AC

Ta có

AD BC AB CD

AB CD AC BD AD BC AB CD AC BD

 =

⇒ =

 + =

AC=2AB (gt) ⇒2AB CD =2AB BDCD BD=

Bài Phân tích

+ Để chứng minh AMPN hình bình hành ta chứng minh cặp cạnh đối song song dựa vào góc nội tiếp, góc tạo tiếp tuyến dây cung

+ Để chứng minh TA = TS ta nghỉ đến việc chứng minh TA, TS tiếp tuyến đường tròn(O)

Cách giải

T

G

H P N

M

O F

K

E

C B

A

S

Ta thấy APF =180 ANS AMS 180− = = −APE

suy F, P, Ethẳng hàng

Ta có  APM AEM= góc nội tiếp chắn cung AM, AEM SEC = (đối đỉnh)

(68)

2

1

1

K

F I E

O O'

H C

B

A

Mà EF S PAN= tứ giác ANFPnội tiếp Vậy APM PAN = ⇒AN PM/ /

Chứng minh tương tự ta có: AM PN/ / ⇒ANPM hình bình hành + Các tam giác SKF, SON cân có chung đỉnh Snên đồng dạng suy raKF // ON

tương tự KE // OM suy SF SK SE

SN SO SM= = suy MN // EF

Từ HGE HFE HMN  = = suy tứ giác MNGHnội tiếp Giả sử TS cắt (O) (K) S S1, 2

thì TSTS TM TN TH TG TSTS 1= = = 2

suy TS TS1 = 2 suy S S1 ≡ 2 ≡S Vậy TS tiếp tuyến (O)

Tứ giác ANPM hình bình hành nên AP MN cắt trung điểm Imỗi đường Ta có theo tính chấtgóc tạo tiếp tuyến dây cung:

   

IAM PES FST NAS= = = Ta lại có AMI AMN ASN  = = Vậy ∆AIM∆ANS suy AM SN AI AS =

Tương tự AN SM AI SN AM SN = =

Từ theo tính chất tiếp tuyến TS tiếp xúc với (O) suy TM SM22 AM22

TN = SN = AN

Vậy TA tiếp xúc với (O) Suy TA = TS

Bài Phân tích:

Ta có CA2 =CB CH.

nên để chứng minh CA CK= , ta chứng minh CK2 =CB CH. .

Điều làm ta nghĩ đến chứng minh ∆CKH∆CBK,

do cần chứng minh K 1 =B1

Xét góc phụ với hai góc trên, cần chứng minh

  ECK BCF=

Muốn cần chứng minh C C 1= 2

Chỉ cần chứng minh hai góc phụ với chúng E1 K2 (do CEKF tứ giác nội tiếp)

Cách giải

Tứ giác CEKF có: E F + =900+900 =1800 nên tứ giác nội tiếp, suy   1

E K= Do hai góc phụ với chúng C C 1 = 2

(69)

Do hai góc phụ với chúng K 1 =B1

CKH CBK

∆ ∆ (g.g) CK CH CK2 CB CH.

CB CK

⇒ = ⇒ = (1)

Theo hệ thức lượng tam giác ABCvuông Ata có:

2

CA = CB. CH (2)

(70)

Chủ đề 5: CHỨNG MINH HAI ĐƯỜNG THẲNG SONG SONG 1 PHƯƠNG PHÁP CHỨNG MINH.

- Dựng mối quan hệ góc: sole nhau, đồng vị nhau, phía bù nhau,…

- Dựng mối quan hệ bắc cầu: vng góc song song với đường thứ ba

- Áp dụng định lý đảo định lý Talet

- Áp dụng tính chất tứ giác đặt biệt, đường trung bình tam giác

- Dùng tính chất hai dây chắn hai cung đường trịn

2 CÁC VÍ DỤ.

Mức độ 1: Nhận biết

Câu 1. Cho hai đường trịn

( )

O ,

( )

O′ có bán kính R, R′ (R>R′) tiếp xúc với

nhau điểm A Qua A kẻ cát tuyến cắt hai đường tròn

( )

O ,

( )

O′ hai điểm M , N

Chứng minh rằng: OM O N// ′

Lời giải

Ta có:

OMA =OAM ( tam giác OAM cân)

OAM =O AN′ ( đối đỉnh)

O AN ′ =O NA′ ( tam giác O AN′ cân)

Vậy OMA =O NA′ ( sole trong) nên OM O N// ′

Câu 2. Cho tam giác ABC nội tiếp đường tròn tâm O Gọi H trực tâm tam giác D

điểm đối xứng A qua O Chứng minh BH CD// Lời giải O

O’ A

M

(71)

Ta có:

BHAC (do H trực tâm tam giác)

DCAC (góc nội tiếp chắn nửa đường trịn) //

BH CD

Câu 3. Cho tam giác ABC cân A Kẻ hai đường cao BE, CF tam giác Chứng minh

rằng: EF BC//

Lời giải

Xét tam giác BFC CEB, có:

 

 

(

)

90 can chung

F E

B C ABC

BC

 = = °

 = ∆

  

 nên hai tam giác nhau.

Vậy BF CE

BA = CAEF BC// Mức độ 2: Thông hiểu

Câu 1. Cho đường tròn

(

O R,

)

đường kính AB, kẻ tiếp tuyến Ax lấy điểm P

cho AP>R Dựng tiếp tuyến PM (M tiếp điểm) Chứng minh BM OP// Lời giải

A

C B

D H

A

B C

(72)

Ta có:

AMPO(tính chất hai tiếp tuyến xuất phát từ điểm nằm ngồi đường trịn)

AMMB (góc nội tiếp chắn nửa đường trịn) //

OP BM

⇒ (cùng vng góc với đường AM )

Câu 2. Cho đường tròn

( )

O ngoại tiếp tam giác ABC Gọi E, F chân đường cao

hạ từ C, B tam giác ABC Chứng minh EF d// với d tiếp tuyến đường tròn

( )

O A

Lời giải

Ta có: EFCB tứ giác nội tiếp  BEC=BFC= °90 (do BF, CE đường cao)

xAC = ABC (cùng chắn cung AC)

▪  ABC= AFE (góc góc đối ngồi tứ giác EFCB nội tiếp)

 

xAC=AFE (so le trong) nên Ax EF//

A B

M P

x

O

A

B C

E

F x

(73)

Câu 3. Cho đường tròn

( )

O , hai điểm C, D thuộc đường tròn, lấy B trung điểm cung

nhỏ CD Kẻ đường kính BA, tia BA lấy điểm S ngồi đường tròn, SC cắt đường tròn

điểm thứ hai M , DM cắt BA K, BM cắt AC H Chứng minh: HK CD// Lời giải

Ta có: HMK =HAK (cùng chắn hai cung BC, BD) nên tứ giác HKAM nội tiếp

▪  HKM =HAM (góc nội tiếp chắn cung HM)

▪   HAM =CAM =CDM (góc nội tiếp chắn cung CM)

 

HKM CDM

⇒ = (đồng vị) nên HK CD//

Mức độ 3: VDT

Câu 1. Cho tam giác ABC vuông A, lấy điểm D nằm A, B Đường trịn đường kính BD cắt BC E, đường thẳng CD, AE cắt đường tròn điểm F, G Chứng

minh: AC FG//

Lời giải

A B

C

D

E

F G C

B

D

H K

(74)

Ta có:CAD =CED= °90 nên tứ giác ADEC nội tiếp

DFG =DEG (góc nội tiếp chắn cung DG)

DEG  =DEA=DCA (góc nội tiếp chắn cung AD)

 

DFG DCA

⇒ = (sole trong) nên FG AC//

Câu 2. Cho nửa đường tròn đường kính AB điểm M nằm nửa đường

tròn (khác A, B) Trên nửa mặt phẳng bờ ABchứa nửa đường tròn kẻ tiếp tuyến Ax Tia BM

cắt Ax I, tia phân giác góc IAM cắt nửa đường trịn E, cắt BM F, tia BE cắt K

Chứng minh: FK AI//

Lời giải

Xét tam giác FABAMBF, BEAF nên K trực tâm tam giác

Ta có: (gt)

(cmt) AI AB FK AB

⊥   ⊥

 nên FK AI//

Câu 3. Cho tứ giác ABCD nội tiếp đường tròn

( )

O Gọi P trung điểm cung nhỏ AB(phần

không C, D) Hai dây PC, PD cắt AB E, F.Các dây AC, PD cắt I

Các dây BD, PC cắt K Chứng minh CID =CKD IK AB// Lời giải

Ta có:  IDK=ICK (cùng chắn hai cung AP, PB) nên tứ giác CDKI nội tiếp

A B

C

D P

E F

I K

A B

I

K F

E

(75)

▪  1

(

sd sd

)

1

(

sd sd

)

2

KEB= PB+ AC = PA+ AC =PDC

▪  PDC=IKC (góc nội tiếp chắn cung IC)

 

IKE KEB

⇒ = (sole trong) nên AB IK//

Mức độ 4: VDC

Câu 1. Cho ba điểm A, B, C thẳng hàng theo thứ tự Một đường thẳng d qua A

vng góc AC Vẽ đường trịn đường kính BC lấy điểmmột điểm M bất kỳ, tia CM cắt d

tại D, tia AM cắt đường tròn điểm thứ hai N, tia DB cắt đường tròn điểm thứ hai P Chứng minh: AD NP//

Lời giải

Ta có: DAB =DMB= °90 nên tứ giác ABMD nội tiếp

▪  BPN =BMN (góc nội tiếp chắn cung BN)

▪  BMN=BDA (góc nội tiếp chắn cung AB)

 

BPN BDA

⇒ = (so le trong) nên NP AD//

Câu 2. Cho đường tròn

( )

O Trên lấy điểm A cố định kẻ tiếp tuyến Ax A.Lấy M

tùy ý Ax, kẻ tiếp tuyến BM với đường tròn

( )

O Gọi I trung điểm MA K giao

điểm thứ hai BI với đường tròn

( )

O Tia MK cắt

( )

O C Chứng minh: BC MA// Lời giải

A B C

D

M N

(76)

Ta có: 2

IM =IA =IK IB nên IM IB IK = IM

I chung nên hai tam giác ∆IMK, ∆MBK đồng dạng với Suy IMK =MBK

Ta lại có: MBK =BCK (góc nội tiếp chắn cung BK)

 

IMK BCK

⇒ = (sole trong) nên AM BC//

Câu 3. Cho đường trịn

( )

O điểm M nằm ngồi đường tròn Từ M kẻ hai tiếp tuyến MA, MB đến đường tròn, cung nhỏ AB lấy điểm C Từ C hạ vng góc lên MA, MB, AB D, E, F Gọi H giao điểm AC, DF G giao điểm BC, EF Chứng

minh tứ giác CHFG nội tiếp HG AB//

Lời giải

Ta có:

Tứ giác ADCF có  ADC= AFC= °90 nên tứ giác nội tiếp

Tứ giác BECFBEC =BFC= °90 nên tứ giác nội tiếp

   90 

HFC=DAC= ABC= ° −FCG GFC  =EBC=CAB= ° −90 FCH

Do đó, HFG +HCG=180° nên tứ giác HCGF nội tiếp đường tròn

Ta có: CHG   =CFG=CBE=CAB A

B

C M

K I

(77)(78)

Chủ đề 6: CHỨNG MINH HAI ĐƯỜNG THẲNG VNG GĨC

I PHƯƠNG PHÁP CHỨNG MINH

1 Tính chất hai tia phân giác hai góc kề bù

2 Hai đường thẳng cắt tạo thành góc 90 độ Tổng hai góc phụ 90 độ

4 Đường thẳng vng góc với hai đường thẳng song song vng góc với đường thẳng thứ ba

5 Tính chất góc nội tiếp chắn nửa đường trịn

6 Định nghĩa ba đường cao tam giác, định nghĩa đường trung trực đoạn thẳng Định lý Pitago đảo

8 Tính chất đường kính đường trịn qua trung điểm dây cung Tính chất tiếp tuyến đường tròn

10 Tiếp tuyến chung đường nối tâm hai đường tròn, dây cung chung đường nối

tâm hai đường trịn

11 Sử dụng hai góc kề bù

12 Sử dụng chứng minh tam giác tam giác vng 13 Sử dụng tính chất tam giác cân

14 Sử dụng tính chất giao điểm ba đường cao tam giác 15 Sử dụng phép quay góc vng góc quay vng 16 Chứng ming phản chứng

II CÁC VÍ DỤ

MỨC ĐỘ 1: NHẬN BIẾT

Câu 1: Cho tam giác ABCnhọn, dựng đường trịn tâm O đường kính BC, cắt AB E Chứng minh CE⊥AB

Hướng dẫn giải

Vì ∠ BEC góc nội tiếp chắn đường kính BC nên ∠ BEC=900 Do CE⊥AB

Câu 2: Cho nửa đường trịn

( )

O đường kính AB, kẻ tiếp tuyến By Lấy điểm M thuộc By Gọi I trung điểm AM Chứng minh IO⊥AB

Hướng dẫn giải

Dễ thấy IO đường trung bình tam giác AMB ( đoạn nối trung điểm hai cạnh bên) nên IO // MB Lại có MB⊥AB MB tiếp tuyến

( )

O , từ IO⊥AB.

I

O

M

B A

y C A

B E

(79)

Câu 3: Cho đường trịn tâm O đường kính AB, lấy C∈

( )

O cho CA=CB Chứng minh CO⊥AB

Hướng dẫn giải

Dễ thấy O trung điểm AB nên CO đường trung tuyến tam giácABC. Lại có CA=CB nên tam giác ABC cân C, đường trung tuyếnCO đường cao, hay CO⊥AB

MỨC ĐỘ 2: THÔNG HIỂU

Câu 4: Cho tam giác nhọn ABC, đường trịn đường kính AB cắt AC, BC E, F Gọi H giao điểm AF BE Chứng minh CH⊥AB

Hướng dẫn giải

Vì ∠ AEB, AEB∠ góc nội tiếp chắn đường kính AB nên

0

AEB 90 , AEB 90

∠ = ∠ = Suy H giao điểm đường cao AF BE nên trực tâm tam giác ABC, CH đường cao lại CH⊥AB

Câu 5: Cho đường trịn

( )

O đường kính AB MN Tiếp tuyến B cắt AN P, gọi E trung điểm BP Chứng minh OE⊥AM

Hướng dẫn giải

Dễ thấy OE đường trung bình tam giác ABP ( đoạn nối trung điểm hai cạnh bên) nên OE // AP Vì ∠ MAN góc

P N

M

B A

E O

B C

A E

O F

H

O

A B

(80)

nội tiếp chắn đường kính MN nên MAN 90

∠ = hay AP⊥AM Từ OE⊥AM

Câu 6: Cho tam giác ABC nội tiếp

( )

O , đường phân giác ∠ BAC cắt

( )

O D Chứng minh OD⊥BC

Hướng dẫn giải

Vì ∠ BAD = DAC∠ nên DB =DC, suy DB = DC. Lại có

OB = OC nên OD đường trung trực đoạn BC hay OD⊥BC MỨC ĐỘ 3: VẬN DỤNG THẤP

Câu 7: Cho tam giác ABC nhọn

(

AB < BC

)

, nội tiếp đường tròn tâm O Kẻ đường cao AD đường kính AA’, gọi E chân đường vng góc kẻ từ B lên AA’

a) Chứng minh tứ giác ADBE nội tiếp b) Chứng minh DE⊥AC

Hướng dẫn giải

a) Vì đỉnh E D nhìn cạnh AB góc

(

0

)

90

=

nên tứ giác ADBE nội tiếp

b) Do tứ giác ADBE nội tiếp nên ∠ EDC = BAE∠ ( bù với góc ∠ BDE), mà ∠ DCA' = BAE∠ ( chắn cung A’B) nên

EDC = DCA'

∠ ∠ Do DE song song A’C (cặp góc so le nhau) Lại có ∠A'CA góc nội tiếp chắn đường kính nên o

A'CA 90

∠ = hay A'C⊥AC Từ DE⊥AC

Câu 8: Cho tam giác ABCnhọn Đường trịn

( )

O đường kính BC cắt AB, AC Mvà N Gọi H giao điểm BN CM, K trung điểm AH Chứng minh

KN⊥ON

Hướng dẫn giải O

D

C B

A

O

E

D C

B A

(81)

Vì ∠ CMB, BNC∠ góc nội tiếp chắn đường kính BC nên

0

CMB 90 , BNC 90

∠ = ∠ = Suy H giao điểm đường cao CM BN nên trực tâm ∆ABC, AH đường cao

cịn lại hay AH⊥BC Vì NK đường trung tuyến ứng với cạnh huyền AH ∆AHN vuông N nên KN = KH hay ∆KHN cân K,

KNH = KHN

∠ ∠ , lại có ∠ KHN = NCB∠ ( phụ với ∠ HAN) nên KNH = NCB

∠ ∠ (1) Hơn ∠ HNO = HBO∠ ( ∆BON cân O) (2).Từ (1), (2) ta có KNO = KNH HNO

∠ ∠ + ∠ o

= NCB∠ + ∠ HBO = 90 Vậy KN⊥ON

Câu 9: Cho đường trịn

( )

O đường kính AB, bán kính CO⊥AB, M thuộc cung nhỏ AC (M khác A C) Trên đoạn BM lấy điểm E cho BE=AM Chứng minh MC⊥CE

Hướng dẫn giải

Vì CO⊥AB nên C điểm cung AB CA = CB. Từ xét ∆AMC ∆BEC có AM = BE (gt), ∠ MAC = EBC∠ ( chắn

 MC)

CA = CB (cmt) nên ∆AMC = BEC (c-g-c)∆ Suy ∠ MCA = ECB∠

Khi o

MCE = MCA + ACE = ECB + ACE = ACB = 90

∠ ∠ ∠ ∠ ∠ ∠

( góc nội tiếp chắn đường kính AB), MC⊥CE

MỨC ĐỘ 4: VẬN DỤNG CAO

Câu 10: Cho tam giác ABC nhọn

(

AB < AC < BC

)

nội tiếp đường trịn

( )

O , có hai đường cao BD CE Chứng minh OA⊥ED

Hướng dẫn giải

Kẻ thêm đường kính AN gọi I giao điểm OA ED. Vì đỉnh E D nhìn cạnh BC góc

(

0

)

90

= nên tứ giác BEDC nội tiếp, từ ∠ AED = DCB∠ bù với góc ∠ BED

Lại có ∠ EAI = BCN∠ , đó∠ AED + EAI = DCB + BCN = ACN = 90∠ ∠ ∠ ∠ o nên o

AIE = 90

∠ hay OA⊥ED.

K N

H

C A

B M

O

M

O A

C

B E

N O C B

D E

(82)

Câu 11: Cho đường tròn

( )

O đường kính AB E điểm nằm

( )

O , E khác A B Đường phân giác ∠ AEB cắt đoạn AB F Gọi I giao điểm đường trung trực đoạn EF với OE Chứng minhIF⊥AB

Hướng dẫn giải Giả sử EF cắt đường tròn

( )

O điểm thứ hai K Vì ∠ AEK = BEK∠ nên KA =KB, suy KA = KB. Lại có

OA = OB nên OK đường trung trực đoạn AB hay OK⊥AB Vì I thuộc đường trung trực đoạn EF nên IE = IF tức ∆IEFcân I Do ∠ IFE = IEF∠ , lại có ∠ OKE = IEF∠ (∆OEKcân O) nên ∠ IFE = OKE∠ cặp góc đồng vị Suy IF // OK, từ IF⊥AB.

Câu 12: Cho đường tròn

( )

O , BC dây khơng qua tâm Kẻ tiếp tuyến với đường tròn

( )

O B C chúng cắt A Trên cung nhỏ BC lấy điểm M kẻcác đường vng góc MI, MH, MK xuống cạnh tương ứng BC, AC, AB. Gọi giao điểm BM, IK P; giao điểm CM, IH Q Chứng minh PQ⊥MI

Hướng dẫn giải

Vì tứ giác BKMI, CHMI có tổng hai góc đối 180 nên tứ giác nội tiếp Ta có ∠ I = B1 ∠ 

1

sd KM

 

 

  đường tròn ngoại tiếp BKMI, mà ∠ B = C1 ∠ 

1

sd BM

 

 

  nên ∠ I = C1 ∠ Chứng minh tương tự ta có ∠ I = B2 ∠ Từ

o

1 2

PMQ + PIQ = PMQ + I + I PMQ + C + B 180

∠ ∠ ∠ ∠ ∠ = ∠ ∠ ∠ =

nên tứ giác PMQI nội tiếp Suy ∠ Q = I1 ∠ mà ∠ I = C1 ∠ nên

1

Q = C

∠ ∠ , PQ // BC ( cặp góc đồng vị nhau) Theo giả thuyết BC⊥MI nên PQ⊥MI

I

K F O E

B A

C

O I A

P Q

1

M

B

H K

2

(83)

Chủ đề 7: CHỨNG MINH BA ĐIỂM THẲNG HÀNG

1 PHƯƠNG PHÁP CHỨNG MINH

PHƯƠNG PHÁP CHỨNG MINH

Chứng minh đường thẳng đồng quy

- Áp dụng tính chất đường đồng quy tam giác

- Chứng minh đường thẳng qua điểm: Ta hai đường thẳng cắt điểm chứng minh đường thẳng cịn lại qua điểm

- Dùng định lý đảo định lý Talet

2 CÁC VÍ DỤ

Mức độ 1: NB

Câu 1: Cho tam giác ABC có góc B C nhọn,đường cao AH Dựng phía ngồi tam

giác ABC tam giác vuông cân ABD, ACE( BAD=CAE=90o) Gọi M trung điểmcủa DE Chứng minh H, A, M thẳng hàng

Giải

Dựng hình bình hành AEFD M

⇒ trung điểm củaAF (t/c hình bình hành) EF=DA =BA Mặt khác EA=CA(gt);  AEF =CAB (Cùng bù với DAE)

EFA ABC

⇒ ∆ = ∆ (c – g – c)

  1

A =C ( Hai góc tương ứng) Mà   o

1 90 A +C =

  o

1 90 A A

⇒ + =

   o

1 180 A A A

⇒ + + = hay  o A 180

F H = ⇒M , A,H thẳng hàng

Câu 2: Cho∆ABC có trực tâm H nội tiếp

( )

O đường kính CM , gọi I trung điểm AB Chứng minh H, I, M thẳng hàng

Giải

MBBC, AHBC(suy từ giả thiết) //

MB AH

MA BH// (cùng vng góc với AC) AMBH

⇒ hình bình hành

AB

⇒ cắt MH trung điểm I AB MH(t/c hình bình hành)

1 1

2 3 M

F

E

D

H C

B A

M I H

O

C B

(84)

Suy H, I, M thẳng hàng

Câu 3: Chứng minh rằng: trung điểm hai cạnh bên hai đường chéo hình thang ln thẳng hàng

Giải

Giả sử hình thang ABCD

(

AB CD//

)

M , N , P,Q thứ tự trung điểm củaAD, BC, BD,AC

Cần chứng minh M , N, P,Q thẳng hàng Từ (gt) ⇒MN, MP, MQ thứ tự đường

trung bình hình thang ABCD, ∆ABD,∆ACD //

MN AB

⇒ ; MP AB// ; MQ CD// hay MQ AB// M

⇒ , N, P, Q thẳng hàng (theo tiên đề Ơclít) Mức độ 2: TH

Câu 4: Cho

( )

O đường kính AB Điểm M chuyển động

( )

O , MA;MB Kẻ MH vng góc với AB Vẽ đường trịn

( )

O1 đường kính MH cắt đường thẳng MA MB C D Chứng minh rằng:

a) C, D, O1 thẳng hàng b) ABCD nội tiếp

Giải a)Ta có:

 o

90

AMB= (góc nội tiếp chắn nửa

( )

O )

 o

90 CMD

⇒ =

CD

⇒ đường kính

( )

O1 Suy C, D, O1 thẳng hàng

b) MCHD hình chữ nhật nội tiếp

( )

O1  

MCD MH D

⇒ = (2 góc nội tiếp chắnCD)

Mà       o

D 180

MC = ⇒B MCD+ACD= +B ACD= Vậy ABCD nội tiếp

Câu 5: Cho đường trịn

( )

O đường kính AB Trên

( )

O lấy điểm D (khác A, B) Lấy điểm C đoạn AB, kẻ CHAD

(

HAD

)

Phân giác BAD cắt

( )

O E, cắt CH F Đường thẳng DF cắt

( )

O N Chứng minh N, C, E thẳng hàng

Giải

(gt) ⇒HC DB// (cùng vng góc với AD)  

1 C B

⇒ = (2 góc đồng vị)

O

C

D M

B O

H A

1

Q

P N

M

D C

(85)

B 1 =N1 (2 góc nội tiếp chắn AD) ⇒C 1 =N1 ⇒ Tứ giác AFCN nội tiếp

  1 A N

⇒ = (2 góc nội tiếp chắn FC) Hay  A1 =FNC mà  A1=A2(gt)

 

A FNC

⇒ = mà   A2 =DNE=FNE (2góc nội tiếp chắn DE)  

FNC FNE

⇒ = mà NC NE thuộc nửa mặt phẳng bờ DN Suy tia NC NE trùng ⇒N, C, E thẳng hàng

Câu 6: Cho hình chữ nhật ABCDO giao điểm đường chéo Điểm M đoạn OB, lấy E đối xứng với A qua M ;H hình chiếu điểm E BC, vẽ hình chữ nhật

EHCF Chứng minh M , H, F thẳng hàng Giải

Gọi I giao điểm HF CE H

⇒ ,I, F thẳng hàng

( )

* (t/c hình chữ nhật) Cần chứng minh: M ,I,F thẳng hàng

1 E

MA=ME= A (gt)và

2

OA=OC= AC(t/c hình chữ nhật) OM

⇒ đường trung bình ∆ACE  

//

OM CE ODC ICF

⇒ ⇒ = (2 góc đồng vị)

ODC =OCD  ICF=IFC(vì ∆OCDcân O, ∆ICFcân I , t/c hình chữ nhật)

  //

OCD IFC IF AC

⇒ = ⇒ mà IM AC// (doIM đường trung bình ∆ACE) M

⇒ , I , F thẳng hàng (tiên đề Ơclít) Kết hợp với

( )

* ta có: M , H,F thẳng hàng

Mức độ 3: VDT

Câu 7: Cho ∆ABC điểm M tam giác Gọi A1, B1, C1 thứ tự điểm đối xứng M qua trung điểm cạnhBC, CA,AB Gọi O giao điểm

1

BB CC1 Chứng minh điểm A,O,A1 thẳng hàng Giải

Gọi D, E, Fthứ tự trung điểm BC,CA,AB EF

⇒ đường trung bình ∆ABCMB C1 1(suy từ giả thiết) 1

1

2

EF BC B C

⇒ = = EF BC B C// // 1 1

// BC B C

BC=B C1 1

BCB C

⇒ hình bình hành

O

⇒ trung điểm BB1và CC1(t/c hình bình hành) + Tương tự ta có:

1

ABA B hình bình hành

AA

⇒ cắt BB1tại O trung điểm BB1 AA1

(86)

Suy A,O,A1 thẳng hàng

Câu 8: Cho ∆ABC nhọn, nội tiếp đường tròn

( )

O , điểm M cung nhỏ BC E, F thứ tự điểm đốixứng M qua AB, AC, gọi H trực tâm ∆ABC

Chứng minh E, H, Fthẳng hàng Giải

Gọi B′ giao điểm BH AC; A′ giao điểm AH BC Tứ giác HA CB′ ′ nội tiếp

    

1 A

H A CB′ ′ BCA BMA BE

⇒ = = = = (t/c đối xứng trục) ⇒ Tứ giác AHBEnội tiếp

  A

EHB E B MAB ⇒ = =

Tương tự ta có:  A HC′ = ABC, CHF =MAC

           o

180 EH B H A HCCH F MAB ACB ABC MAC ACB ABC BAC

⇒ + + + = + + + = + + =

 o

180

EHF = ⇒E, H, F thẳng hàng

Câu 9: Cho ∆ABC nhọn, đường cao AH, BDCE Gọi M , N , P, Q thứ tự hình chiếu H AB, BD, CE AC Chứng minh M , N, P, Q thẳng hàng

Giải

+ Từ (gt) ⇒MH CE// ; NH AC// BM BH BN

BE BC BD

⇒ = = (định lý Talét)

// MN ED

( )

1 (định ký Talét đảo)

+ Chứng minh tương tự ta có: PQ ED// 2

( )

+ Áp dụng hệ thức lượng tam giác vuông HACHABta có:

AH = AQ AC=AM AB AQ AB AM AC

⇒ = mà AB AD

AC = AE (vì ∆DAB∽∆EAC(g.g))

AQ AD

AM AE

⇒ = hay AQ AM MQ ED//

AD = AE ⇒ (định lý Talét đảo) Kết hợp với

( )

1 ,

( )

2 ta có:

M , N, Q thẳng hàng M , P, Q thẳng hàng (tiên đề Ơclít) Do M , N , P, Q thẳng hàng

Mức độ 4: VDC

Câu 10: Cho đường tròn

( )

O dây cung AB Lấy I thuộc đoạn AB cho IA>IB Gọi D điểm cung nhỏ AB, DI cắt

( )

O điểm thứ hai C Tiếp tuyến với

( )

O C cắt AB K Lấy điểm E cho

KE=KI = IE, EC cắt

( )

O F Chứng minh D, O, F thẳng hàng

Giải

Ta có 1

(

 

)

2

I = sđ CB + AD Mà  AD=DB (gt)

O M H B' C' A' F

E B C

(87)

(

 

)

1

2 s

I BC đDB sđ DBC

⇒ = + =

  

1

1

I ICK sđ DBC

⇒ = = ⇒ ∆KIC cân K =>KI =KC

2

KI =KE= IE (gt)

KC IK KE IE CIE

⇒ = = = ⇒ ∆ vuông C

 o

90

DCF DF

⇒ = ⇒ đường kính

( )

O Suy D, O, F thẳng hàng

Câu 11: Cho

( )

O đường kính AB Trên

( )

O lấy điểm D (khác A, B) Lấy điểmbất kỳ đoạn AB, kẻ CHAD

(

HAD

)

Phân giác BAD cắt

( )

O E, cắt CH

F Đường thẳng DF cắt

( )

O N Chứng minh N, C, E thẳng hàng Giải 1 1 1 2 N F H D E C O A B 2 1

(gt) ⇒HC DB// (cùng vng góc với AD)  

1 C B

⇒ = (2 góc đồng vị)

B 1 =N1(2góc nội tiếp chắn AD) ⇒N 1 =C1 Suy tứ giác AFCN nội tiếp

  A N

⇒ = (2 góc nội tiếp chắn FC) Hay  A1 =FNC mà  A1=A2(gt)

 

A FNC

⇒ = mà   A2 =DNE=FNE (2 góc nội tiếp chắn DE)  

FNC FNE

⇒ = mà NC NE thuộc nửa mặt phẳng bờ DN Suy tia NCNE trùng nhaunên N, C, E thẳng hàng

Câu 12: Cho ∆ABC, đường tròn bàng tiếp góc A tiếp xúc với tia AB N Kẻ đường kính MN Trên tia đối tia AB lấy điểm K cho AK =BN Chứng minh K,

C, M thẳng hàng Giải

Gọi I, J theothứ tự tâm đường tròn bàng tiếp góc A, góc BABC

( )

I tiếp xúc với BC AC thứ tự P H

( )

J tiếp xúc với BC BAthứ tự Q K′

(88)

Ta có:

CA CB+ −AB=CA CP+ +PBAB

– – –

CA CH NB AB AH NB AB AN NB AB NB

= + + = + = + = (t/c tiếp tuyến)

– –

CA CB AB NB

⇒ +

Tương tự ta có: CA CB+ –AB = 2AK′ AK AKBN KK

⇒ = = ⇒ ≡

Mặt khác ∆PIC đồng dạng ∆QJC(g.g) Nên IC IP IM JC = JQ = JKCIM =CJK (2 góc so le củaMN JK// )

ICM

⇒ ∆ đồng dạng ∆JCK(c.g.c)  

ICM JCK

⇒ = Suy tia CK CM đối Vậy K, C, M thẳng hàng

3 BÀI TẬP TỰ LUYỆN

Mức độ 1: NB

Câu 13: Cho tam giác ABC vuông A Trên cạnh AC lấy điểm M, dựng đường trịn (O) có đường kính MC đường thẳng BM cắt đường trịn (O) D đường thẳng AD cắt đường tròn(O) S

1 Chứng minh ABCD tứ giác nội tiếp

2 Chứng minh CA tia phân giác góc SCB

3 Gọi E giao điểm BC với đường tròn(O) Chứng minh đường thẳng BA, EM, CD đồng quy

4 Chứng minh DM tia phân giác góc ADE

5 Chứng minh điểm M làtâm đường trònnội tiếp tam giácADE

Lời giải: 3 2 1 1 F O M S D E B A C H×nh a F C A B E D S M O 1 1 2 H×nhb

1 Ta có ∠CAB = 900( tam giácABC vng A); ∠MDC = 900( góc nội tiếp chắn

nửa đường trịn ) => ∠CDB = 900như D A nhìn BC góc 900

nên A D nằm đường trịnđường kính BC => ABCD tứ giác nội tiếp ABCD tứ giác nội tiếp => ∠D1= ∠C3( nội tiếp chắn cung AB)

∠D1= ∠C3=> SM =EM=> ∠C2 = ∠C3(hai góc nội tiếp đường trịn(O) chắn hai cung

bằng nhau)

=> CA tia phân giác góc SCB

TH2(Hình b)

(89)

∠ABC = ∠CME(cùng phụ ∠ACB); ∠ABC = ∠CDS (cùng bù∠ADC) => ∠CME = ∠CDS

=> CE CS = =>SM =EM => ∠SCM = ∠ECM => CA tia phân giác góc SCB

3 Xét ∆CMB Ta có BA⊥CM; CD ⊥BM; ME ⊥BC BA, EM, CD ba đường cao tam giácCMB nên BA, EM, CD đồng quy

4 Theo Ta có SM =EM =>∠D1= ∠D2=> DM tia phân giác góc ADE.(1)

5 Ta có ∠MEC = 900(nội tiếp chắn nửa đường tròn (O)) => ∠MEB = 900

Tứ giác AMEB có ∠MAB = 900;∠MEB = 900=> ∠MAB + ∠MEB = 1800mà hai

góc đối nên tứ giác AMEB nội tiếp đường tròn=> ∠A2 = ∠B2

Tứ giác ABCD tứ giác nội tiếp => ∠A1= ∠B2( nội tiếp chắn cung CD)

=> ∠A1= ∠A2=> AM tia phân giác góc DAE (2)

Từ (1) (2) Ta có M tâm đường trònnội tiếp tam giácADE

Câu 14: Cho tam giác ABC vuông ở.A điểm D nằm A và.B Đường trịn đường kính BD cắt BC E Các đường thẳngCD, AE cắt đường tròntại F, G.Chứng minh:

1 Tam giácABC đồng dạng với tam giácEBD Tứ giác ADEC AFBC nội tiếp

3 AC // FG

4 Các đường thẳngAC, DE, FB đồng quy

Lời giải:

1 Xét hai tam giácABC EDB Ta có ∠BAC = 900( tam giác

ABC vng A); ∠DEB = 900( góc nội tiếp chắn nửa đường

trịn )

=> ∠DEB = ∠BAC = 900;lại có ∠ABC góc chung => ∆DEB ∼∆

CAB

2 Theo ∠DEB = 900=> ∠DEC = 900(vì hai góc kề bù);

∠BAC = 900( ∆ABC vng A) hay ∠DAC = 900=> ∠DEC

+ ∠DAC = 1800mà hai góc đối nên ADEC tứ giác nội tiếp

* ∠BAC = 900( tam giácABC vng A); ∠DFB = 900( góc nội tiếp chắn nửa

đường tròn ) hay ∠BFC = 900như F A nhìn BC góc 900nên

A F nằm đường trịnđường kính BC => AFBC tứ giác nội tiếp Theo ADEC tứ giác nội tiếp => ∠E1 = ∠C1lại có ∠E1 = ∠F1=> ∠F1 = ∠C1mà

đây hai góc so le nên suy AC // FG

4 (HD) Dễ thấy CA, DE, BF ba đường cao tam giácDBC nên CA, DE, BF đồng quy S

Câu 15: Cho đường trịn(O) đường kính AB Trên đoạn thẳng OB lấy điểm H ( H khơng trùng O, B);trên đường thẳngvng góc với OB H, lấy điểm M ngồi đường trịn;MA MB thứ tự cắt đường tròn (O) C D Gọi I giao điểm AD

BC

1 Chứng minh MCID tứ giác nội tiếp

G

1 O

S D

E B

A C

(90)

2 Chứng minh đường thẳngAD, BC, MH đồng quy I

3 Gọi K tâm đường trònngoại tiếp tứ giác MCID, Chứng minh KCOH tứ giác nội Lời giải:

1 ∠BIC = 900( nội tiếp chắn nửa đường trịn ) => ∠BID

= 900(vì hai góc kề bù);DE ⊥AB M => ∠BMD =

900

=> ∠BID + ∠BMD = 1800mà hai góc đối tứ

giác MBID nên MBID tứ giác nội tiếp

2 Theo giả thiết M trung điểm AB; DE ⊥AB M nên M trung điểm DE (quan hệ đường kính dây cung)

2

1 1

/ /

1

O'

E

3 2 1

I

O D

C M

A

(91)

Chủ đề 8: CHỨNG MINH BA ĐƯỜNG THẲNG ĐỒNG QUY 1 PHƯƠNG PHÁP CHỨNG MINH

Cách Lợi dụng định lí đường đồng quy tam giác

 Sử dụng định lí ba đường cao tam giác đồng quy điểm

 Sử dụng định lí ba đường trung tuyến tam giác đồng quy điểm Điểm gọi trọng tâm tam giác

 Sử dụng định lí: 1.Ba đường phân giác tam giác đồng quy điểm

 Giao điểm hai đường phân giác nằm đường phân giác góc thứ ba

 Sử dụng định lí ba đường trung trực tam giác đồng quy điểm

Cách Sử dụng tính chất đường chéo cắt tai trung điểm đường của hình bình hành, hình chữ nhật, hình thoi, hình vng

Cách Lùi quen thuộc, chứng minh ba điểm thẳng hàng giao điểm hai đường nằm đường thẳng thứ ba

2 CÁC VÍ DỤ Mức độ 1: NB

Câu 1. Cho tam giác ABC có ba đường trung tuyến AD BE CF, , Chứng minh AD, BE, CF đồng quy

Giải:

Áp dụng tính chất: Trong tam giác, ba đường trung tuyến qua điểm

Trong tam giác ABCAD BE CF, , ba đường trung tuyến nên AD, BE, CF cùn qua điểm

Vậy AD, BE, CF đồng quy

Câu 2. Cho tam giác ABC có ba đường cao AD BE CF, , Chứng minh AD, BE, CF đồng quy

Giải:

Áp dụng tính chất: Trong tam giác, ba đường cao qua điểm

Trong tam giác ABC có ba đường cao AD BE CF, , nên AD, BE, CF qua điểm Vậy AD, BE, CF đồng quy

Câu 3. Cho tam giác ABC có ba đường phân giác AD BE CF, , Chứng minh AD, BE, CF đồng quy

Giải:

(92)

Trong tam giác ABC có ba đường phân giác AD BE CF, , nên AD BE CF, , qua điểm

Vậy AD, BE, CF đồng quy Mức độ 2: TH

Câu 1. Chotam giác ABC cân A, kẻ đường cao AH

(

HBC

)

Gọi M N, trung điểm AC AB, Chứng minh AH BM CN, , đồng quy

Giải:

Vì ∆ABC cân A có đường cao AH nên AH đường trung tuyến ∆ABC ,

M N trung điểm AC AB, nên BM CN, đường trung tuyến ∆ABC Vậy ba đường trung tuyến AH BM CN, , đồng quy

Câu 2. Cho tam giác ABC cân A, kẻ đường cao BH CK,

(

HAC K, ∈AB

)

Gọi M trung điểm BC Chứng minh AM BH CK, , đồng quy

Giải:

Vì ∆ABC cân A có đường trung tuyến AM nên AM đường cao ∆ABC Vì tam giác có ba đường cao qua điểm, ba đường cao AM BH CK, , đồng quy

Câu 3. Cho tam giác ABC cân A, kẻ đường cao AH

(

HBC

)

Gọi BD CE, đường phân giác góc B góc C

(

DAC E, ∈AB

)

Chứng minh AH BD CE, , đồng quy

Giải:

Vì ∆ABC cân A có đường cao AH nên AH đường phân giác ∆ABC Vì tam giác có ba đường phân giác qua điểm, ba đường phân giác AH,

,

BD CEđồng quy Mức độ 3: VDT

Câu 1. Cho hình bình hành ABCD Gọi E F, theo thứ tự trung điểm AB CD, Gọi M giao điểm AF DE N giao điểm BF CE Chứng minh rằng:

a) EMFN hình bình hành

b) Các đường thẳng AC EF MN, , đồng quy Giải:

a) Tứ giác AECFAE CF AE€ , =CF nên tứ giác AECF hình bình hành Suy AF CE

Chứng minh tương tự, BF DE

(93)

b) Gọi O giao điểm AC EF Ta chứng minh MN qua O AECF hình bình hành, O trung điểm AC nên O trung điểm EF EMFN hình bình hành nên đường chéo MN qua trung điểm O EF Vậy AC EF MN, , đồng quy O

Câu 2. Trên hình vẽ bên, cho ABCD hình bình hành Chứng minh rằng:

a) EFGH hình bình hành

b) Các đường thẳng AC BD EF GH, , , đồng quy Giải:

a) Chứng minh EG=HF EH; =GF

b) Gọi O giao điểm AC EF Tứ giác AECFAE=CF AE CF, € nên hình bình

hành

Suy O trung điểm AC EF,

ABCD hình bình hành, O trung điểm AC nên O trung điểm BD EGHF hình bình hành, O trung điểm EF nên O trung điểm GH Vậy AC BD EF GH, , , đồng quy O

Câu 3. Cho hình bình hành ABCD Lấy điểm E cạnh AB, lấy điểm F cạnh CD cho AE=CF Chứng minh ba đường thẳng AC BD EF, , đồng quy

Giải:

Gọi O giao điểm AC BD Hãychứng minh AECF hình bình hành để suy ba điểm E O F, , thẳng hàng

Câu 4. Cho hình bình hành ABCDE F, theo thứ tự trung điểm AB CD, a) Tứ giác DEBF hình gì?

b) Chứng minh đường thẳng AC BD EF, , cắt điểm Giải:

a) Tứ giác DEBF hình bình hành Học sinh tự chứng minh

b) Gọi O giao điểm hai đường chéo hình bình hành ABCD, ta có O trung điểm BD

Theo câu a), DEBF hình bình hành nên trung điểm O BD trung điểm EF Vậy AC BD CF, , cắt điểm O

(94)

Bài 1. Cho∆ABC với đường caoAH Vẽ phía ngồi∆ABCcác tam giác,ACE vuông cân CABDvuông cân B.Trên tia đối tiaAH lấy điểmK choAK =BC Chứng minh

1) BECK

2)Ba đường thẳngAH BE CD, , đồng quy điểm Giải:

1 1

K

H D

E A

B

C

Ta có: C 1 =A1 ⇒ BCE=KAC Xét ∆BCEKAC có:

( )

gt BC=KA

CE= AC (∆ACE vuông cân C)  

BCE=KAC(cmt)

(

)

BCE KAC c g c

⇒ ∆ = ∆

  BEC KCA

⇒ =

Mặt khác: KCA +ECK = °90 ⇒BEC +ECK = °90 Vậy BEKC

b) Chứng minh tương tự: DCKB

tam giác KBC có ba đường cao KHBC BE, ⊥KC CD, ⊥KB Vậy ba đường thẳngAH BE CD, , đồng quy điểm

(95)

D

N M

J I

B

O C

A

M thuộc đường trịn đường kính IJ nên JMI = °90 hay JMCI Tương tự INCJ

Tam giác CIJ có đường cao CA JM IN, , đồng quy D Vậy IN JM AB, , đồng quy điểm D

Bài 3. Cho hình thang ABCD

(

AB<CD

)

Gọi E giao điểm hai cạnh bên AD BC, F trung điểm AB Chứng minh AC BD EF, , đồng quy

Cho tam giác ABC, dựng tam giác MAB NBC PAC, , thuộc miền tam giác ABC Chứng minh MC NA PB, , đồng quy

Giải:

1 2 2

1

3 2 1

4 3

1 2 2 1

K

N

P

M A

B C

Dễ thấy ∆AMC= ∆ABP c g c

(

)

C 1=P1

(96)

Trong ∆PCK, có: C   1+C2+K2+P2 =180°⇒ ° +60

(

C 1+P2

)

+K2 =180° ⇒K2 = °60 Tương tự: ∆ABN = ∆MBCN 1=C3 mà  N1+N2 = °60

  60 N C

⇒ + = ° mà C4 = °60 NKC

⇒ ∆ có N   2+C3+C4+K3=180° ⇒K3 = °60 Chứng minh tương tự: K1= °60

Theo chứng minh ta có: K  1=K2 =K3 = °60 ⇒K  1+K2+K3 =180° , ,

A K N

⇒ thẳng hàng Vậy AN MC BP, , đồng quy 2 BÀI TẬP TỰ LUYỆN Mức độ 1: NB

Câu 1. Cho tam giác MNP có ba đường trung tuyến MD NE PF, , Chứng minh MD, NE, PF đồng quy

Câu 2. Cho tam giác MNP có ba đường cao MD NE PF, , Chứng minh MD, NE, PF đồng quy

Câu 3. Cho tam giác MNP có ba đường phân giác MD NE PF, , Chứng minh MD, NE, PF đồng quy

Mức độ 2: TH

Bài 2. Cho ∆ABCcân tạiB Tia phân giác gócB cắt đường trung tuyến AC K Gọi ,

I J trung điểm củaAB BC, Chứng minh BK AJ CI, , đồng quy

Bài 3. Cho hai đường thẳng a b, cắt O, đường thẳng a, lấy ba điểm A B C, , choOA=AB=BC, đường ta lấy ba điểm L M N, , cho LO=OM =MN Chứng minh rằngAL BN CM, , đồng quy điểm

Bài 4. Cho∆ABC với đường caoAH Vẽ điểm D E, choAB AC, thứ tự đường trung trực đoạnHD HE, Gọi M N, giao điểm củaDE vớAB AC, Chứng minh rằng, ba đường thẳngAH BN CM, , đồng quy điểm

Bài 5. Cho∆ABC với điểmM nằm tam giác GọiH K L, , theo thứ tự hình chiếu vng góc củaM trênBC CA AB, , Các đường thẳngh l k, , quaA B C, , vng góc với

, ,

KL LH HK Chứng minh rằng, ba đường thẳng h l k, , đồng quy điểm Mức độ 3: VDT

Bài 2. Cho ∆ABC AB( <AC) phân giácAD Vẽ tia Cx/ /AD cắt đường trung trực cạnhACtạiE, By/ /AD cắt đường trung trực cạnhABtạiF Chứng minh rằng:

(97)

Bài 3. Cho∆ABC nhọn không cân tạiAvới đường caoAH VẽHMAB HN, ⊥ AC KẻHE/ /AC HF, / /AB Chứng minh rằngEF MN BC, , đồng quy điểm

Mức độ 4: VDC

Bài 1. Ở bên ngoài∆ABC, vẽ tam giác đềuABC BCA CAB1, 1, Chứng minh rằngAA BB CC1, 1, 1đồng quy mộtđiểm.(Tứ giác nội tiếp)

Bài 2. Cho∆ABC với trung tuyếnAM.Trên cạnhAC AB, thứ tự lấy điểmD E, choAB=3AE AC, =3AD Chứng minh rằng,AM BD CE, , đồng quy điểm

Bài 3. Cho tứ giácABCD, người ta kẻ hai đường thẳng song song với đường chéoAC cắt cạnhBA BC, tạiG H, Cắt cạnhDA DC, thứ tự tạiE F, Chứng minh

, ,

GE HF BDđồng quy điểm

Bài 4. Cho∆ABC với trực tâmHA B C1, 1, theo thứ tự giao điểm đường trung trực tam giácHBC HCA HAB, , Chứng minh AA BB CC1, 1, 1đồng quy điểm

(98)

Chủ đề 9: CHỨNG MINH ĐƯỜNG THẲNG LÀ TIẾP TUYẾN CỦA ĐƯỜNG TRÒN

1 PHƯƠNG PHÁP CHỨNG MINH

Phương pháp 1:

Chứng minh khoảng cách từ tâm O đến đường thẳng (d) bán kính R ( Phương pháp thường dung chưa biết giao điểm (d) (O) ) Phươngpháp 2:

Nếu biết đường thẳng (d) (O) có giao điểm A Ta cần chứng minh minh

OAd

3 Phương pháp 3:Phương pháp trùng khít

Để chứng minh đường thẳng (d) tiếp tuyến (O) ta dựng đường thẳng (d’) tiếp tuyến (O) sau chứng minh (d) (d’) trùng Do (d) tiếp tuyến (O)

2 CÁC VÍ DỤ

Mức độ 1: NB.

Câu 1: Chọn sai câu sau: đường thẳng a tiếp tuyến đường tròn (O; 6cm)

nếu

a) Khoảng cách từ O đến a OH = 6cm

b) Đường thẳng a cắt đường tròn (O; 6cm) hai điểm phân biệt

c) Đường thẳng a qua điểm C thuộc đường trịn (O; 6m)

d) Đường thẳng a vng góc với OH H; H thuộc đường tròn (O; 6cm)

Câu 2: Cho tam giác ABC có AB=3, AC=4, BC=5 Vẽ đường tròn(B;BA) Chứng minh AC

là tiếp tuyến đường tròn

Hướng dẫn giải

Ta có: 2 2 2 2

5 25; 25;

BC = = AB +AC = + = ⇒BC = AB +AC ABC

⇒ ∆ vuông A

AB AC

⇒ ⊥

Suy ra: AC tiếp tuyến (B;BA)

Câu 3: Cho hình vẽ, có 

60 ;

BAC = AO=10cm Chọn đáp án của:

4

3 5

A C

(99)

a) Độdài bán kính OB là:

A B C D đáp án khác

b) Độ dài tiếp tuyến AB là:

A C B D đáp án khác

Mức độ 2: TH.

Câu 4: Cho đường tròn (O) đường kính AB Ax, By tia tiếp tuyến (O) (Ax, By

nửa mặt phẳng bờ đường thẳng AB) Trên Ax lấy điểm C, By lấy điểm D cho 

90

COD= Chứng minh rằng: CD tiếp xúc với đường tròn (O)

Hướng dẫn giải

Từ C vẽ tiếp tuyến CD’ đường tròn (O) (D’ thuộc By) tiếp xúc với (O) tiếp điểm H

Ta có: OC phân giác AOH(t/c hai tiếp tuyến cắt nhau)

Và OD’ phân giác góc BOH

Mà hai góc AOH BOH hai góc kề bù nên ∠OCD’ = 900

⇒ ta có ∠COD’ = ∠COD= 900 mà D, D’ thuộc By nên suy D′ ≡D Vì CD’ tiếp tuyến (O) ⇒ CD tiếp tuyến (O)

O

A

B

(100)

Câu 5: Cho đường tròn (O; 3cm) điểm A đường tròn Qua A kẻ tiếp tuyến Ax, lấy lấy điểm B cho AB=OA

a) Tính độ dài đoạn OB;

b) Gọi I trung điểm đoạn OB, AI cắt đường tròn (O) C Tứ giác CBAO hình gì? Vì sao?

c) Chứng minh BC tiếp tuyến đường tròn (O) Hướng dẫn giải a) OB=2 3cm

b) Ta có: OBAC AOC

∆ cân O

OABC

⇒ hình vng

c) Theo câu b tứ giác OABC hình vng ⇒OCBC

⇒ BC tiếp tuyến (O)

Câu 6: Cho đường trịn (O), dây AB khác đường kính Qua O kẻ đường vng góc với AB, cắt

tiếp tuyến A đường tròn điểm C

a) Chứng minh CB tiếp tuyến đường tròn

b) Cho bán kính đường trịn 15cm, AB=24cm Tính độ dài OC

Hướng dẫn giải a) Ta có: OCAB ⇒ OC qua trung điểm AB

⇒OC đường cao đồng thời trung tuyến ∆ABC

⇒∆ABC cân C

⇒  ACO BCO

AC CB

 =

 

=

 ⇒ ∆AOC = ∆BOC c

(

− −g c

)

OB BC

⇒ ⊥

⇒Bc tiếp tuyến (O) b) OC=22,5cm

O

A

B

(101)

Mức độ 3: VDT

Câu 7: Cho tam giác ABC nhọn Vẽ đườngtròn tâm O đường kính BC cắt AB, AC

E vàF BF CE cắt I.Gọi M trung điểm AI Chứng minh: MF tiếp tuyến (O)

Hướng dẫn giải Ta chứng minh I trực tâm tam giác ABC

Trong tam giác vng AFI có FM trung tuyến nên MF = MA = MI, suy tam giác MFA cân M, suy  AFM =MAF

Ta có: OFC =FCO (Tam giác OCF cân O)

Từ đó: Suy ra.

90 MFO=

Vậy nên MF tiếp tuyến (O)

I

F

E

O

A

B

C

(102)

Câu 8: Cho tam giác ABC vng A, đường cao AH Đường trịn đường kính BH cắt AB D, đường trịn đường kính CH cắt AC E Chứng minh DE tiếp tuyến chung (I) (J)

Hướng dẫn giải

Để chứng minh DE tiếp tuyến đường trịn tâm I đường kính BH ta chứng minh

IDDE hay ∠DOE = 90o

Vì D, E thuộc đường trịn đường kính BH HC nên ta có: ∠BDH =∠CEH = 900

 tứ giác ADHE hình chữ nhật

Gọi O giao điểm AH DE, ta có OD = OH = OE = OA  ∆ODH cân O ⇔∠ODH = ∠OHD

Ta có ∆IDH cân I ⇔∠IDH = ∠IHD

 ∠IDO = 900⇔ID ⊥DE

Ta có IDDE D, ∈

( )

I  DE tiếp xúc với (I) D

Chứng minh tương tự ta có DE tiếp xúc với (J) E

Câu 9: Cho tam giác ABC nhọn, đường cao BDvà CE cắt H Gọi I trung điểm

BC Chứng minh ID, IE tiếp tuyến đường tròn ngoại tiếp tam giác ADE Hướng dẫn giải

Gọi O trung điểm AH

Tam giác ADH vng D có DO trung tuyến nên ta có:

2 AH

DO= =OA=OH

Tam giác AEH vng E có EO trung tuyến nên ta có:

2 AH

EO= =OA=OH ⇒OA = OD = OE, O tâm đường tròn ngoại tiếp tam giác ADE

Tam giác OAD cân O) ⇒∠ODA = ∠OAD (1)

∆BDC vng D có DI trung tuyến ⇒

2 BC

DI = =IC, ⇒tam giác ICD cân I, ⇒∠IDC = ∠DCI (2)

H giao điểm hai đường cao BD CE

⇒H trực tâm ∆ABC,

O

E D

I H J

B

(103)

AHBC F (3)Khi ∠OAD +ICD=90o (2)

Từ (1), (2) (3) ta có

∠ODA + ∠IDC = ∠OAD +∠ICD = 900

Ta có ODDI D, ∈

( )

O ⇒ID tiếp xúc với (O) D

Chứng minh tương tự ta có IE tiếp xúc với (O) E (DPCM)

Mức độ 4: VDC

Câu 10: Cho nửa đường tròn (O) đường kính AB= 2R, dây cung AC Gọi M điểm

giữa cung AC Đường thẳng kẻ từ C song song với BM cắt tia AM K cắt tia OM D OD cắt AC H

1 Chứng minh tứ giác CKMH nội tiếp

2 Chứng minh CD = MB DM = CB

3 Xác định vị trí điểm C nửa đường tròn (O) để AD tiếp tuyến nửa đường tròn

4 Trong trường hợp AD tiếp tuyến cửa nửa đường trịn (O), tính diện tích phần tam giác ADC ngồi đường trịn (O) theo R

Hướng dẫn giải  

180

H +K = ⇒ tứ giác CKMH nội tiếp

2 tứ giác DMBC hình bình hành

3 ∆ADCcó AK ⊥ CD DH ⊥ AC nên M trực tâm tam giác Suy ra: CM ⊥ AD

Vậy ADAB ⇔ CM // AB ⇔ AM =BC

Mà  AM =MC nên AM =BC⇔  AM =MC=BC

⇒ sđBC = 600

4 Tính diện tích phần tam giác ADC ngồi (O) theo R: Gọi S diện tích phần tam giác ADC ngồi đường trịn (O)

S1là diện tích tứ giác AOCD

S2là diện tích hình quạt góc tâm AOC Ta có: S = S1 – S2 ∗ Tính S1:

AD tiếp tuyến đường tròn (O) ⇔    60

AM =MC=BC = ⇒AOD=600

F I

O H

D E

A

B

(104)

Do đó: AD = AO tg 600= R 3 ⇒ SADO=

2

1

2 2

R AD AO= R R=

AOD COD

∆ = ∆ (c.g.c) ⇒ SAOD= SCOD ⇒ SAOCD= SADO=

2

R =

3 R

∗ Tính S2:  120

AC = ⇒ Squạt AOC=

2 0 120 360 R

π =

3 R

π ∗ Tính S:

S = S1 – S2= R2 –

3 R

π = 2

3 3

R −πR = 2

(

)

3 3

R −π (đvdt)

Câu 11: Cho tam giácnhọn ABC có ∠B = 450 Vẽ đường trịnđường kính AC có tâm O, đường

trịnnày cắt BA BC D E

1 Chứng minh AE = EB

2 Gọi H giao điểmcủa CD AE, Chứng minh đường trung trực đoạn HE qua trung điểm I BH

3 Chứng minh OD tiếp tuyến đường trònngoại tiếp tam giácBDE Hướng dẫn giải

1 ∠AEC = 900(nội tiếp chắn nửa đường tròn )

=> ∠AEB = 900( hai góc kề bù); Theo giả thiết ∠ABE = 450 => ∆AEB tam giác vuông cân E => EA = EB

2 Gọi K trung điểm HE (1); I trung điểm HB => IK đường trung bình tam giác HBE => IK // BE mà ∠AEC = 900nên BE ⊥HE E => IK ⊥HE K (2)

Từ (1) (2) => IK trung trực HE Vậy trung trực đoạn HE qua trung điểm I BH

3 theo I thuộc trung trực HE => IE = IH mà I trung điểm BH => IE = IB

∠ADC = 900(nội tiếp chắn nửa đường tròn ) => ∠BDH = 900(kềbù ∠ADC) => tam giác BDH vng D có DI trung tuyến (do I trung điểm BH) => ID = 1/2 BH hay ID = IB => IE = IB = ID => I tâm đường tròn ngoại tiếp tam giác BDE bán kính ID Ta có ∆ODC cân O (vì OD OC bán kính ) => ∠D1= ∠C1 (3)

(105)

Theo ta có CD AE hai đường cao tam giác ABC => H trực tâm tam giác ABC => BH đường cao tam giác ABC => BH ⊥AC F => ∆AEB có

∠AFB = 900

Theo ∆ADC có ∠ADC = 900=> ∠B1= ∠C1( phụ ∠BAC) (5)

Từ (3), (4), (5) =>∠D1= ∠D2mà ∠D2+∠IDH =∠BDC = 900=> ∠D1+∠IDH = 900= ∠IDO => OD ⊥ID D => OD tiếp tuyến đường tròn ngoại tiếp tam giác BDE

Câu 12: Cho đường trịn (O) đường kính AB.C điểm thay đổi đường tròn (O) Tiếp

tuyến (O) C cắt AB D Qua O vẽ đường thẳng vng góc với phân giác góc ODC, đường cắt CD M Chứng minh đường thẳng d qua M song song với AB tiếp xúc với (O) C thay đổi

Hướng dẫn giải

Ta thấy đường thẳng d (O) chưa có giao điểm nào, ta dùng cách để giải tốn

Vẽ OHd H

(

d

)

Ta cần chứng minh OH = OC

Ta có tam giác DMO cân D ⇒DMO = DOM

HMO =DOM (so le trong)  

DMO HMO

⇒ =

Từ ta có: ∆CMO= ∆HMO

⇒ OH = OC Vậy d tiếp tuyếncủa (O) C thay đổi

F

1

1

1

/

/ _

_ K

H I

E D

O

C B

A

d

A

D

O

B

M

C

(106)

Chủ đề 10: CÁC BÀI TỐN VỀ TÍNH TỐN ĐỘ DÀI CẠNH, ĐỘ LỚN GĨC, DIỆN TÍCH HÌNH

1 PHƯƠNG PHÁP.

1 Tổng ba góc tam giác, góc ngồi tam giác, tổng góc tứ giác

2 Tính chất tam giác cân, tam giác vng cân, tam giác

Tam giác cân: ∆ABC cân A    180 A B C

2 − ⇒ = =

Tam giác vuông cân: ∆ABC vuông cân A

  BC AB AC

2 B C 45  = = 

⇒ 

 = = 

Tam giác đều: ∆ABC

2 ABC

AB AH

2 AB S

4 ∆

 =  ⇒ 

 =



với AH đường cao tam giác

3 Tính chất tứ giác đặc biệt: hình thang, hình thang cân, hình bình hành, hình chữ nhật,

hình thoi, hình vng

4 Tính chất trọng tâm tam giác, đường trung bình tam giác, đường trung bình

hình thang

5 Định lí Ta-lét, hệ định lí Ta-lét

6 Tính chất phân giác, phân giác ngồi tam giác

7 Tam giác đồng dạng, tam giác

Chú ý:

Tỉ số chu vicủa hai tam giác đồng dạng tỉ số đồng dạng

Tỉ số hai đường caotương ứng hai tam giác đồng dạng tỉ số đồng dạng

Tỉ số hai đường trung tuyến tương ứng hai tam giác đồng dạng tỉ số đồng

dạng

Tỉ số hai đường phân giáctương ứng hai tam giác đồng dạng bằngtỉ số đồng dạng

Tỉ số hai diện tích tương ứngcủa hai tam giác đồng dạng bình phương tỉ số đồng

dạng

(107)

ABC

∆ vuông A, đường cao AH

2

2 2

2

2 2

AB BH.BC; AC CH.BC AB AC BC

AH BH.CH AB.AC BC.AH

1 1

AH AB AC

= = + = = = =      ⇒      +

9 Hệ thức cạnh góc tam giác vng

ABC

∆ vuông A AB BC.sin C BC.cos B

AB AC tan C AC.cot B

= =

⇒  = =

10 Diện tích đa giác

Hình chữ nhật: S=ab (a, b cạnh hình chữ nhật)

Tam giác vuông: S ab

2

= (a,b cạnhgóc vng)

Tam giác thường: S ah

2

= (với a: cạnh; h:chiều cao tương ứng với cạnh a)

Hình thang: S

(

a b h

)

2 +

= (với a, b: đáy; h: chiều cao)

Hình bình hành: S=ah (a: cạnh; h: chiều cao ứng với a)

Tứ giác có hai đường chéo vng góc: S d d1

2

= (với d , d1 hai đường chéo)

11 Cơng thức tính độ dài đường trịn, độ dài cung trịn

Cơng thức tính độ dài đường trịn: C=2πR (R: Bán kính đường trịn)

Độ dài cung tròn:

180 Rn

l =π (n: số đo độ cung trịn)

12 Cơng thức tính diện tích hình trịn, hình quạt trịn

Diện tích hình trịn:

SR

Diện tích hình quạt trịn:

2

360

q

R n S

2 CÁC VÍ DỤ.

Mức độ 2: THÔNG HIỂU

Câu 1:Cho tam giác ABC cân A, đường cao AD, BE, cắt H Gọi O tâm

đường tròn ngoại tiếp tam giácAHE

1 Chứng minh tứ giác CEHD nội tiếp

2 Bốn điểm A, E, D, B nằm đường tròn

3 Chứng minhED =

2

(108)

4 Tính độ dài đoạn thẳng DE biết DH =2cm AH; =6cm Hướng dẫn giải

1 Chứng minh tứ giác CEHD nội tiếp

Xét tứ giác CEHD, ta có: CEH =900; CDH =90  

90 CEH CDH

⇒ + =

Do đó, tứ giác CEHD nội tiếp

2 Bốn điểm A, E, D, B nằm đường trịn

Ta có:  

90

BEA=BDA= ⇒ E D nhìn đoạn AB góc vng Vậy bốn điểm A,

E, D, B nằm đường tròn

3 Chứng minh

2

ED = BCvàDE tiếp tuyến đường tròn

( )

O

+ Tam giác ABC cân A nên D trung điểm BC ⇒ ED đường trung tuyến tam giác

vuông BEC ⇒

2 ED = BC

+ Ta có

2

ED = BCDE=DB⇒ ∆DBE cân D ⇒ E 3 =B1

( )

1

+ Dễ thấy: ∆AOE cân O⇒ A1=E1

( )

2 Mà  A1 =B1 (do phụ với BCA)

( )

3

Từ

( )

1

( )

2

( )

3 , ta có: E 1=E3 ⇒E   2+E3 =E2+E1=90

Do đó: 

90

DEO= Vậy DE tiếp tuyến đường trịn

( )

O

4 Tính độ dài đoạn thẳng DE biết DH =2cm AH; =6cm

Theo giả thiết AH =6cm =>OH =OE=3 ; cm DH =2cm=>OD=5 cm Áp dụng định lí Pitago

cho tam giác OED vng E ta có 2

ED =OD OEED2 =5 – 2 ⇒ ED=4cm

Câu 2:Cho đường tròn (O)và điểm A cho OA=3R Qua

A kẻ tiếp tuyến AP AQ đường tròn (O),với P Q

tiếp điểm.Lấy M thuộc đường tròn (O) cho PM song song với AQ.Gọi N giao điểm thứ đường thẳng AM đường tròn (O).Tia PN cắt đường thẳng AQ K

1 Chứng minh APOQ tứ giác nội tiếp

2 Chứng minh

KA =KN KP

3 Kẻ đường kính QS đường tròn (O).Chứng minhtiaNS

tia phân giác gócPNM

H

1

3 2 1 1

O

E

D C

B

(109)

4 Gọi G giao điểm đườngthẳng AO PK.Tính độ dài đoạn thẳng AG theo bán kính R

Hướng dẫn giải

1 Xét tứ giác APOQ có

 90

APO (Do AP tiếp tuyến (O) P)

 90

AQO (Do AQ tiếp tuyến (O) Q)

  180 APO AQO

   ,mà hai góc góc đối nên tứ

giác APOQ tứ giác nội tiếp

2 Xét

Δ

AKN ΔPAK có AKP góc chung

 

APNAMP ( Góc nt……cùng chắn cung NP)

NAK  AMP(so le PM //AQ

( )

~

AKN PKA gg AK NK AK2 NK KP

PK AK

    (đpcm)

3 Kẻ đường kính QS đường trịn (O)

Ta có AQQS (AQ tt (O) Q)

Mà PM//AQ(gt) nên PMQS

Đường kính QS PM nên QS qua điểm

chính cung PM nhỏ  

sd PSsd SM  PNSSNM (hai góc nt chắn cung nhau)

Hay NS tia phân giác góc PNM

5 Chứng minh

Δ

AQO vng Q, có

QGAO(theo Tính chất tiếp tuyến cắt nhau)

Theo hệ thức lượng tam giác vng ta có 2 3 3 OQ R

OQ OI OA OI R

OA R

AI OA OI R R R

    

     

Do ΔKNQ ~ΔKQP (gg)

KQ KN KP

  mà

AKNK KPnên AK=KQ

Vậy ΔAPQ có trung tuyến AI PK cắt G nên G trọng tâm

2 16

3 3

AG AI R R

   

Câu 3:Cho hình thang vng ABCD

(

A D 90 = = 0

)

AB=BC =8, DC =12 Tính số đo góc BC hình thang ABCD

Hướng dẫn giải Kẻ BKCD K

(

CD

)

(110)

Ta có: A D K 90  = = = nên tứ giác ABKDlà hình chữ nhật DK =AB=8 Mà CD 12= suy

1

CK BC

2

= cosC C 60 

2

⇒ = ⇒ = Từ ta tính ABC 120  =

Mức độ 3: VẬN DỤNG

Câu 4:Cho hình thang cân ABCD

(

AB CD

)

C=600, DB tia phân giác ADC Biết

=

AB , tính chu vi vàdiện tích hình thang ABCD

Hướng dẫn giải

Ta có: DB tia phân giác ADCADB =BDC Mà ⇒ ABD=BDC (so le trong)

Do  ABD= ADB⇒ ∆ABDcân A ⇒AD=4

Kẻ BEAD E

(

CD

)

Suy ABED hình thoi ⇒AB=BE=ED=DA=4

Tam giác BECđều ⇒CE=EB=BC=4 Từ tính chu vi hình thang ABCD 20

Diện tích hình thang 12 3

(

dv td

)

Câu 5:Cho tam giác ABC

(

AB< AC

)

có ba góc nhọn Đường trịn tâm O đường kính BC cắt cạnh AC, AB E, F Gọi H giao điểm BE CF D giao điểm AH BC.

a Chứng minh: ADBC AH AD =AE AC

b Chứng minh EFDOlà tứ giác nội tiếp

c Trên tia đối tia DE lấy điểm L choDL = DF Tính số đo góc BLC. Hướng dẫn giải

a Do FCAB BE, ⊥AC⇒H trực tâm ⇒ AHBC

Ta có tứ giác HDCE nội tiếp

Xét tam giác đồng dạng EAH DAC (2 tam giác

vng có Achung)

AH AE

AC AD

⇒ = ⇒AH AD =AE AC (đpcm) b Do AD phân giác FDE

nênFDE=2FBE=2 FCE=FOE

Vậy tứ giác EFDO nội tiếp (cùng chắn cung EF)

c Vì AD phân giác FDE

C B

A

F

E

L R

S

D O

Q

(111)

⇒ DB phân giác FDL

⇒ F, L đối xứng qua BC ⇒ ∈L đường trịn tâm O

Vậy BLC góc nội tiếp chắn nửa đường tròn tâm O ⇒BLC=900

Câu 6:Cho đường trịn tâm O, đường kính AB=2R Gọi d1 d2 hai tiếp tuyến đường tròn

( )

O hai điểm A vàB Gọi I trung điểm OA E điểm thuộc

đường trịn

( )

O (E khơng trùng với A B) Đường thẳng d qua điểm E vng góc với

EI cắt hai đường thẳng d1, d2 lần lượttại M, N

a Chứng minh AMEI tứ giác nội tiếp

b Chứng minh ENI =EBI góc MIN  = 900

c Chứng minh AM.BN=AI.BI

d Gọi F điểm cung AB khơng chứa E đường trịn (O) Hãy tính diện

tích tam giác MIN theo R ba điểm E, I,F thẳng hàng

Hướng dẫn giải

a Xét từ giác MAIE có góc vng A, E (đối nhau) nên chúng nội tiếp đường

trịn đường kính MI

b Tương tự ta có tứ giác ENBI nội tiếp đường trịn đường kính

IN Vậy góc ENI =EBI (vì chắn EI) Tương tự  EMI=EAI (vì chắn EI)

Mà góc  

EAI+EBI=90 (∆EAD vuông E) ⇒

(

 

)

0 MIN=180 − EMI ENI+ =180 −90 =90

c Xét tam giác vuông MAI IBN Ta có NIB =IMA (góc có

cạnh thẳng góc)

⇒ ∆MAI∽∆IBN ⇒ AM AI

IB = BN ⇔ AM.BN=AI.BI (1)

d Gọi G điểm đối xứng Fqua AB. Ta có AM+BN=2OG 2

( )

(Vì tứ giác AMNBlà hình

thang cạnh OG cạnh trung bình AM BN)

Ta có: AI R, BI 3R

2

= =

Từ (1) (2) ⇒AM + BN = 2R AM.BN = 3R2

4

Vậy AM, BN nghiệm phương trình

X – 2RX R

+ = ⇒ AM R

2

= hay BN 3R

2 =

Vậy ta có tam giác vng cân MAI cân A NBI cân B

⇒MI = R R

2 = NI =

3R 3R = ⇒S(MIN)=

2 R 3R 3R

2 2 =

M E

I

A O B

F G

(112)

Mức độ 4: VẬN DỤNG CAO

Câu 7:Cho ∆ABC nhọn có A =700

(

CA<CB

)

nội tiếp đường tròn

( )

O Các đường cao AE, CFcắt H Vẽ đường thẳng d vng góc với OF F, d cắt CA Q Tính số đo FHQ

Hướng dẫn giải

Gọi giao điểm d

( )

O M, N (Mthuộc cung AC không chứa B), CF kéo dài cắt

( )

O I, BI cắt MN K

Theo toán bướm ta chứng minh Flà trung điểm KQ

Chứng minh Hvà I đối xứng qua ABnên HKIQ hình bình hành

Suy FHQ =FIK mà FIK =CAB=700 Vậy 

FHQ=70

Câu 8:Cho đường trịn

(

O; R

)

có đường kính BC=10cm Lấy A∈

( )

O cho AB=6cm Tia phân giác BAC cắt BCtại D cắt

( )

O E Tính độ dài AD, AE

Hướng dẫn giải

Ta có: 

BAC=90 (góc nội tiếp chắn đường trịn)

∆ABC vng A suy AC=8cm

Tính DB, DC:

Có DB AB

DC =AC (tính chất phân giác)

DB DB DC DB DC 10

DC 4 7

+

⇒ = ⇒ = = =

30 40 DB ; DC

7

⇒ = =

Từ DF DC

AB= BC tính

30 DF

7

=

Rồi suy AD DF 24

7

(113)

Ta có: ∆ADB∽∆ACEvà AD AB

AC = AE, ta tính A=7

Câu 9:Cho tam giác ∆ABC, M điểm nằm tam giác ABC Gọi '

A =AM∩BC, '

B =BM∩AC, C' =CM∩AB Tính AM' BM' CM'

AA +BB +CC Hướng dẫn giải

 Kẻ AH MK vng góc với BC

(

H, K∈BC

)

Ta có: ' ' ' MBC ' MBC ABC ABC A M MK

S A M A A AH

S

MK A A S

AH S  =  ⇒ =   = 

Tương tự:

' '

AMC AMB

' '

ABC ABC

S S

B M C M

;

B B = S C C = S

' ' '

' ' '

A M B M C M AA BB CC

⇒ + + =

Mặt khác:

' ' ' ' ' '

' ' ' ' ' '

AM BM CM AA A M BB B M CC C M

AA BB CC AA BB CC

− − −

+ + = + +

' ' '

' ' '

A M B M C M

1 1

AA BB CC = − + − + − =

3 BÀI TẬP TỰ LUYỆN LOẠI 1: TÍNH SỐ ĐO GĨC

Bài Cho hình thang ABCD có A = =D 90, AB AD 1CD

= = Gọi M điểm cạnh AB,

qua M kẻ tia Mxvng góc với MD, Mxcắt BC N Tính số đo góc MDN MND

Bài Cho ∆ABC đều, cạnh ABlấy điểm M, tia đối tia CA lấy điểm N cho BM=CN Trung trực đoạn MN cắt tia phân giác BAC I Tính số đo AIC Bài Cho

(

O; R

)

và tam giác ABCnội tiếp đường tròn, gọi BE, CF đường cao

ABC

∆ Cho biết EF R

2

= , tính số đo

 BAC

LOẠI 2: TÍNH ĐỘ DÀI VÀ DIỆN TÍCH Bài Cho tam giác ABC vuông A

(

AB<AC

)

Tia phân giác ABC cắt AC

tại M Đường trịn đường kính MC cắt tia

BM điểm thứ hai H, AB cắt CH tạiD

(114)

Bài Cho tứ giác ABCD nộiđường trịn (O;R) đường kính AC.Gọi AC cắt BD E,, gọi K,M chân đường vng góc kẻ từ A C xuống BD ( biết K thuộc đoạn

BE,KB;KE).Đường thẳng qua K song song với BC cắt AC P

1.Cứng minh tứ giác AKPD nội tiếp

2.Chứng minh KP⊥PM

3 Biết 

ABD=60 AK=x.Tính BD theo R x

Bài Cho tam giác ABC nhọn có 

BAC=60 , BC 10cm= Đường trịn tâm O, đường kính BC cắt cạnh AB, AC theo thứ tự D, E Tính diện tích phần mặt phẳng giới hạn dây DE

 DE

LOẠI 3: TÍNH CÁC TỈ SỐ

Bài Cho hình vng ABCD, E điểm thuộc cạnh CD.Tia AE cắt BC F, vẽ tia Axvng

góc với AE cắt CD K Gọi I trung điểm KF Tính tỉ số ID

CF Bài Cho tam giác ABC nhọn có 

A=60 AB<AC Vẽ đường cao BF, CF tam giác

ABC.Gọi I trung điểm BC,K trung điểm EF Tính tỉ số AK

AI

Bài Cho tam giác ABC nhọn nội tiếp

( )

O , đường cao AD, BE, CF tam giác ABC lần

lượt cắt

( )

O ' ' '

A , B , C Tính

' ' '

(115)

Chủ đề 11: CHỨNG MINH ĐẲNG THỨC HÌNH HỌC

I PHƯƠNG PHÁP CHỨNG MINH

1. Sử dụng tam giác đồng dạng, tam giác 2. Hệ thức lượng tam giác vng

3. Tính chất : đường trịn, hình vng, hình chữ nhật, hình thoi

4. Dựng hình phụ

5.

II CÁC VÍ DỤ Mức độ 1: NB

Câu 1. Cho nửa đường trịn đường kính AB=2R Từ A B kẻ hai tiếp tuyến Ax, By

Qua điểm M thuộc nửa đường tròn kẻ tiếp tuyến thứ ba cắt tiếp tuyến Ax, By

C D

a) Chứng minh AC+BD=CD

b) Chứng minh COD = ° 90

c) Chứng minh

4 AB AC BD=

Hướng dẫn giải

a) Theo tính chất hai tiếp tuyến cắt ta có: CA=CM , DB=DMAC+BD=CM +DM

CM +DM =CDAC+BD=CD

b) Theo tính chất hai tiếp tuyến cắt ta có :OC tia phân giác góc AOM ,

OD tia phân giác góc BOM, mà AOMBOM hai góc kề bù ⇒COD = °90

c) Theo COD= °90 nên tam giác COD vuông OOMCD (OM tiếp tuyến)

áp dụng hệ thức cạnh đường cao tam giác vng ta có

OM =CM DM

OM =R, CA=CM, DB=DM

2

4 AB

AC BD R AC BD

⇒ = ⇒ =

Câu 2. Cho hình vng ABCD, điểm E thuộc cạnh BC Qua B kẻ đường thẳng vng

góc với DE, đường thẳng cắt đường thẳng DE DC theo thứ tự H K

a) Chứng minh BHCD tứ giác nội tiếp

b) Tính góc CHK

c) Chứng minh KC KD =KH KB

A B

C

D y x

M

(116)

Hướng dẫn giải

a) Theo giả thiết ABCD hình vng nên BCD= °90 , BHDE H nên BHD = ° 90

Như H C nhìn BD góc 90° nên H C nằm đường

trịn đường kính BD suy BHCD tứ giác nội tiếp

b) BHCD tứ giác nội tiếp BDC +BHC=180°

( )

1

BHK góc bẹt nên  KHC+BHC=180°

( )

2

Từ

( ) ( )

1 , ⇒CHK =BDCBDC =45° (vì ABCD hình vng) CHK=45°

c) Xét ∆KHCKDB ta có CHK =BDC=45°, K góc chung

KC KH

KHC KDB KC KD KH KB

KB KD

⇒ ∆ ~ ∆ ⇒ = ⇒ =

Câu 3. Cho nửa đường trịn tâm O đường kính AB=2R Trên tia đối tia BA lấy điểm

G (khác với điểm B) Từ điểm G A B, , kẻ tiếp tuyến với đường tròn

( )

O Tiếp tuyến

kẻ từ G cắt hai tiếp tuyến kẻ từ A B C D Gọi N tiếp điểm tiếp

tuyến qua G với đường tròn

( )

O Chứng minh

Hướng dẫn giải

Ta có : BDAG AC, ⊥AGBD//AC⇒ ∆GBD~ ∆GAC g g

( )

AC DB

CG DG

⇒ =

Mặt khác CA=CN DB, =DN (Tính chất hai tiếp tuyến xuất phát từ điểm)

CN DN CG = DG

CN DN CG = DG

O N

A B

C

D

G O

K

A B

C D

(117)

Mức độ 2: TH

Câu 1. Cho hình bình hành ABCD có đỉnh D nằm đường trịn đường kính AB=2R

Hạ BN DM vng góc với đường chéo AC

a) Chứng minh tứ gíac CBMD nội tiếp

b) Chứng minh : DB DC =DN AC

Hướng dẫn giải

a) Góc ADB= °90 (Góc nội tiếp chắn nửa đường tròn)

AD/ /BCDBBC

Xét tứ giác DMBCDMC =DBC= °90 nên nội tiếp

b) Ta có ∆DBN ~ ∆CAD (g.g)

DAC =DBN ( chắn cung DN)

 

BDN =BAN (cùng chắn cung BN)=DCA

C

D D

DN

B DC N C

A A

DB

C =D

⇒ = ⇒

Câu 2. Cho tam giác ABC nội tiếp trịn tâm O có độ dài cạnh BC=a, AC=b, AB=c

E điểm nằm cung BC không chứa điểm A cho cung EB cung EC, AE cắt

cạnh BC D Chứng minh : AD2 =AB ACDB DC

Hướng dẫn giải

Ta có BAD =CAE( Do EB =EC)

Và  AEC=DBA( Hai góc nội tiếp chắn cung AC) nên ∆BAD~ ∆EAC

D

E b

C

A

B c

a

O D

N

A B

C

(118)

( )

BA AE

AB AC AE AD AD AC

⇒ = ⇒ =

Ta có  ADC=BDC CAD =DBE

(2 góc nội tiếp chắn cungCE) nên ∆ACD~ ∆BDE

AD DB

AD DE DB DC DC DE

⇒ = ⇒ =

(

)

AD AEAD =DB DC

Hay

AD =AD AEDB DC=AB ACDB DC (do

( )

1 )

Câu 3. Cho tam giác ABCAD đường phân giác góc A Chứng minh rằng:

AD =AB AC BD CD

Hướng dẫn giải

Trên AD lấy điểm k cho:  ABK = ADC Dễ thấy AD=AKDK

 

ABK = ADC; BAK =CAK

( )

ABK ADC g g

⇒ ∆ ~ ∆ AK AC AK AD AB AC

AB AD

⇒ = ⇒ = (1)

 

BDK =ADC,  BKD=ACD(do ∆ABK ~ ∆ADC)

( )

BDK ADC g g

⇒ ∆ ~ ∆ BD DK DK AD BD DC

AD DC

⇒ = ⇒ = (2)

Trừ vế theo vế (1) (2), ta được: AK ADDK AD =AB ACBD DC

Hay:

AD = AB ACBD DC (đpcm)

Mức độ 3: VDT

Câu 1. Cho đường tròn

( )

O ,BC dây BC <2R Kẻ tiếp tuyến với đường tròn

(O) B C chúng cắt A Trên cung nhỏ BC lấy điểm M kẻ đường

vng góc MI MH MK, , xuống cạnh tương ứng BC AC AB, ,

a) Chứng minh tam giác ABC cân

b) Các tứ giác BIMK, CIMH nội tiếp

c) Chứng minh

MI =MH MK

Hướng dẫn giải

D C

A

B

(119)

a) Theo tính chất hai tiếp tuyến cắt ta có AB=AC⇒ ∆ABC cân A

b) Theo giả thiết MIBCMIB= °90 ; MKABMKB= °90

  180

MIB MKB

⇒ + = ° mà hai góc đối ⇒ tứ giác BIMK nội tiếp

* ( Chứng minh tứ giác CIMH nội tiếp tương tựtứ giác BIMK )

c) Theo tứ giác BIMK nội tiếp ⇒ KMI+KBI =180°, tứ giác CIMH nội tiếp

  180

HMI HCI

⇒ + = °.Mà ⇒KBI =HCI (vì tam giác ABC cân A) ⇒ KMI =HMI

( )

1

Theo tứ giác BIMK nội tiếp  B1=I1 (nội tiếp chắn cung KM ); tứ giác CIMH nội

tiếp H 1=C1( nội tiếp chắn cung IM ) Mà  B1 =C1 (

= sđ ) ⇒ =I 1 H1

( )

2

Từ

( ) ( )

1 , MKI MIH MI MK MI MH MK MH MI

⇒ ∆ ~ ∆ ⇒ = ⇒ =

Câu 2. Cho đường trịn

( )

O , đường kính AB cố định, điểm I nằm A O cho

3

AI = AO Kẻ dây MN vuông góc với AB I Gọi C điểm tùy ý thuộc cung lớn MN

sao cho C không trùng với M N, B Nối AC cắt MN E

a) Chứng minh tứ giác IECB nội tiếp đường tròn

b) Chứng minh ∆AME~ ∆ACM

AM =AE AC

c) Chứng minh

AE ACAI IB= AI

Hướng dẫn giải

BM

O K

A

B C

M

H

I

1

1

(120)

a) Ta có EIB= °90 (giả thiết)

 90

ECB= ° (góc nội tiếp chắn nửa đường tròn)

Kết luận: Tứ giác IECB tứ giác nội tiếp

b) Ta có : sđAM = sđAN,  AME=ACM

Góc A chung, suy ∆AME~ ∆ACM

Do :

AC AM

AM AE AC

AM = AE ⇔ =

c) MI đường cao tam giác vuông MAB nên MI2 = AI IB

Trừ vế hệ thức câu b) với hệ thức

Ta có: 2

AE ACAI IB= AMMI =AI

Câu 3. Cho đường tròn

( )

O có tâm O, đường kính BC Lấy điểm A đường tròn

( )

O cho AB> AC Từ A, vẽ AH vng góc với BC (H thuộc BC) Từ H, vẽ HE

vng góc với AB HF vng góc với AC (E thuộc AB, F thuộc AC)

a) Chứng minh AEHF hình chữ nhật OA vng góc với EF

b) Đường thẳng EF cắt đường tròn

( )

O P Q (E nằm P F)

Chứng minh

AP =AE AB Suy APH tam giác cân

c) Gọi D giao điểm PQ BC; K giao điểm cùa AD đường tròn

( )

O (K

khác A) Chứng minh AEFK tứ giác nội tiếp

d) Gọi I giao điểm KF BC Chứng minh IH2 =IC ID

Hướng dẫn giải

I M

O

N

A B

(121)

a) Tứ giác AEHF hình chữ nhật có góc vng Góc HAF =EFA (vì AEHF hình chữ nhật)

Góc OAC =OCA (vì OA=OC)

Do đó: góc OAC +AFE= ° ⇒90 OAEF

b) OAPQPA = AQ

Do đó: ∆APE~ ∆ABP

2 AP AE

AB AP AP AE AB

⇒ = ⇒ =

Ta có :

AH =AE AB (hệ thức lượng ∆HAB vng H, có HE chiều cao) AP=AH APH

⇒ ⇒ ∆ cân A

c) ∆DCF ~ ∆DEB g g

( )

DE DF =DC DB

DC DB=DK DA(Phương tích điểm D)⇒DE DF =DK DA

Do ∆DFK ~ ∆DAEDKF =DEA⇒ tứ giác AEFK nội tiếp

d) Góc ICF  =AEF =DKF⇒ ∆ICF ~ IKD g g

( )

suy IC ID =IF IK

  

( )

.

IKH =FAH =FHI⇒ ∆IFH ~ IHK g g suy IH2 =IF IK

Vậy

IH =IC ID

Mức độ 4: VDC

Câu 1. Cho tam giác MNP cân M có cạnh đáy nhỏ cạnh bên, nội tiếp đường tròn

(

O R;

)

Tiếp tuyến N P đường tròn cắt tia MP tia MN E D

a) Chứng minh :

NE =EP EM

b) Chứng minh tứ giác DEPN tứ giác nội tiếp

c) Qua P kẻ đường thẳng vng góc với MN cắt đường tròn

( )

O K (K không trùng

với P)

F

D E

H P

C I A

B

Q

O

(122)

Chứng minh rằng: 2 MN +NK = R

Hướng dẫn giải

a) ∆NEM ~ ∆PEN ( g.g)

2

NE ME

NE ME PE

EP NE

⇒ = => =

b) ( tam giác MNP cân M )

 

PNE=NPD (=NMP )⇒ DNE=DPE

Hai điểm N P, thuộc nửa mặt phẳng bờ DE nhìn DE góc

nên tứ giác DNPE nội tiếp

c) ∆MPF đồng dạng ∆MIP( g.g )

MNI NIF

∆ ~ ∆ ( g.g )

( )

2 IF

.IF NI

NI MI

MI NI

⇒ = ⇒ =

Từ (1) (2) : 2

(

)

2

( )

3 MP +NI =MI MF+IF =MI = R

( phụ )KPN =NPINK =NI

( )

4

Do tam giác MNP cân MMN =MP

( )

5

Từ (3), (4), (5) suy điều phải chứng minh

Câu 2. Cho tam giác vuông cân ADB DA

(

=DB

)

nội tiếp đường trịn tâm O Dựng

hình bình hành ABCD Gọi H chân đường vng góc kẻ từ D đến AC, K giao điểm

của AC với đường tròn

( )

O Chứng minh :

a) HBCD tứ giác nội tiếp

b) DOK=2.BDH

c)

CK.CA=2.BD   MNP=MPN

2

(1)

MP MI

MP MF MI

MF MP

=> = => =

 

NMI =KPN HNPP I

F H

D N K

O

(123)

Hướng dẫn giải

a) DH⊥AC (gt) DHC = ° 90

BD AD

BD BC BC / /AD

⊥ 

⇒ ⊥

 ⇒DBC = °90

Hai đỉnh H B, nhìn đoạn DC góc khơng đổi 90°⇒HBCD nội tiếp

trong đường trịn đường kính DC

b) D 1 C (1 1s BH

2

= = ® đường trịn đường kính DC)

  1

C =A (so le trong, AD BC// ) ⇒D 1 =A1

 

1

DOK=2A (Góc tâm góc nội tiếp chắn DK

( )

O )⇒DOK =2D1 =2BDH

c) 

AKB=90 (góc nội tiếp chắn nửa đường tròn) ⇒BKC =DHA= °90 , C 1=A1(c/m

trên)

AHD CKB

⇒ ∆ = ∆ (cạnh huyền – góc nhọn) ⇒AH=CK

AD=BD (∆ADBcân), AD=BC (c/m trên) ⇒AD=BD=BC

Gọi I=AC∩BD Xét ∆ADB vuông D, đường cao DH

Ta có : 2

BD =AD =AH.AI=CK.AI (hệ thức tam giác vuông) (1)

Tương tự : 2

BD =BC =CK.CI (2) Cộng vế theo vế (1) (2) ta được:

2 2

CK.AI CK.CI+ =2BD ⇒CK(AI CI)+ =2BD ⇒CK.CA=2BD (đpcm)

Câu 3. Cho tam giác ABC có đường cao AH Trên cạnh BC lấy điểm M (M

khơng trùng B C H, , ), từ M kẻ MP MQ, vuông góc với cạnh AB AC,

a) Chứng minh APMQ tứ giác nội tiếp xác định tâm O đường tròn

ngoại tiếp tứ giác

b) Chứng minh MP+MQ= AH

Hướng dẫn giải

1

1

B

H I

C K

O A

(124)

a) Ta có MPAB⇒APM = °90 , MQAC⇒AQM = °90 P Q nhìn

BC góc 90° nên P Q nằm đường trịn đường kính

AMAPMQ tứ giác nội tiếp

AM đường kính đường trịn ngoại tiếp tứ giác APMQ nên tâm O đường tròn

ngoại tiếp tứ giác APMQ trung điểm AM

b) Tam giác ABCAH đường cao

2

ABC

S BC AH

⇒ =

Tam giác ABMMP đường cao

2

ABM

S AB MP

⇒ =

Tam giác ACMMQ đường cao

2

ACM

S AC MQ

⇒ =

Ta có SABM +SACM =SABC

1 1

2AB MP 2AC MQ 2BC AH AB MP AC MQ BC AH

⇒ + = ⇒ + =

AB=BC=CA (vì tam giác ABC đều) MP+MQ= AH

Câu 4. Cho hình vng ABCD nội tiếp đường trịn

(

O R;

)

Gọi P điểm cung

nhỏ CD Chứng minh rằng: PA PC+ 2= PB

Hướng dẫn giải

Gọi K giao điểm PB AC

 

PCK =PBA(hai góc nội tiếp chắn cung AP)

 

CPK = APB( hai góc nội tiếp chắn cung nhau)

O

P

D

C A

B

K

M

B H C

Q A

(125)

( )

PCK PBA g g

⇒ ∆ ~ ∆ PC CK

PB AB

⇒ = (1)

 

PAK =PBC(góc nội tiếp chắn cung CP)

 

APK =CPB ( hai góc nội tiếp chắn cung nhau)

( )

PAK PBC g g

⇒ ∆ ~ ∆ PA AK

PB BC

⇒ = (2)

Cộng vế theo vế (1) (2), ta được: PA PC AK CK AK CK AC

PB PB BC AB AB AB

+

+ = + = = =

Hay : PA PC+ 2= PB (đpcm)

Câu 5. Cho tam giác ABC nội tiếp đường tròn tâm O D điểm cung BC

không chứa đỉnh A Gọi I E F, , hình chiếu D đường thẳng

, ,

BC AB AC Chứng minh rằng: BC AB AC DI = DE +DF

Hướng dẫn giải

Lấy K cạnh BC cho: CDK =ADB

Ta có CDK = ADB(cách vẽ)

 

DCK =DAB(góc nội tiếp chắn cung BD)

( )

CDK ADB g g

∆ ~ ∆

DI DE, thứ tự hai đường cao ∆CDKADB nên:

CK DI CK AB

AB DE DI DE

⇒ = ⇒ = (1)

Mặt khác: BDK  =BDA+ADK;   ADC=ADK+CDK : CDK =ADBBDK = ADC

Lại có : BDK =ADC; CBK =DAC (góc nội tiếp chắn cung CD)

( )

DBK DAC g g

⇒ ∆ ~ ∆

DI DF, thứ tự hai đường cao tương ứng ∆DBKDAC nên:

BK DI BK AC

AC DF DI DF

⇒ = => = (2)

Cộng (1) (2) vế theo vế, ta được: CK BK AB AC

DI + DI = DE+DF hay:

BC AB AC DI = DE+DF E

I C

F A

B

(126)

Chuyên đề 12 CÁC BÀI TẬP CỰC TRỊ HÌNH HỌC

Gồm 22 tập mẫu hướng dẫn chi tiết 22 tập tương tựđể rèn luyện A.Phương pháp giải tốn cực trị hình học

1 Dạng chung toán cực trị hình học:

“ Trong tất hình có chung tính chất, tìm hình mà đại lượng (độ dài đoạn thẳng, sốđo góc, sốđo diện tích …) có giá trị lớn giá trị nhỏ nhất” có thểđược cho dạng:

a) Bài tốn dựng hình

Ví dụ: Cho đường tròn (O) điểm P nằm đường trịn, xác định vị trí dây qua điểm P cho dây có độ dài nhỏ nhất

b) Bài tốn vểchứng minh

Ví dụ: Chứng minh dây qua điểm P đường trịn (O), dây vng góc với OP có độ dài nhỏ nhất

c) Bài tốn tính tốn

Ví dụ: Cho đường trịn (O;R) điểm P nằm đường trịn có OP = h, Tính độ dài nhỏ nhất dây qua P

2 Hướng giảibài tốn cực trị hình học:

a) Khi tìm vị trí hình H miền D cho biểu thức f có giá trị lớn ta phải chứng tỏđược:

+Với vị trí hình H miền D f ≤ m ( m số )

+Xác định vị trí hình H miền D cho f = m

b) Khi tìm vị trí hình H miền D cho biểu thức f có giá trị nhỏ ta phải chứng tỏđược:

+Với vị trí hình H miền D f ≥ m ( m số )

+Xác định vị trí hình H miền D để f = m

B Bài tập vận dụng

Phần I Một sốbài tập mẫu có lời giải chi tiết

Bài tập 1 Cho nửa đường trịn đường kính BC = 2R Từđiểm A nửa đường tròn vẽ AH ⊥

BC Nửa đường trịn đường kính BH, CH có tâm O1; O2 cắt AB, AC thứ tự D E

a) Chứng minh tứ giác ADHE hình chữ nhật, từđó tính DE biết R = 25 BH = 10 b) Chứng minh tứ giác BDEC nội tiếp đường trịn

c) Xác định vịtrí điểm A để diện tích tứgiác DEO1O2 đạt giá trị lớn Tính giá trị

Hướng dẫn giải:

O1 O2

D

O

B H C

A

(127)

a) Ta có BAC = 900 (vì góc nội tiếpchắn nửa đường trịn) Tương tự có BDH =CEH = 900

Xét tứ giác ADHE có A  =ADH=AEH = 900 => ADHE hình chữ nhật

Từđó DE = AH mà AH2 = BH.CH (Hệ thức lượng tam giác vuông) hay AH2 = 10 40 = 202 (BH = 10; CH = 2.25 - 10 = 40) => DE = 20

b) Ta có:BAH= C (góc có cạnh tương ứng vng góc) mà DAH =ADE (1)

(Vì ADHE hình chữ nhật) => C =ADE C BDE + = 1800 nên tứ giác BDEC nội tiếp đường tròn

c) Vì O1D = O1B =>∆O1BD cân O1 => B =BDO1 (2)

Từ(1), (2) =>ADE +BDO1 = B BAH + = 900 => O1D //O2E

Vậy DEO2O1 hình thang vng D E

Ta có Sht =

1 2

1 1

(O D O E).DE O O DE O O

2 + = ≤

(Vì O1D + O2E = O1H + O2H = O1O2 DE < O1O2 )

2 2

ht

1 BC R

S O O

2

≤ = =

Dấu "=" xảy DE =O1O2

⇔ DEO2O1 hình chữ nhật

⇔A điểm cung BC Khi maxSDEO2O1 =

2 R

Bài tập 2.Cho đường trịn (O), đường kính AB, d1, d2 các đường thẳng qua A, B vng góc với đường thẳng AB M, N điểm thuộc d1, d2 cho MON = 900

1) Chứng minh đường thẳng MN tiếp tuyến đường tròn (O) 2) Chứng minh AM AN =

4 AB

3) Xác định vị trí M, N để diện tích tam giác MON đạt giá trị nhỏ

Hướng dẫn giải:

1) Gọi H hình chiếu O đường thẳng MN Xét tứ giác OAMH

N

M

O

A B

(128)

    A+ =H 180 (do A= =H 90 )

=> OAMH tứ giác nội tiếp đường tròn

Tương tự tứ giác OANH nội tiếp

=> A   1 =M , B1 1=N1 (2 góc nội tiếp chắn cung)

   

1 1 A B M N 90

⇒ + = + = => AHB = 900

=> MN tiếp tuyến

2) Ta có AM = MH, BN = NH, theo hệ thức lượng tam vng, ta có: AM BN = MH NH = OH2 =

4 AB

(đpcm)

3

2 = ∆MON

S OH MN >

2

1 OH AB (Vì AMNB hình thang vuông)

Dấu “=” chỉkhi MN = AB hay H điểm cung AB

⇔M, N song song với AB ⇔AM = BN = AB

Vậy SMON nhỏ AM = BN =

AB

Bài tập 3. Cho∆ABC có góc nhọn, trực tâm H nội tiếp đường tròn (O) Vẽđường kính AK

a) Chứng minh tứ giác BHCK hình hình hành

b) Vẽ OM ⊥BC (M ∈ BC) Chứng minh H, M, K thẳng hàng AH = 2.OM

c) Gọi A’, B’, C’ chân đường cao thuộc cạnh BC, CA, AB của∆ABC Khi BC cốđịnh xác định vịtrí điểm A để tổng S = A’B’ + B’C’ + C’A’ đạt giá trị lớn

Hướng dẫn giải:

a) Ta có ACK = 900

(vì góc nội tiếp chắn nửa đường trịn) Nên CK ⊥AC mà BH ⊥AC (vì H trực tâm)

=> CK // BH tương tự có CH // BK => Tứgiác BHCK hbh (đpcm)

b) OM ⊥BC => M trung điểm BC

(định lý đường kính dây cung) => M trung điểm HK (vì BHCK hình bình hành) => đpcm ∆AHK có OM đường trung bình => AH = 2.OM

M

H O

B A

(129)

c) Ta có AC C ′ =BB C′ = 900=> tứ giác BC’B’C nội tiếp đường tròn => AC B′ ′ = ACBmà

 

ACB=BAx (Ax tiếp tuyến A) => Ax // B’C’

OA ⊥Ax => OA ⊥ B’C’ Do SAB’OC’ =

1R.B’C’

Tương tự: SBA’OC’ =

2

1R.A’C’; SCB’OA’ =

1R.A’B’

ABC

S∆ =

R(A’B’ + B’C’ + C’A’)=

AA’.BC <

2

(AO + OM).BC

⇒ A’B’ + B’C’ + C’A’, lớn A, O, M thẳng hàng ⇔ A đỉểm cung lớn BC

Bài tập 4.Cho đường tròn (O), đường kính AB cốđịnh, điểm I nằm A O cho AI =

3AO

Kẻ dây MN vng góc với AB I, gọi C điểm tùy ý thuộc cung lớn MN cho C không trùng với M, N B Nối AC cắt MN E

1) Chứng minh tứ giác IECB nội tiếp 2) Chứng minh hệ thức: AM2 = AE.AC

3) Hãy xác định vị trí điểm C cho khoảng cách từN đến tâm đường tròn ngoại tiếp tam giác CME nhỏ

Hướng dẫn giải:

O1 E

I

C

O

N M

B A

1 Theo giả thiết MN ⊥AB I

 

ACB = 90 hay ECB = 90

 

EIB + ECB = 180

mà hai góc đối tứ giác IECB nên tứ giác IECB tứ giác nội tiếp

2 Theo giả thiêt MN ⊥AB, suy A điểm MN nênAMN = ACM  (hai

góc nội tiếp chắn hai cung nhau) hayAME = ACM , lại có CAM góc chung tam giác AMEđồng dạng với tam giác ACM AM = AE

AC AM

⇒ ⇒ AM2 = AE.AC

3 Theo AMN = ACM   ⇒ AM tiếp tuyến đường trịn ngoại tiếp ∆ECM Nối

MB ta có AMB= 900, tâm O1 đường trịn ngoại tiếp ∆ECM phải nằm

(130)

Ta thấy NO1 nhỏ NO1 khoảng cách từN đến BM⇒NO1 ⊥BM Gọi O1

chân đường vuông góc kẻ từ N đến BM ta O1 tâm đường trịn ngoại tiếp ∆

ECM có bán kính O1M

Do để khoảng cách từ N đến tâm đường tròn ngoại tiếp ∆ ECM nhỏ C phải giao điểm đường trịn (O1), bán kính O1M với đường trịn (O)trong O1

là hình chiếu vng góc N BM

Bài tập 5.Cho đường tròn ( O; R ) điểm A nằm ngồi đường trịn cho OA = R Từ A

vẽ tiếp tuyến AB, AC với đường tròn (B, C tiếp điểm) Lấy D thuộc AB; E thuộc AC cho chu vi tam giác ADE 2R

a) Chứng minh tứ giác ABOC hình vng

b) Chứng minh DE tiếp tuyến đường tròn (O; R) c) Tìm giá trị lớn diện tích ∆ADE

Hướng dẫn giải:

a) Ta có:  

ABO=ACO=90 (tính chất tiếp tuyến) (1)

AB = AC 2

OA OB

= − = R = OB = OC (2)

Từ(1) (2) suy ABOC hình vng b) Theo ta có: AD + DE + AE = 2R (3)

Suy ra: DE = BD + CE (4)

Vẽ OM ⊥ DE (M∈DE) (5)

Trên tia đối tia CA lấy điểm F cho CF = BD; suy ∆BDO = ∆COF (c-g-c)

⇒OD = OF; lại có DE = FE nên ∆ODE = ∆OFE (c-c-c)

⇒OM = OC = R

(hai đường cao tương ứng) (6) Từ (5) (6) suy DE tiếp tuyến đường tròn

(O;R)

c) Đặt: AD = x; AE = y ADE S xy

2

⇒ = (x, y > 0)

Ta có: DE 2 2

AD AE x + y

= + = (định lí Pitago)

Vì AD + DE + AE = 2R 2

x + y + x y

(131)

Áp dụng BĐT – Cơsi cho hai số khơng âm ta có:

2

x + y ≥2 xy x + y ≥ 2xy (7)

Dấu “=” xảy x = y

Từ(6) (7) suy ra: xy+ 2xy≤2R ⇔ xy 2

(

+ 2

)

≤2R

(

2R

)

xy

2+

⇔ ≤ ⇔xy 2R2

3 2

+ ⇒SADE

(

)

2

2 ADE

R

S - 2 R 2

≤ ⇔ ≤

+

Vậy max SADE =

(

)

3 2 R− ⇔x = y⇔∆ADE cân A

Bài tập 6.Cho đường (O, R) đường thẳng d khơng qua O cắt đường trịn hai điểm A, B Lấy điểm M tia đối tia BA kẻ hai tiếp tuyến MC, MD với đường tròn (C, D

các tiếp điểm) Gọi H trung điểm AB

1) Chứng minh điểm M, D, O, H nằm đường tròn

2) Đoạn OM cắt đường tròn I Chứng minh I tâm đường tròn nội tiếp tam giác MCD

3) Đường thẳng qua O, vng góc với OM cắt tia MC, MD thứ tự P Q Tìm vị trí điểm M d cho diện tích tam giác MPQ bé

Hướng dẫn giải:

1) Vì H trung điểm AB nên OHAB hay  90

OHM = Theo tính chất tiếp

tuyến ta lại có ODDM hay ODM=900 Suy điểm M, D, O, H nằm

một đường trịn

2) Theo tính chất tiếp tuyến, ta có MC = MD ⇒∆MCD cân M ⇒ MI đường phân giác CMD Mặt khác I điểm cung nhỏ CD nên 

2

DCI = sđDI =

2sđ 

CI = MCI

⇒ CI phân giác MCD Vậy I tâm đường tròn nội tiếp tam giác MCD

3) Ta có tam giác MPQ cân M, có MO đường cao nên diện tích tính:

1

2 ( )

2

OQM

S = S = OD QM =R MD+DQ Từ S nhỏ ⇔ MD + DQ nhỏ Mặt

d

I

B A

O M

C

D H

(132)

khác, theo hệ thức lượng tam giác vng OMQ ta có 2

DM DQ=OD =R không đổi nên MD + DQ nhỏ ⇔ DM = DQ = R Khi OM = R hay M giao điểm

của d với đường trịn tâm O bán kính R

Bài tập 7 Cho hai đường tròn (O) và(O )′ cắt A B Vẽ AC, AD thứ tựlà đường kính

của hai đường trịn (O) (O )′

a) Chứng minh ba điểm C, B, D thẳng hàng

b) Đường thẳng AC cắt đường tròn(O )′ E; đường thẳng AD cắt đường tròn (O) F (E, F khác A) Chứng minh điểm C, D, E, F nằm đường tròn

c) Một đường thẳng d thay đổi qua A cắt (O) và(O )′ thứ tự M N Xác định vị trí d đểCM + DN đạt giá trị lớn

Hướng dẫn giải:

a) Ta có ABC ABDlần lượt góc nội tiếp chắn nửa đường trịn (O) (O/)

 

ABC ABD 90

⇒ = =

Suy C, B, D thẳng hàng

 

CFD CED 90

⇒ = = suy CDEF tứ giác nội tiếp

c) Ta có  

CMA=DNA=90 (góc nội tiếp chắn nửa đường trịn); suy CM // DN hay

CMND hình thang

Gọi I, K thứ tựlà trung điểm MN CD Khi IK đường trung bình hình thang CMND Suy IK // CM // DN (1) CM + DN = 2.IK (2)

Từ(1) suy IK ⊥ MN ⇒ IK ≤ KA (3) (KA số A K cốđịnh)

Từ(2) (3) suy ra: CM + DN≤ 2KA Dấu “ = ” xảy IK = AK⇔d ⊥

AK A

Vậy đường thẳng d vng góc AK A (CM + DN) đạt giá trị lớn 2KA

Bài tập 8. Từ điểm A nằm ngồi đường trịn (O;R) ta vẽ hai tiếp tuyến AB, AC với đường tròn (B, C tiếp điểm) Trên cung nhỏ BC lấy điểm M, vẽ MI⊥AB, MK⊥AC (I∈

AB,K∈AC)

d

K I

N M

F E

O/ O

C

D B

A

b) Xét tứ giác CDEF có:

 

CFD=CFA=90 (góc nội tiếp chắn nửa đường tròn (O))

 

(133)

a) Chứng minh: AIMK tứ giác nội tiếp đường tròn b) Vẽ MP⊥BC (P∈BC) Chứng minh: MPK =MBC

c) Xác định vị trí điểm M cung nhỏBC đểtích MI.MK.MP đạt giá trị lớn

Hướng dẫn giải:

a) Ta có: 

AIM=AKM=90 (gt), suy tứ giác AIMK nội tiếp đường tròn đường kính

AM

b) Tứ giác CPMK có  

MPC=MKC=90 (gt) Do CPMK tứ giác nội tiếp

 

MPK MCK

⇒ = (1) Vì KC tiếp tuyến (O) nên ta có: MCK =MBC (cùng chắn MC ) (2) Từ(1) (2) suy MPK =MBC(3)

c) Chứng minh tương tự câu b ta có BPMI tứ giác nội tiếp Suy ra: MIP =MBP(4) Từ(3) (4) suy MPK =MIP Tương tự ta chứng minh MKP =MPI

Suy ra: MPK~ ∆MIP⇒ MP MI

MK = MP ⇒MI.MK = MP2 ⇒ MI.MK.MP = MP3

Do MI.MK.MP lớn MP lớn (4)

- Gọi H hình chiếu O BC, suy OH số(do BC cốđịnh)

Lại có: MP + OH ≤ OM = R⇒ MP ≤ R – OH Do MP lớn R – OH O, H, M thẳng hàng hay M nằm cung nhỏBC (5) Từ(4) (5) suy max (MI.MK.MP) = ( R – OH )3 ⇔M nằm cung nhỏ BC

Bài tập 9. Cho nửa đường trịn đường kính AB = 2R Từ A B kẻ hai tiếp tuyến Ax, By Qua

điểm M thuộc nửa đường tròn kẻ tiếp tuyến thứ ba cắt tiếp tuyến Ax, By

C D Các đường thẳng AD BC cắt N 1) Chứng minh AC + BD = CD

2) Chứng minh ∠COD = 90°

3) Chứng minh AC BD = 4) Chứng minh OC // BM

5) Chứng minh AB tiếp tuyến đường trịn đường kính CD

6) Chứng minh MN ⊥ AB

H

O P

K I

M

C B

A

(134)

7)Xác định vị trí M để chu vi tứgiác ACDB đạt giá trị nhỏ

Hướng dẫn giải:

Theo tính chất hai tiếp tuyến cắt ta có: CA = CM; DB = DM => AC + BD = CM + DM

Mà CM + DM = CD => AC + BD = CD

1 Theo tính chất hai tiếp tuyến cắt ta có: OC tia phân giác góc AOM; OD tia phân giác góc BOM, mà ∠AOM ∠BOM hai góc kề bù => ∠COD = 900 Theo ∠COD = 900 nên tam giác COD vng O có OM ⊥ CD ( OM tiếp tuyến )

Áp dụng hệ thức cạnh đường cao tam giác vuông ta có OM2 = CM DM, Mà OM = R; CA = CM; DB = DM => AC BD =R2 => AC BD =

3 Theo ∠COD = 900 nên OC ⊥ OD(1)

Theo tính chất hai tiếp tuyến cắt ta có: DB = DM; lại có OM = OB =R => OD trung trực BM => BM ⊥ OD(2) Từ(1) Và (2) => OC // BM ( Vì vng góc với OD)

4 Gọi I trung điểm CD ta có I tâm đường trịn ngoại tiếp tam giác COD

đường kính CD có IO bán kính

Theo tính chất tiếp tuyến ta có AC ⊥ AB; BD ⊥ AB => AC // BD => tứ giác ACDB hình thang Lại có I trung điểm CD; O trung điểm AB => IO đường trung bình hình thang ACDB

IO // AC, mà AC ⊥ AB => IO ⊥ AB O => AB tiếp tuyến O đường trịn

đường kính CD

5 Theo AC // BD => , mà CA = CM; DB = DM nên suy => MN // BD mà BD ⊥ AB => MN ⊥ AB

6 Ta có chu vi tứ giác ACDB = AB + AC + CD + BD mà AC + BD = CD nên suy chu vi tứgiác ACDB = AB + 2CD mà AB không đổi nên chu vi tứ giác ACDB nhỏ CD nhỏ nhất, mà CD nhỏ CD khoảng cách giữ Ax By tức CD vng góc với Ax By Khi CD // AB => M phải trung điểm cung AB

Bài tập 10 (SởThái Bình 2015 – 2016) Cho nửa đường trịn tâm O đường kính AB = 2R Điểm M di chuyển nửa đường tròn (M khác A B) C trung điểm dây cung AM

Đường thẳng d tiếp tuyến với nửa đường tròn B Tia AM cắt d điểm N Đường thẳng OC cắt d E

/ /

y x

N C

D I

M

B O

A

4 AB

BD AC BN CN =

(135)

a) Chứng minh: tứ giác OCNB nội tiếp

b) Chứng minh: ACAN = AO.AB c) Chứng minh: NO vng góc với AE

d) Tìm vịtrí điểm M cho (2.AM + AN) nhỏ

Hướng dẫn giải:

6

a) Phần đường kính OC qua trung điểm C AM ⇒ OC ⊥ AM ⇒  o OCN=90

BN tiếp tuyến (O) B ⇒ OB ⊥ BN ⇒  o OBN=90

Xét tứ giác OCNB có tổng hai góc đối:   o o o OCN+OBN=90 +90 =180 Do tứ giác OCNB nội tiếp

b) Xét ∆ACO ∆ABN có: A1 chung; ACO = ABN=90o

⇒∆ACO ~ ∆ABN (g.g)

⇒ AC AO

AB= AN

Do ACAN = AO.AB (đpcm) c) Theo chứng minh trên, ta có:

OC ⊥ AM ⇒ EC ⊥ AN ⇒EC đường cao ∆ANE (1)

OB ⊥ BN ⇒ AB ⊥ NE ⇒AB đường cao ∆AME (2)

Từ(1) (2) suy O trực tâm ∆ANE (vì O giao điểm AB EC)

⇒NO đường cao thứ ba ∆ANE

Do đó; NO ⊥AE (đpcm)

d) Ta có: 2.AM + AN = 4AC + AN (vì C trung điểm AM) 4ACAN = 4AO.AB = 4R.2R = 8R2

Áp dụng bất đẳng thức Cơ-si cho hai sốdương, ta có:

4AC + AN ≥

2 4AC.AN= 8R = 2R

⇒ Tổng 2.AM + AN nhỏ = 2R⇔ 4AC = AN

⇔ AN = 2AM ⇔M trung điểm AN

∆ABN vuông B có BM đường trung tuyến nên AM = MB

⇒ AM =BM ⇒M điểm nửa đường trịn đường kính AB 1

E N

C

O

A B

(136)

Vậy với M điểm nửa đường trịn đường kính AB (2.AM + AN) nhỏ

nhất = 2R

Bài tập 11 (Sở Hải Dương 2015 – 2016) Cho đường trịn (O) đường kính AB cốđịnh đường

kính CD thay đổi khơng trùng với AB Tiếp tuyến A đường tròn (O) cắt

đường thẳng BC BD E F Gọi P Q trung điểm

đoạn thẳng AE AF

1) Chứng minh ACBD hình chữ nhật;

2) Gọi H trực tâm tam giác BPQ Chứng minh H trung điểm OA;

3) Xác định vị trí đường kính CD để tam giác BPQ có diện tích nhỏ

Hướng dẫn giải:

a) Có   

90

ACB=CBD=ADB= ( Các góc nội tiếp chắn

nửa đường trịn)

⇒Tứ giác ACBD hình chữ nhật ( Tứ giác có ba góc vng)

b) Có PO đường trung bình tam giác AEB ⇒PO // EB mà EB ⊥ BF⇒ PO⊥BF

Xét tam giác PBF có BA ⊥PF; PO⊥BF nên BA PO đường cao tam giác PBF mà BA PO căt

tại O nên O trực tâm tam giác PBF⇒FO đường cao thứ ba tam giác PBF hay FO⊥PB (1)

Lại có H trực tâm tam giác PBQ nên QH ⊥ PB

(2)Từ (1) (2) ⇒ QH // FO Xét tam giác AOF có Q

trung điểm AF; QH // FO nên H trung điểm AO

c) ( ) ( )

2

BPQ

S = AB AP+AQ = AB AE+AF (3)

Áp dụng bất đẳng thức Cô si với hai số không âm AE AF ta có: AE + AF

2 AE AF

≥ (4)

( Dấu “=” xảy ⇔AE =AF) Từ(3) (4)

2

BPQ

SAB AE AF

⇒ ≥ (5)

Lại có: Áp dụng hệ thức tam giác vng EBF ta có: AE.AF = AB2 (6) Từ(5) (6) ta có SBPQ

2 AB

Xảy dấu AE = AF

⇒ Tam giác EBF vuông cân B

⇔ACBD hình vng nên CD vng góc AB

Vậy: Khi đường kính CD vng góc với đường kính AB tam giác PBQ có diện tích nhỏ

O H

Q P

F E

D C

(137)

Bài tập 12 (SởVĩnh Phúc năm 2009 – 2010) Trên đoạn thẳng AB cho điểm C nằm A B Trên nửa mặt phẳng có bờ AB kẻ hai tia Ax By vng góc với AB Trên tia Ax lấy điểm I, tia vng góc với CI C cắt tia By K Đường tròn đường kính IC cắt IK P ( P khác I)

a) Chứng minh tứ giác CPKB nội tiếp đường tròn, chỉrõ đường tròn b) Chứng minh CIP =PBK

c) Giả sử A, B, I cốđịnh Hãy xác định vị trí điểm C cho diện tích tứ giác ABKI lớn

Hướng dẫn giải:

a) Có:  

90

CPK =CPI = (góc nội tiếp chắn nửa đường trịn);

Do ByAB nên CBK =900

Suy ra:  

180

CPK +CBK = hay tứ giác CPKB nội tiếp đường trịn đường kính CK

b) Ta có: CIP =PCK (góc nội tiếp góc tạo tia tiếp tuyến dây chắn

một cung); (1)

Mặt khác tứ giác PCBK nội tiếp nên: PCK =PBK (2)

Từ(1) (2) ta có điều phải chứng minh

c) Từ giả thiết suy tứ giác AIKB hình thang vng, gọi s diện tích AIKB,

đó ta có: 1( )

s= AI +KB AB Dễ thấy s lớn KB lớn (do A, B, I

cốđịnh)

Xét tam giác vng AIC BKC có: KCCI KBCA suy ra: BKC = ACI (góc có cạnh tương ứng vng góc) hay ∆ACI đồng dạng với ∆BKC(g-g)

Suy ra: AC AI BK AC BC

BK = BC ⇔ = AI , đó: BK lớn ⇔ACBC lớn Theo BĐT Cơsi có:

2 2

2

AC CB AB AC CB≤ +  =

  , dấu “=” xảy C

trung điểm AB Vậy diện tích tứ giác AIBK lớn Clà trung điểm AB

Bài tập 13 (SởĐà Nẵng 2009)Cho đường tròn (O), đường kính AB cốđịnh, điểm I nằm A O cho AI =

2

3

AO Kẻ dây MN vng góc với AB I Gọi C điểm tùy ý

thuộc cung lớn MN cho C không trùng với M, N B Nối AC cắt MN E a) Chứng minh tứ giác IECB nội tiếp đường tròn

A C B

K

y

I

x

(138)

b) Chứng minh ∆AME ∆ACM AM2 = AE.AC c) Chứng minh AE.AC - AI.IB = AI2

d) Hãy xác định vị trí điểm C cho khoảng cách từN đến tâm đường tròn ngoại tiếp tam giác CME nhỏ

Hướng dẫn giải:

a) *

EIB

=

90

(giả thiết)

*

ECB

90

=

(góc nội tiếp chắn nửa đường trịn) * Kết luận: Tứ giác IECB tứ giác nội tiếp

b) Ta có:

* sđ

cungAM

= sđ

cungAN

*

AME

= ∠

ACM

*GócAchung,suyra∆AME ∆ACM * Do đó:

AC

AM

AM

=

AE

AM2 = AE.AC

c) * MI đường cao tam giác vuông MAB nên MI2 = AI.IB * Trừ vế hệ thức câu b) với hệ thức

* Ta có: AE.AC - AI.IB = AM2 - MI2 = AI2

d) * Từ câu b) suy AM tiếp tuyến đường tròn ngoại tiếp tam giác CME Do

tâm O1 đường trịn ngoại tiếp tam giác CME nằm BM Ta thấy khoảng cách NO1 nhỏ NO1

BM.)

* Dựng hình chiếu vng góc N BM ta O1 Điểm C giao đường

tròn cho với đường trịn tâm O1, bán kính O1M

Bài tập 14 (SởPhú Yên 2009 – 2010)Cho hình bình hành ABCD có đỉnh D nằm đường

trịn đường kính AB = 2R Hạ BN DM vng góc với đường chéo AC a) Chứng minh tứ giác: CBMD nội tiếp

b) Chứng minh rằng: DBDC = DN.AC

c) Xác định vị trí điểm D để diện tích hình bình hành ABCD có diện tích lớn tính diện tích trường hợp

Hướng dẫn giải:

A

B

M

E

C

I

O

1

(139)

a Góc ADB = 900 (Góc nội tiếp chắn nửa đường trịn)

mà AD//BC (gt) => DB⊥BC

Xét tứ giác DMBC có góc DMC = góc DBC =900 => Tứ giác nội tiếp b Ta có ∆DBN đồng dạng với ∆CAD

(  DAC=DBN,   BDN =BAN =DCA)

=>

DC

DN DB

AC

= => DBDC = DN.AC c SABCD = DH.AB

Do AB không đổi = 2R

=> SABCD max ⇔DH max ⇔ D nằm cung AB

Bài tập 15.(Sở Hải Dương 2009 – 2010)Cho đường tròn (O), dây AB không qua tâm Trên cung nhỏ AB lấy điểm M (M không trùng với A, B) Kẻ dây MN vng góc với AB H Kẻ MK vng góc với AN

(

K AN

)

1) Chứng minh: Bốn điểm A, M, H, K thuộc đường tròn

2) Chứng minh: MN phân giác góc BMK

3) Khi M di chuyển cung nhỏ AB Gọi E giao điểm HK BN Xác định vị trí điểm M để (MK.AN + ME.NB) có giá trị lớn

Hướng dẫn:

Chú ý: Kể trường hợp đặc biệt MN qua O

1) Từ giả thiết:

AKM 90

=

0,

AHM 90

=

H M

N

O D

C

B A

O

N K

H

E

B A

(140)

Bốn điểm A, K, H, M thuộc đường tròn

2)

NAH

=

NMK

=

1

2

KH

(1)

 

NAH NMB

=

=

1

2

NB

(2)

Từ (1) (2)

NMK NMB

 

=

⇒ MN phân giác góc KMB

3)

MAB MNB

 

1

2

=

=

sđMB;

MAB MKH

 

1

2

=

=

MH

 

MNB MKH

=

K,M,E,N

cùng thuộc đường tròn

 

MEN MKN 180

ME NB

+

=

MAN MNB AMBN

1

1

1

S

MK.AN; S

ME.NB; S

MN.AB

2

2

2

MK.AN ME.BN MN.AB

=

=

=

+

=

(

MK.NA ME.NB

)

+

lớn ⇔ MN.AB lớn

⇔MN lớn (Vì AB= const ) ⇒ M AB

Bài tập 16 (Sở Bắc Giang 2009 – 2010)Cho đường tròn tâm O đường kính AB cố định H thuộc đoạn thẳng OA( H khác A;O trung điểm OA) Kẻ dây MN vng góc với AB H MN cắt AK E

1 Chứng minh tứ giác HEKB nội tiếp

2 Chứng minh tam giác AME đồng dạng với tam giác AKM

3 Cho điểm H cố định, xác định vị trí K để khoảng cách từ N đến tâm đường tròn ngoại tiếp tam giác MKE nhỏ

Hướng dẫn:

1/ ∆AHI vuông H (vì CA⊥HB)

∆AHI nội tiếp đường trịn đường kính AI ∆AKI vng H (vì CK⊥AB)

∆AKI nội tiếp đường trịn đường kính AI

Vậy tứ giác AHIK nội tiếp đường trịn đường kính AI

Ta có CA⊥HB( Gt)

CA⊥DC( góc ACD chắn nửa đường tròn)

=> BH//CD hay BI//CD (1)

.

A

B

C

D

M

I

O

H

(141)

Ta có AB⊥CK( Gt)

AB⊥DB( góc ABD chắn nửa đường trịn)

=> CK//BD hay CI//BD (2)

Từ (1) (2) ta có Tứ giác BDCI hình bình hành( Có hai cặp cạnh đối song song) Mà DI cắt CB M nên ta có MB = MC

=> OM⊥BC( đường kính qua trung điểm dây vng góc với dây đó)

2/

Vì BD tia phân giác góc B tam giác ABC; nên áp dụng tính chất đường phân giác ta có:

AB BC BC AB BC AB DC AD

2 = ⇒ = ⇔

=

Vì ∆ABC vng A mà BC = 2AB nên ACB = 300; ABC = 600

Vì B1 = B2(BD phân giác) nên ABD = 300

Vì ∆ABD vng A mà ABD = 300 nên BD = 2AD = 2 = 4cm

=> AB2 =BD2 −AD2 =16−4=12

Vì ∆ABC vng A => BC = AC2 + AB2 = 36+12 =4

Vì CH tia phân giác góc C tam giác CBD; nên áp dụng tính chất đường phân

giác ta có: BH DH

HB DH HB DH BC DC 3 4 = ⇒ = ⇔ =

Ta có: (1 3)

3 3 = + ⇒     = = + ⇔    = = + BH HD BH HD BH HD BH HD BH ) ( 2 ) ( ) (

4 = − = −

+ =

BH Vậy BH =2 3( 3−1)cm

Bài tập 17. Cho nửa đường trịn đường kính AB = 2R Từ A B kẻ hai tiếp tuyến Ax, By Qua

điểm M thuộc nửa đường tròn kẻ tiếp tuyến thứ ba cắt tiếp tuyến Ax, By

C D Các đường thẳng AD BC cắt N Chứng minh AC + BD = CD

2 Chứng minh ∠COD = 900

3 Chứng minh

4

AC BD= AB

4 Chứng minh OC // BM

5 Chứng minh AB tiếp tuyến đường tròn đường kính CD

6 Chứng minh MN ⊥ AB

D

A

B

C

E

H

1

(142)

7.Xác định vị trí M để chu vi tứgiác ACDB đạt giá trị nhỏ

Hướng dẫn giải:

/

/

y x

N C

D I

M

B O

A

Theo tính chất hai tiếp tuyến cắt ta có: CA = CM; DB = DM => AC + BD = CM + DM

Mà CM + DM = CD => AC + BD = CD

Theo tính chất hai tiếp tuyến cắt ta có: OC tia phân giác góc AOM; OD tia phân giác góc BOM, mà ∠AOM ∠BOM hai góc kề bù => ∠COD = 900 Theo ∠COD = 900 nên tam giác COD vuông O có OM ⊥CD ( OM tiếp tuyến )

áp dụng hệ thức cạnh đường cao tam giác vng ta có OM2 = CM DM, Mà OM = R; CA = CM; DB = DM => AC BD =R2 => AC BD =

4 AB

Theo ∠COD = 900 nên OC ⊥ OD(1)

Theo tính chất hai tiếp tuyến cắt ta có: DB = DM; lại có OM = OB =R => OD trung trực BM => BM ⊥ OD(2) Từ(1) Và (2) => OC // BM ( Vì vng góc với OD)

Gọi I trung điểm CD ta có I tâm đường tròn ngoại tiếp tam giác COD đường kính CD có IO bán kính

Theo tính chất tiếp tuyến ta có AC ⊥ AB; BD ⊥ AB => AC // BD => tứ giác ACDB hình thang Lại có I trung điểm CD; O trung điểm AB => IO đường trung bình hình thang ACDB

=> IO // AC, mà AC ⊥ AB => IO ⊥ AB O => AB tiếp tuyến O đường trịn

đường kính CD

6 Theo AC // BD =>

BD AC BN

CN = , mà CA = CM; DB = DM nên suy

DM CM BN CN =

=> MN // BD mà BD ⊥ AB => MN ⊥ AB

7 ( HD): Ta có chu vi tứ giác ACDB = AB + AC + CD + BD mà AC + BD = CD nên suy chu vi tứ giác ACDB = AB + 2CD mà AB không đổi nên chu vi tứ giác ACDB nhỏ

nhất CD nhỏ nhất, mà CD nhỏ CD khoảng cách giữ Ax By tức CD vng góc với Ax By Khi CD // AB => M phải trung điểm cung AB

Bài tập 18.Cho (O),dây cung AB Từ điểm M cung AB(M≠A M≠B),kẻ dây cung MN vng góc với AB H.Gọi MQ đường cao tam giác MAN

(143)

2 C/m:NQ.NA=NH.NM

3 C/m Mn phân giác góc BMQ

4 Hạ đoạn thẳng MP vng góc với BN;xác định vị trí M cung AB để MQ.AN+MP.BN có giác trị lớn

Hướng dẫn giải:

Có hình vẽ,cách c/m tương tự Sau C/m hình 9-a

1/ C/m:A,Q,H,M nằm đường trịn.(Tuỳ vào hình vẽ để sử dụng phương pháp sau:-Cùng làm với hai đàu …mot goc vuong

-Tổng hai góc đối

2/C/m: NQ.NA=NH.NM

Xét hai ∆vuông NQM ∆NAH đồng dang

3/C/m MN phân giác góc BMQ Có hai cách:

 Cách 1: Gọi giao điểm MQ AB I.C/m tam giác MIB cân M Cách 2: Góc QMN=NAH(Cùng phụ với góc ANH)

Góc NAH=NMB(Cùng chắn cung NB)⇒đpcm

4/ xác định vị trí M cung AB để MQ.AN+MP.BN có giác trị lớn Ta có 2S∆MAN=MQ.AN

2S∆MBN=MP.BN

2S∆MAN + 2S∆MBN = MQ.AN+MP.BN

Ta lại có: 2S∆MAN + 2S∆MBN =2(S∆MAN + S∆MBN)=2SAMBN=2

2 MN AB×

=ABMN

Vậy: MQ.AN+MP.BN=ABMN

Mà AB khơng đổi nên tích ABMN lớn ⇔MN lớn nhất⇔MN đường kính

⇔M điểm cung AB

Bài tập 19. Cho đường trịn(O) đường kính AB Gọi I trung điểm OA Vẽ đường tron tâm I qua A,trên (I) lấy Pbất kì, AP cắt (O) Q

1 Chứng minh đường tròn(I) (O) tiếp xúcnhau A Chứng minh IP // OQ

3 Chứng minh AP = PQ

(144)

Hướng dẫn giải:

1 Ta có OI = OA – IA mà OA IA bán kính đường trịn(O) đường tròn(I) Vậy đường tròn(O) đường tròn(I) tiếp xúcnhau A

2 ∆OAQ cân O ( OA OQ bán kính ) => ∠A1 = ∠Q1

∆IAP cân I ( IA IP bán kính ) => ∠A1 = ∠P1 => ∠P1 = ∠Q1 mà hai góc đồng vị nên suy IP // OQ

H I O

Q

P

B A

1

1

3 ∠APO = 900 (nội tiếp chắn nửa đường tròn ) => OP ⊥ AQ => OP đường cao

∆OAQ mà ∆OAQ cân O nên OP đường trung tuyến => AP = PQ

4 (HD) Kẻ QH ⊥ AB ta có SAQB =

2ABQH mà AB đường kính khơng đổi nên SAQB lớn QH lớn QH lớn Q trùng với trung điểm cung AB

Để Q trùng với trung điểm cung AB P phải trung điểm cung AO

Thật P trung điểm cung AO => PI ⊥ AO mà theo PI // QO => QO ⊥ AB

tại O => Q trung điểm cung AB H trung với O; OQ lớn nên QH lớn

Bài tập 20 (SởHà Tĩnh 2009 – 2010)Cho đường trịn tâm O có đường kính CD, IK (IK

không trùng CD)

1 Chứng minh tứ giác CIDK hình chữ nhật

2 Các tia DI, DK cắt tiếp tuyến C đường tròn tâm O thứ tựở G; H a Chứng minh điểm G, H, I, K thuộc đường tròn

b Khi CD cốđịnh, IK thay đổỉ, tìm vị trí G H diện tích tam giác DIJ đạt giá trị nhỏ

Hướng dẫn giải:

1 Ta có CD đường kính, nên:

∠CKD = ∠CID = 900 (T/c góc nội tiếp) Ta có IK đường kính, nên:

∠KCI = ∠KDI = 900 (T/c góc nội tiếp) Vậy tứ giác CIDK hình chữ nhật

2 a Vì tứ giác CIDK nội tiếp nên ta có:

∠ICD = ∠IKD (t/c góc nội tiếp)

Mặt khác ta có: ∠G = ∠ICD (cùng phụ với ∠GCI)

(145)

Vậy tứ giác GIKH nộitiếp b Ta có: DC ⊥GH (t/c)

⇒ DC2 = GCCH mà CD đường kính,nên độ dài CD khơng đổi

⇒ GC CH khơng đổi

Để diện tích ∆GDH đạt giá trị nhỏ GH đạtgiá trị nhỏ Mà GH = GC + CH nhỏ GC = CH

Khi GC = CH ta suy ra: GC = CH = CD IK ⊥CD

Bài tập 21 (SởKhánh Hòa 2009 - 2010) Cho đường tròn (O; R) Từ điểm M nằm (O; R) vẽ hai tiếp tuyến MA MB (A, B hai tiếp điểm) Lấy điểm C cung nhỏ AB (Ckhác với A B) Gọi D, E, F la hình chieu vuong goc cua C

AB, AM, BM

a Chứng minh AECD tứ giác nội tiếp b Chứng minh: CDE CBA =

c Gọi I giao điểm AC vàED, K giao điểm CB DF Chứng minh IK//AB d Xác định vị trí điểm C cung nhỏAB để (AC2 + CB2) nhỏ Tính giá trị nhỏ OM = 2R

Hướng dẫn giải:

a Chứng minh AECD l tứ gic nội tiếp

Xét tứ giác AECD ta có:

- Hai góc đối

  90 ( ; )

AEC ADC= =  CD AB CE AM⊥ ⊥

Nn tổng chng b

Do đótứ giác AECD nội tiếp đường trịn b Chứng minh: CDE CBA =

Tứ giác AECD nội tiếp đường tròn

 = ( )

CDE CA E chắn cung CE ĐiểmC thuộc cung nhỏ AB nên:

 = ( )

CA E CBA cuøng chaén cung CA

Suy ra: CDE CBA =

c Chứng minh IK//AB

 

   

        

 

1 2

0

Xét DCE BCA ta có:

D ( )

DCE KCI

E ( )

EAD IDK( ; )

EAD DCE 180 ( nội tiếp)

KCI IDK 180 B cmt

A cùngchắncungCD

A D A D FBC

tứ giác AECD  =  ⇒ =  =  = = = = + = ⇒ + =  

Suy tứ gic ICKD nội tiếp => CIK CDK cùngchắn =

(

CK

)

A

B

M

C

D

E

F

I

K

A

D D

A

(146)

M CAB CDK cùngchắn =

(

CBF

)

Suy CIK CBA ở =

(

vị trí đồng vị

)

IK//AB (đpcm)

d Xác định vị trí điểm C cung nhỏ AB để (AC2 + CB2) nhỏ Tính giá trị nhỏ OM = 2R

Gọi N trung điểm AB Ta có :

AC2 + CB2 = 2CD2 + AD2 + DB2 =2(CN2 – ND2) + (AN+ND)2 + (AN – ND)2 = 2CN2 – 2ND2 + AN2 + 2AN.ND + ND2 + AN2 – 2AN.ND + ND2

= 2CN2 + 2AN2 = 2CN2 + AB2/2

AB2/2 ko đổi =>CA2 + CB2 đạt GTNN CN đạt GTNN C giao điểm ON

vớicung nhỏ AB => C điểm cung nhỏ AB

Khi OM = 2R OC = R hay C trung điểm OM => CB = CA = MO/2 = R

Do đó: Min (CA2 + CB2 ) = 2R2

Phần II BÀI TẬP TƯƠNG TỰ

Bài tập (Sở Quảng Ngãi 2015 – 2016) Cho nửa đường trịn đường kính AB C điểm nằm hai điểm A B Trên nửa mặt phẳng có bờ AB chứa nửa đường trịn, vẽ hai tia Ax By tiếp xúc với nửa đường tròn cho Trên tia Ax lấy điểm I (với I khác A);

đường thẳng vng góc với CI C cắt tia By K Đường trịn đường kính IC cắt tia IK E

1 Chứng minh tứ giác CEKB nội tiếp đường tròn Chứng minh AI BK =AC CB

3 Chứng minh điểm E nằm nửa đường trịn đường kính AB

4 Cho điểm A; B; I cố định Hãy xác định vị trí điểm C cho diện tích hình thang ABKI lớn

Bài tập (Tỉnh Hải Dương 1998 – 1999)Cho tam giác ABC vuông cân A, trêncạnh BC lấy điểm M Gọi (O1) đường tròn tâm O1 qua M tiếp xúc với AB B, gọi (O2) đường tròn tâm O2 qua M tiếp xúc với AC C Đường trịn (O1) (O2) cắt D (D khơng trùng với A)

1) Chứng minh tam giác BCD tam giác vuông 2) Chứng minh O1D tiếp tuyến (O2)

3) BO1 cắt CO2 E Chứng minh điểm A, B, D, E, C nằm đường tròn 4) Xác định vị trí M để O1O2 ngắn

Bài tập 3.(Đề Hải Dương 1999 – 2000)Cho tam giác ABC, cạnh BC lấy điểm E, qua E

kẻ đường thẳng song song với AB AC chúng cắt AC P cắt AB Q

1) Chứng minh BP = CQ

2) Chứng minh tứ giác ACEQ tứ giác nội tiếp Xác định vị trí E cạnh BC để đoạn PQ ngắn

(147)

Bài tập (Đề Hải Dương 2003-2004)Cho hình vng ABCD, M điểm đường chéo BD, gọi H, I K hình chiếu vng góc M AB, BC AD

1) Chứng minh:

MIC =

HMK

2) Chứng minh CM vng góc với HK

3) Xác định vị trí M để diện tích tam giác CHK đạt giá trị nhỏ

Bài tập 5.(Đề Hải Dương 2005 - 2006)Cho nửa đường tròn đường kính MN Lấy điểm P tuỳ ý nửa đường trịn (P ≠ M, P ≠ N) Dựng hình bình hành MNQP Từ P kẻ PI vng góc với đường thẳng MQ I từ N kẻ NK vng góc với đường thẳng MQ K

1) Chứng minh điểm P, Q, N, I nằm đường tròn

2) Chứng minh: MP PK = NK PQ

3) Tìm vị trí P nửa đường tròn cho NK.MQ lớn

Bài tập 6.(Đề Hải Dương 2006 - 2007)Cho điểm A ngồi đường trịn tâm O Kẻ hai tiếp tuyến AB, AC với đường tròn (B, C tiếp điểm) M điểm cung nhỏ BC (M≠B, M≠C) Gọi D, E, F tương ứng hình chiếu vng góc M đường thẳng AB, AC, BC; H giao điểm MB DF; K giao điểm MC EF

1) Chứng minh:

a) MECF tứ giác nội tiếp b) MF vng góc với HK

2) Tìm vị trí điểm M cung nhỏ BC để tích MDME lớn

Bài tập 7.(ĐềHà Nội năm 2006 - 2007)Cho đường trịn tâm 0, đường kính AB = 2R, C trung điểm OA, kẻ dây cung MN vuông góc với OA C Lấy điểm K tuỳ ý thuộc cung BM nhỏ Gọi H giao điểm AK MN

a) Chứng minh tứ giác BCHK nội tiếp

b) Tính AH AK theo R

c) Xác định vị trí điểm K để tổng KM + KN + KB đạt giá trị lớn nhất, tính giá trị lớn

Bài tập 8.Cho tam giác ABC nội tiếp (O; R) M di động AB N di động tia đối tia CA cho BM = CN

a) Đường tròn ngoại tiếp tam giác AMN cắt (O) A D Chứng minh D cốđịnh b) Tính góc MDN

c) MN cắt BC K Chứng minh DK vng góc với MN d) Đặt AM = x Tính x để diện tích tam giác AMN lớn

Bài tập 9.Hai đường tròn tâm O tâm I cắt hai điểm A B Đường thẳng d qua A

cắt đường tròn (O) (I) P, Q Gọi C giao điểm hai đường thẳng PO QI

a) Chứng minh tứ giác BCQP, OBCI nội tiếp

b) Gọi E, F trung điểm AP, AQ, K trung điểm EF Khi đường thẳng d quay quanh A K chuyển động đường nào?

(148)

Bài tập 10.Cho tam giác ABC nhọn nội tiếp (O), H trực tâm tam giác ABC, M điểm cung BC không chứa điểm A

a Xác định vị trí M để tứ giác BHCM hình bình hành

b Gọi N E điểm đối xứng M qua AB AC Chứng minh ba điểm

N H, E thẳng hàng

c Xác định vị trí M để NE có độdài lớn

Bài tập 11.Cho (O) điểm A nằm (O) Từ A kẻ hai tiếp tuyến AB, AC cát tuyến AMN với (O) (B, C, M, N thuộc (O); AM<AN) Gọi E trung điểm dây MN, I giao điểm thứ hai đường thẳng CE với (O)

a Chứng minh bốnđiểm A, O, E, C nằm đường tròn b Chứng minh góc AOC=góc BIC

c Chứng minh BI//MN

d Xác định ví trí cát tuyến AMN để diện tích tam giác AIN lớn

Bài tập 12.Cho đường trịn (O) đường kính AB=2R điểm M di chuyển nửa đường tròn Người ta vẽ đường tròn tâm E tiếp xúc với (O) M tiếp xúc với AB N Đường tròn cắt MA, MB điểm thứ hai C, D

a Chứng minh CD//AB

b Chứng minh MN tia phân giác góc AMB đường thẳng MN qua điểm K cố định

c Chứng minh tích KM.KN cố định

d Gọi giao điểm tia CN, DN với KB, KA C', D' Tìm vị trí M để chu vi tam giác NC'D' đạt giá trị nhỏ

Bài tập 13.Cho đường trịn (O) điểm A ởngồi đường trịn.Từ A vẽ tiếp tuyến AB,AC với

đường tròn (O)

1 C/m OA vng góc BC

2 Vẽ cát tuyến AMN (O).Gọi E trung điểm MN.C/m A,O,E,C thuộc

đương tròn xác định tâm K Tia CE cắt (O) I.C/m BI//MN

4 Tìm vị trí cát tuyến AMN để diện tích tam giác AIN lớn

Bài tập 14.Cho đường tròn (O; R), Với kí hiệu có hình chứng minh: a)Tứ giác CAIM, BDMI nội tiếp

b)Tam giác CID vuông c)EF // AB

d)Khi M cố đinh I thay đổi AO, tìm vị trí I để ACBD lớn

e) Cho biết OI =

R và AM = R Hãy tính độ dài đoạn thẳng CD diện tích tam

giác CID theo R

Bài tập 15.Cho đường trịn (O) điểm A nằm ngồi đường trịn.Từ A vẽ tiếp tuyến AB cát tuyến ACD (nằm giũa A D )

(149)

2) Gọi H trung điểm CD Chứng minh tứ giác ABOE có bốn điểm thuộc đường trịn

3) Vẽ tia Bx // CD cắt (O) I, IE cắt (O) K.Chứng minh AK tiếp tuyến (O) 4) Đường thẳng BH cắt (O) F.Chứng minh KF // CD

5) Tím vị trí cát tuyến ACD đề diện tích tam giác AID lớn

Bài tập 16.Cho đường tròn ( O,R ), đường thẳng d khơng qua O cắt đường trịn hai điểm A

và B Từ điểm C d (C nằm ngồi đường trịn), kẻ hai tiếp tuyến CM CN ( M N thuộc (O) ) Goi H trung điểm AB,đường thẳng OH cắt tia CN K Đoạn thẳng CO cắt (O) I Chứng minh:

1) C,O,H,N thuộc đường tròn

2) KN.KC= KH.KO

3) I cách CM, CN, MN

4) Một đường thẳng qua O song song MN cắt tia CM CN E F.Xác định vị trí C d để diện tích tam giác CEF nhỏ

Bài tập 17. Cho tam giác ABC vng A có M trung điểm BC Có hai đường thẳng lưu động vng góc với M cắt đoạn AB AC D E Xác định vị trí D E để diện tích tam giác DME đạt giá trị nhỏ

Bài tập 18.Cho nửa đường trịn tâm O đường kính AB điểm C thuộc đoạn AB, M điểm nửa đường trịn Đường thẳng qua M vng góc MC cắt tiếp tuyến qua

A B nửa đường tròn E F

1) Khi M cố định,C di động.Tìm vị trí C để AE.BF lớn 2) Khi C cố định,M di động.Tìm vị trí M để SCEF lớn

Bài tập 19.Cho nửa đường tròn tâm O đường kính AB=2R,M điểm nửa đường trịn(khác A B).Tiếp tuyến (O) M cắt tiếp tuyến A B nửa đường tròn (O) C D

1)Tìm giá trị nhỏ của:

a)Độ dài đoạn thẳng CD diện tích tam giác COD

b) Diện tích chu vi tứ giác ACDB

c)Tổng diện tích tam giác ACM BDM 2) Tìm giá trị lớn của:

a) Diện tích chu vi tam giác MAB b) Tích MAMB

Bài tập 20.(Đề thi tuyển vào lớp 10, 95 - 96 Thành phố Hồ Chí Minh)Cho hình vng ABCD cố định cạnh a Điểm E dichuyển cạnh CD ( E≠ D ) Đường thẳng AE cắt đường thẳng BC F, đường thẳng vng góc với AE A cắt đường thẳng CD K 1) Chứng minh ABF = ADK,suy AKF vuông cân

2) Gọi I trung điểm FK.Chứng minh làtâm đường tròn qua A,C, F,K I di

chuyển đường thẳng cố định E di động CD

3) Chứng minh tứ giác ABFI nội tiếp

(150)

5) Hãy vị trí E để EK ngắn

Bài tập 21.Cho hai đường tròn (O; R ) (O; R’) cắt A B Một đường thẳng (d) quay quanh A cắt (O) (O’) C D

1) Chứng minh đường trung trực đoạn thẳng CD qua điểm cố định Xác địmh điểm cố định

2) Với vị trí đường thẳng (d) tam giác BCD có diện tích lớn

Bài tập 22.Cho tam giác ABC vuông A (AB < AC).Lấy điểm D thuộc cạnh AC Vẽ đường trịn đường kính CD cắt BD E cắt AE F

a) Chứng minh A, B, C, E thuộc đường tròn b) Chứng minh BĈA = AĈ F

c) Gọi M, N điểm đối xứng D qua AB BC Chứng minh tứ giác BNCM nội tiếp

(151)

CD13: GÓC VỚI ĐƯỜNG TRÒN

KIẾN THỨC CƠ BẢN

- Góc ABE có đỉnh A nằm đường trịn

 

O cạnh cắt đường trịn gọi góc

nội tiếp (Hình) Trong trường hợp góc nội tiếp có sốđo khơng vượt q 900 sốđo của

chúng nửa sốđo góc tâm, chắn cung Các góc nội tiếp có sốđo nửa sốđo cung bị chắn Vì thế, góc chắn cung (hoặc chắn cung nhau) chúng nhau, góc nội tiếp cung bị chắn

Trên hình vẽ ta có:    1sđ

2

ABEADEADEAE

- Cho đường tròn

 

O dây cung AB Từđiểm A ta kẻ tiếp tuyến Ax với đường trịn,

BAx gọi góc tạo tia tiếp tuyến với dây cung AB (Hình) Cũng góc nội tiếp, số

đo góc tia tiếp tuyến dây cung nửa sốđo cung bị chắn : sđ 1sđ

2

BAxAmB

Chú ý: Việc nắm khái niệm, định lý, hệ góc nội tiếp, góc tạo tia tiếp tuyến

dây cung giúp so sánh sốđo góc, từđó chứng minh đường thẳng song song với nhau, tam giác nhau, tam giác đồng dạng với nhau…

I Góc nội tiếp đường trịn

(152)

- Hai góc chắn cung nửa sốđo cung bị chắn Trên hình vẽ:

  

đ đ đ

s s s

2

ABDACDAD - Các góc chắn hai cung Trên hình vẽ:

   

đ đ đ đ

s s s s

ADCDADCDABDCAD

B VÍ DỤ Ví dụ Trên cạnh huyền BC tam giác vng ABC phía ngồi ta dựng hình vng

với tâm điểm O Chứng minh AO tia phân giác góc BAC

Lời giải:

O tâm hình vng nên BOC 900

Lại có BAC 900 suy bốn điểm A B O C, , ,

cùng nằm đường tròn đường kính BC

Đối với đường trịn ta thấy BAO BCO (cùng chắn BO) Mà BCO 450 BAO 450

Do BAC 900, nên CAO BACBAO 450 Vậy BAO CAO, nghĩa AO tia phân

giác góc vng BAC(đpcm)

Ví dụ Cho tam giác nhọn ABC nội tiếp đường tròn

 

O Từđỉnh A ta kẻđường cao AH (H thuộc BC ) Chứng minh BAH OAC

Lời giải:

O

D

C B

(153)

Kẻđường kính AE đường trịn

 

O Ta thấy ACE 900 (góc nội tiếp chắn nửa đường

trịn) Từđó OACAEC 900 (1)

Theo giả thiết ra, ta có: BAHABC 900 (2) Lại AEC ABC (cùng chắn AC) (3)

Từ (1),(2) (3) suy BAH OAC (đpcm)

Lưu ý: Cũng giải tốn theo hướng sau: Gọi D giao điểm tia AH với đường

tròn

 

O , chứng tỏ tứ giác BDEC hình thang cân Từđó suy sđBD sđCE, dẫn đến

 

BADCAE, hay BAH OAC

Ví dụ Cho tam giác đềuABC nội tiếp đường trịn

 

O Trên cung BC khơng chứa A ta lấy điểm P (P khác B P khác C ) Các đoạn PA BC cắt

Q

a) Giả sử D điểm đoạn PA cho PDPB Chứng minh PDB

đều

b) Chứng minh PAPBPC

c) Chứng minh hệ thức 1

PQPBPC

(154)

a) Trước tiên ta nhận thấy tam giác PBD cân P Mặt khác, BPD BPA BCA 600

(hai góc nội tiếp chắn AB đường tròn

 

O ) Vậy nên tam giác PDB

b) Ta có PBPD, để chứng minh PAPBPC ta chứng minh DAPC Thật vậy, xét hai tam giác BPC BDA có: BABC (giả thiết), BDBP (do tam giác BPD

đều) Lại ABD DBC 600, PBCDBC 600 nên ABD PBC Từđó BPC  BDA

(c.g.c), dẫn đến DAPC (đpcm)

c) Xét hai tam giác PBQ PAC ta thấy BPQ 600, APC ABC 600 (hai góc nội tiếp

cùng chắn cung AC) suy BPQ APC PBQ , PBC PAC (hai góc nội tiếp chắn 

PC ) Từđó PBQ PAC (g.g) PQ PC

PB PA

  , hay PQ PAPB PC Theo kết câu b,

ta có PAPBPC nên PQ PB

PC

PB PC Hệ thức tương đương với

1 1

PQPBPC (đpcm)

Ghi chú:

- Tứ giác ABCD có tính chất AB CDBC AD (*) nói ví dụtrên gọi tứgiác điều hòa Loại tứgiác đặc biệt có nhiều ứng dụng việc giải tốn hình học phẳng khác

- Nếu hệ thức (*) dạng AB BC

ADCD nhớ lại tính chất đường phân giác tam giác ta

có thể nêu thêm tính chất tứgiác điều hòa

- Tứ giác ABCD tứgiác điều hòa chỉkhi đường phân giác góc BAD 

BCD cắt điểm đường chéo BD

- Tứ giác ABCD tứgiác điều hòa chỉkhi đường phân giác gócABCADC cắt

nhau đường chéo AC

Ví dụ 4) Cho tam giác ABC nội tiếp đường tròn ( )O Đường

phân giác góc A cắt đường trịn ngoại tiếp tam giác D Gọi I tâm vòng tròn nội tiếp tam giác ABC Chứng minh

DBDCDI

Giải:

Ta ln có DBDC AD phân giác góc A Ta chứng minh tam giác DIB cân D

(155)

(Góc nội tiếp chắn cung CD) mà

 

BADCAD , IBC IBA (Tính chất phân giác) suy

  

IBDABIBAI Nhưng

  

BIDABIBAI (Tính chất góc ngồi) Như tam giác BDI

cân DDBDIDC

Nhận xét: Thơng qua tốn ta có thêm tính chất: Tâm

đường trịn ngoại tiếp tam giác IBC giao điểm phân giác góc A với ( )O

Ví dụ 5) Cho tam giác nhọn ABC nội tiếp đường tròn ( )O

ABAC Lấy điểm M thuộc cung BC không chứa điểm A Vẽ

, ,

MH MK MI vng góc với BC AC AB

MHMKMI

Giải:

Trong tốn có tỷ sốđộ dài ta nghỉđến tam giác đồng dạng

và định lý Thales

Cách 1: Dựng đường thẳng qua A

song song với BC cắt ( )O N Gọi E giao điểm BC MN

Ta có: ABNC

Ta có   đ   đ   

2

BMEBMNs AB AN s NC ANAMC ,

 

MBCMAC  BME AMC MH MK, hai đường cao tương ứng nên: AC BE

MKMH ,

chứng minh tương tựta có: AB CE

MIMH Cộng hai đẳng thức ta có:

BC AC AB

MHMKMI

Cách 2: Ta thấy MH MI, đường cao tam giác MBC MAB, hai tam giác

không đồng dạng với Điều giúp ta nghỉđến việc lấy điểm E cạnh BC

cho BMA DMC để tạo tam giác đồng dạng giữđược hai đường cao tương ứng (Phần lời giải xin dành cho bạn đọc)

O I

D

C B

(156)

2 Góc tạo tia tiếp tuyến dây cung

A PHƯƠNG PHÁP GIẢI

- Sốđo góc tạo tia tiếp tuyến dây cung (tại điểm đường tròn) nửa sốđo

cung bị chắn

- Trên hình vẽ: sđ sđ 1sđ

2

BACxBCBC

B VÍ DỤ

Ví dụ Giả sử A B hai điểm phân biệt đường tròn

 

O Các tiếp tuyến

đường tròn

 

O Các tiếp tuyến đường tròn

 

O A B cắt điểm M Từ

A kẻđường thẳng song song với MB

cắt đường tròn

 

O C MC cắt đường tròn

 

O E Các tia AE MB cắt

K Chứng minh MK2 AK EK. MKKB

Lời giải:

Do MB/ /AC nên

 

BMCACM (1), ta lại có

  

ACMACEMAE

(cùng chắn AE) (2) Từ (1) (2)

suy KME KAM (g.g) MK EK

AK MK

  hay MK2 AK EK. (3) Ta thấy EAB EBK

(cùng chắn BE) Từđó EBK BAK (g.g) BK EK

AK BK

  hay BK2 AK EK. (4) Từ (3)

và (4) suy MK2 KB2 nghĩa MKMB (đpcm).

Ví dụ Cho đường tròn

 

C tâm O, AB dây cung

 

C không qua O I

(157)

bán kính OI P Q Chứng minh tích AP AQ khơng đổi đường trịn ngoại

tiếp tam giác BPQ ln qua điểm cốđịnh khác B

Lời giải: Ta có

 

PQIPIA (cùng chắn PI), nên API AIQ (g.g) Suy AP AI AP AQ. AI2 AIAQ  

(khơng đổi) Giả sửđường trịn ngoại tiếp tam giácBPQcắt AB D

DB

Khi ADP AQB, suy

AD AP

AQAB hay

2

AD ABAP AQAI

(khơng đổi) Do điểm D điểm

cốđịnh (đpcm)

Ví dụ Cho tam giác nhọn ABC có trực tâm H BAC 600 Gọi M N P, , theo thứ

tựlà chân đường cao kẻ từ A B C, , tam giác ABC I trung điểm BC a) Chứng minh tam giác INP

b) Gọi E K trung điểm PB NC Chứng minh điểm

, , ,

I M E K thuộc đường tròn

c) Giả sử IA phân giác NIP Tìm sốđo BCP

Lời giải:

a) Từ giả thiết ta có

1

INIPBC nên tam giác

INP cân I Lại B P N C, , ,

nằm đường trịn tâm I , đường kính BC nên theo mối liên hệ góc nội tiếp góc

tâm chắn cung, ta thấy PIN 2PBN 600 Vậy tam giác INP đều.

b) Rõ ràng bốn điểm I M E, , K nằm đường trịn đường kính AI

c) Từđiều kiện toán ta thấy AI tia phân giác BAC 600, mà I là trung điểm

của BC nên tam giác ABC Từđó suy BCP 300

Ví dụ 4) Cho tam giác cân ABC AB,( AC) Gọi O trung điểm BC Dựng đường

(158)

2 4 .

BCBP CQ tìm vịtrí điểm M để diện tích tam giác APQ lớn

Lời giải:

Ta thấy SABC không đổi nên

APQ

S lớn chỉkhi SBPQC

nhỏ nhất, sởđể ta làm xuất biểu thức có liên quan

đến BP CQ, Ta có AB PQ AC, ,

lần lượt tiếp tuyến điểm

, ,

D M E ( )O nên ta có: ABOD PQ, OM AC, OE BD, CE Từđó ta tính được:

1 2 2

2

BPQC

SR BPPQCQR BDDPEQCE

R BD DP EQ R BP CQ BD

      Mặt khác ta có: 1 1

1800 

 

2

POQDOE  ABC nên suy

 1800   1800   

BOP  POQ QOC  QCO QOC CQO  BPO COQ

2

4

BP BO BC BP CQ BO CO

CO CQ

     Theo bất đẳng thức Cơ si ta có:

2

BPCQBP CQBCSBPQCR BC.

BD

Vậy SBPQC nhỏ

BPCQM trung điểm cung DE

Chủđề Góc có đỉnh ngồi đường trịn

KIẾN THỨC CẦN NHỚ

*) Với đỉnh A nằm đường trịn

 

O ta có góc với đỉnh ởtrong đường trịn (hình)

Sốđo góc nửa tổng số

đo hai cung bị chắn hai cạnh góc tia đối hai cạnh

+ sđ sđ sđ

2

BE CD

BAE  

+ đ sđ sđ

2

BD CE

(159)

*) Với đỉnh A nằm ởngồi đường trịn

 

O ta có sốđo góc nằm ngồi đường trịn nửa

hiệu sốđo hai cung bị chắn + Trên hình vẽ ta có:

  

đ đ đ

s s s

2

CAE   EmCBnD Cần lưu ý đến trường hợp sau:

+ Với đỉnh A nằm đường tròn

( )O AD tếp tuyến ( )O , qua A

vẽ cát tuyến cắt đường trịn

,

BC  sđ sđ

2

CAD   CmDBnD

+ Với Với đỉnh A nằm đường tròn

( )O AB AC, tếp tuyến ( )O ,

(A, B tiếp điểm)

 đ đ

s s

BAC   BmCBnC

3 Áp dụng góc có đỉnh ngồi đường trịn

A PHƯƠNG PHÁP GIẢI Cũng phần góc nội tiếp, góc tạo tia tiếp tuyến dây cung, định lý hệ góc có

đỉnh nằm nằm ngồi đường trịn giúp tìm mối quan hệ sốđo

góc, chứng minh đường song song, tam giác nhau, tam giác đồng dạng với

nhau, hai đường thẳng vng góc với

B VÍ DỤ

Ví dụ ) Trên đường tròn

 

O cho điểm A B C D, , , theo thứ tựđó Gọi A B C D1, , ,1 1 1

lần lượt điểm cung AB BC CD, , DA Chứng minh đường

thẳng AC1 1 B D1 1 vng góc với

(160)

Gọi I giao điểm AC1 1 B D1 1;    , , , theo thứ tự sốđo cung    , , ,

AB BC CD DA Khi     3600

Xét góc A IB1 1 góc có đỉnh nằm

trong đường trịn

 

O Ta có 1 1 sđ1 1 sđ1 1

A IB   A BBC DD 

   

đ 1 đ 1 đ 1 đ 1

1 s s s s A B BB C D DD

 

      1

900

4

     Nghĩa

1 1

ACB D (đpcm)

Ví dụ Cho bốn điểm A D C B, , , theo thứ tựđó nằm đường trịn tâm O đường

kính AB 2R (C D nằm phía so với AB) Gọi E F theo thứ tự hình chiếu vng góc A B, đường thẳng CD Tia AD cắt tia BC I Biết

rằng AEBFR a) Tính sốđo AIB

b) Trên cung nhỏCD lấy điểm K Gọi giao điểm KA KB, với DC M N Tìm giá trị lớn MN K di động cung nhỏCD

Lời giải:

a) KẻOHCD H

CD

,

ta thấy OH đường trung bình

của hình thang ABFE,

suy 1

2

R

OHAEBF

I

D1

C1 B1 A1

O

D C B

(161)

Từđó tam giác OCD đều,

suy sđCOD sđKCD 600.Ta thấy AIB có đỉnh nằm ngồi đường tròn

 

O nên

  

đ đ đ 0

s s s 180 60 60

2

AIB   AmBKCD  

b) Ta thấy AEM NFB suy EM NFAE BF (khơng đổi) MN lớn

và chỉkhi EMNF nhỏ Theo trên, EM NF không đổi nên EMNF nhỏ

EMFNAE BF Vậy giá trị lớn MN EF 2 AE BF

Ví dụ Trong tam giác ABC, đường phân giác BAC cắt cạnh BC D Giả sử

 

T đường tròn tiếp xúc với BC D qua điểm A Gọi M giao điểm thứ hai

của

 

T AC , P giao điểm thứ hai

 

T BM, E giao điểm AP BC

a) Chứng minh EAB MBC b) Chứng minh hệ thức BE2 EP EA.

Lời giải:

a) Gọi N giao điểm thứ hai

của AB với đường tròn

 

T

Do AD phân giác BAC

nên sđDM  sđDN Ta có   sđ sđ sđ sđ

2

MBCMBD   DMDP  DNDP

  

đ

1s

2 NP NAP EAB

   (đpcm)

b) Từkết câu a, ta thấy EBP EAB Từđó EBP EAB (g.g), suy BE EA

EPBE hay

2 .

(162)

Ví dụ Trên đường tròn

 

O ta lấy điểm A C B A C B, , , , ,1 1 1 theo thứ tựđó

a) Chứng minh đường thẳng AA BB CC1, 1, 1 đường phân giác

tam giác ABC chúng đường cao A B C1 1

b) CHứng minh đường thẳng AA BB CC1, 1, 1 đường cao tam giác

ABC chúng đường phân giác tam giác A B C1 1

c) Giả sử

 

T1

 

T2 hai tam giác nội tiếp đường tròn

 

O , đồng thời đỉnh tam

giác

 

T2 điểm cung đường trịn bị chia đỉnh tam giác

 

T1 Chứng minh hình lục giác giao tam giác

 

T1

 

T2

đường chéo nối đỉnh đối song song với cạnh tam giác

 

T1 đồng quy

tại điểm

Lời giải:

a) Ta chứng minh AA1 B C1 1 Thật vậy, gọi M giao điểm AA1 B C1 1, đó:

 đ đ đ đ đ

1 1 1 1

1 s s s s s

2

AMB   ABA BC   ABA BBC 

  

  

1 1

1 90

2

ABB A AB BCC ABC CAB BCA

       (đpcm)

(163)

Gọi M1 giao điểm BB1 AC Ta có 1 sđ1 sđ1  1 1

BM A  AC BACBCAAC C (1)

Lại có 2 sđ1 1  1 1

2

BM A  AC BB CBCAB C C (2) Vì   90

BM ABM A , nên từ (1)

(2) suy AC A1 1 B C C1 1 Tức CC1 chứa đường phân giác AC B1 1

Chứng minh tương tự, ta thu AA1 chứa đường phân giác B AC1 1, BB1 chứa

đường phân giác A B C1 1

c) Kí hiệu đỉnh tam giác

 

T1 A B, C ; A B1, 1 C1 điểm cung  ,

BC CA AB tương ứng Khi

 

T2 tam giác A B C1 1 Các đường AA BB CC1, 1, 1 chứa đường phân giác tam giác

 

T1 nên chúng đồng quy điểm I Giả sử K giao điểm

AB B C1 1 Ta cần chứng minh IK/ /AC

Thật vậy, ta thấy tam giác AB I1 cân B1 nên tam giác AKI cân K Từđó

  

KIAKAIIAC , dẫn đến IK / /AC (đpcm)

Dạng Áp dụng giải toán quỹ tích dựng hình

A PHƯƠNG PHÁP GIẢI

Khái niệm cung chứa góc giúp giải nhiều tốn quỹ tích, dựng hình, chứng minh nhiều điểm thuộc đường trịn

B VÍ DỤ

Ví dụ Cho tam giác cân ABC

ABAC

D điểm cạnh BC Kẻ

/ /

DM AB (MAC ), DN / /AC N

AB

Gọi D' điểm đối xứng D qua MN

Tìm quỹtích điểm D' điểm D di động cạnh BC

(164)

Phần thuận: Từ giả thiết đề ta thấy NBNDND',(1) ba điểm B D D, , ' nằm

đường trịn tâm N Từđó ' 

2

BD DDMC (2) Lại có BND DMC BAC, nên từ (1) (2) suy BD C' BAC(khơng đổi) Vì BC cốđịnh, D' nhìn BC góc BAC khơng đổi,

'

D khác phía với D (tức phía với A so với MN ) nên D' nằm cung chứa góc BAC

vẽtrên đoạn BC (một phần đường tròn ngoại tiếp tam giác ABC)

Phần đảo: Bạn đọc tự giải

Kết luận: Quỹ tích điểm D' cung chứa góc BAC đoạn BC Đó cung BAC

của đường trịn ngoại tiếp tam giác ABC

Lưu ý: Quy trình để giải tốn quỹtích sau:

Để tìm quỹtích điểm M thỏa mãn tính chất

 

T ta tiến hành bước

*Phần thuận: Chỉ điểm có tính chất

 

T thuộc hình

 

H

*Phần đảo: Chứng tỏ điểm thuộc hình

 

H có tính chất

 

T

*Kết luận: Quỹtích điểm M có tính chất

 

T hình

 

H

Chú ý số tốn, sau phần thuận, trước phần đảo ta thêm phần giới hạn quỹ tích

(Bạn đọc tham khảo thêm phần quỹ tích cuối sách này)

Ví dụ Cho đường trịn

 

O dây cung BC cốđịnh Gọi A điểm di động cung

lớn BC đường tròn

 

O (A khác B, A khác C ) Tia phân giác ACB cắt đường

tròn

 

O điểmD khác điểm C Lấy điểm I thuộc đoạn CDsao cho DIDB Đường

thẳng BI cắt đường tròn

 

O điểm K khác điểm B

a) Chứng minh tam giác KAC cân

b) Chứng minh đường thẳng AI qua điểm J cốđịnh

c) Trên tia đối tia AB lấy điểm M cho AMAC Tìm quỹtích điểm M A di động cung lớn BC đường tròn

 

O

(165)

a) Ta có

 sđ sđ ;

2

DBK   DAAK sđ sđ sđ

2

DIB   BDKC Vì sđBD sđDA DBI cân D

nên sđKCsđAK Suy AKCK hay KAC cân K (đpcm)

b) Từkết câu a, ta thấy I tâm đường tròn nội tiếp ABC nên đường thẳng AI

qua điểm J (điểm cung BC khơng chứa A) Rõ ràng J điểm cốđịnh

c) Phần thuận: Do AMC cân A, nên  1

2

BMCBAC Giả sử sốđo BAC 2 (không đổi)

thì A di động cung lớn BC M thuộc cung chứa góc dựng đoạn BC phía

điểm O

Phần đảo: Tiếp tuyến Bx với đường tròn

 

O cắt cung chứa góc vẽtrên đoạn BC điểm X Lấy điểm M Cx (một phần cung chứa góc và vẽtrên đoạn

;

BC MX MC Nếu MBcắt đường tròn

 

O A rõ ràng A thuộc cung lớn BC

đường trịn

 

OBAC 2 ; AMC suy AMC cân A hay ACAM

Kết luận: Quỹtích điểm M cung Cx, phần cung chứa góc vẽtrên đoạn BC

về phía O trừhai điểm C X

Ví dụ Cho trước điểm A nằm đường thẳng d hai điểm C D, thuộc hai nủa

mặt phẳng đối bờ d Hãy dựng điểm B d cho ACB ADB

Lời giải:

O x

J

K M

D

C B

(166)

*Phân tích: Giả sử dựng điểm B d cho ACB ADB Gọi D' điểm đối xứng

của D qua d Khi ADB AD B' , ACB AD B' Suy C D' nằm

nửa cung chứa góc dựng đoạn AB Từđó ta thấy B giao điểm d với đường tròn

ngoại tiếp ACD'

*Cách dựng: Dựng điểm D' điểm đối xứng D qua đường thẳng d Dựng đường

tròn ngoại tiếp tam giác ACD'

Dựng giao điểm B đường thẳng d với đường tròn

ACD'

*Chứng minh: Rõ ràng với cách dựng trên, ta có ACB AD B' ADB

*Biện luận: Nếu ba điểm A C D, , không thẳng hàng, ba điểm thẳng hàng

CD khơng vng góc với d tốn có nghiệm hình

+ Nếu ba điểm A C D, , thẳng hàng d đường trung trực đoạn CD tốn có vơ số

nghiệm hình

+ Nếu ba điểm A C D, , thẳng hàng, dCD d đường trung trực CD

thì tốn khơng có nghiệm hình

Lưu ý: Khái niệm cung chứa góc áp dụng để chứng minh nhiều điểm thuộc

đường trịn Ví dụđể chứng minh bốn điểm A B C D, , , nằm đường trịn, ta

chứng minh hai điểm A B nhìn CD hai góc Nói cách khác,

tứgiác có hai đỉnh kề nhìn cạnh chứa hai đỉnh cịn lại hai góc bốn đỉnh tứgiác thuộc đường tròn

d D' B

D C

(167)

Ví dụ Giả sử AD đường phân giác góc A tam giácABC (DBC ) Trên

AD lấy hai điểm M N cho ABN CBMBM cắt đường tròn ngoại tiếp tam

giác ACM điểm thứ hai E CN cắt đường tròn ngoại tiếp tam giác ABM điểm

thứ hai F

a) Chứng minh bốn điểm B C E F, , , nằm đường tròn

b) Chứng minh ba điểm A E F, , thẳng hàng

c) Chứng minh BCF ACM, từđó suy ACN BCM

Lời giải:

a) Ta có BFC BAN (cùng chắn cung BN); BEC CAN (cùng chắn CM), mà BAN CAN,

suy BFC BEC

Từđó bốn điểm B C E F, , , nằm đường trịn (đpcm)

b) Từkết trên, ta có CFE NFA Do hai tia FA FE trùng nghĩa ba điểm

, ,

A E F thẳng hàng (đpcm)

c) Vì BCF BEFACM BEF nên BEF ACM Từđó suy ACM BCF, dẫn đến

 

ACNBCM (đpcm)

BÀI TẬP VỀNHÀ

Câu Cho đường tròn

O R;

, R 4cm vẽ dây cung AB 5cm, C điểm dây cung AB

sao cho AC 2cm VẽCD vng góc với OA D Tính độdài đoạn thẳng AD

Câu Cho đường tròn

O R;

, AC BD hai đường kính Xác định vị trí hai đường

kính AC BD để diện tích tứ giác ABCD lớn

Câu Cho đường tròn ( ; )O R từđiểm M bên ngồi đường trịn ta kẻhai đường thẳng

(168)

Câu Cho đường trịn

O R;

đường kính AB CD, dây cung

 

O , COD 900, CD cắt

AB M (D nằm C M) OM 2R Tính độdài đoạn thẳng MD MC, theo R

Câu Cho điểm C nằm hai điểm A B Gọi

 

O đường tròn bất kỳđi qua AB

Qua C vẽđường thẳng vng góc với OA, cắt đường trịn

 

O D E Chứng minh

các độ dài AD AE, không đổi

Câu Cho đường trịn

O R;

, hai bán kính OA OB vng góc O C D điểm

trên cung AB cho ACBD hai dây AC BD, cắt M Chứng minh

OMAB

Câu Cho điểm A ởngoài đường tròn

O R;

Vẽ cát tuyến ABC tiếp tuyến AM với

đường tròn

 

O M tiếp điểm Chứng minh ABAC 2AM

Câu Cho đoạn thẳng AB, đường thẳng d d' vng góc với AB A B M

là trung điểm AB Lấy C D, d d, ' cho CMD 900 Chứng minh rằng CD

là tiếp tuyến dường trịn đường kính AB

Câu Từđiểm P nằm ngồi đường trịn

O R;

vẽ hai tiếp tuyến PA PB tới đường tròn

O R;

với A B tiếp điểm Gọi H chân đường vng góc vẽ từ A đến đường kính BC đường trịn Chứng minh PC cắt AH trung điểm I AH

Câu 10 Một đường tròn nội tiếp tam giác ABC tiếp xúc với AB AC, D E, Cho

điểm M thuộc đoạn thẳng AD; CM cắt DE I Chứng minh IM DM

ICCE

Câu 11 Cho đường tròn

 

O r; nội tiếp tam giác ABC tiếp xúc với BC D Vẽđường kính

;

DE AE cắt BC M Chứng minh BDCM

Câu 12 Cho tam giác ABC Một đường tròn tâm Onội tiếp tam giác ABC tiếp xúc với BC

tại D Đường tròn tâm I đường trịn bàng tiếp góc A tam giác ABC tiếp xúc

với BC F Vẽđường kính DE đường trịn

 

O Chứng minh A E F, , thẳng hàng

Câu 13 Đường tròn tâm I nội tiếp tam giác ABC tiếp xúc với BC AB AC, , D E F, ,

Đường thẳng qua E song song với BC cắt AD DF, M N, Chứng minh M

trung điểm đoạn thẳng EN

Câu 14 Cho tam giác nhọn ABC Gọi O trung điểm BC Dựng đường trịn tâm O

đường kính BC Vẽđường cao AD tam giác ABC tiếp tuyến AM AN, với đường

tròn

 

O (M N, tiếp điểm) Gọi E giao điểm MN với AD Hãy chứng minh

2

(169)

HƯỚNG DẪN BTVN

Câu Giải:

Vẽđường kính AEAE 8cm

Điểm B thuộc đường trịn

đường kính AEABE 900

Xét ADCABEDAC

(chung), ADC ABE

900

,

do ADC ABE AD AC AD AC AB

AB AE AE

    Mà AC 2cm AB, 5cm AE, 8cm,

nên 2.5 5

 

8

AD   cm

Câu

Giải:Vẽ AHBD H

BD

Tứ giác ABCD

,

OAOAR OBODR

nên hình bình hành Mà

2

ACBDR tứ giác

ABCD hình chữ nhật, suy

ABCD

SAB ADABDA 900, AHDB nên AB AD. AH DB.

AHAO DB, 2R nên 2

ABCD

SR (không đổi) Dấu “=” xảy HOACBD Vậy hai đường kính AC BD vng góc với diện tích tứ giác ABCD lớn

Câu Giải:

VẽOHAB H

AB

, OKCD K

CD

Ta có ABCD (gt), nên

OHOK (định lý liên hệdây cung khoảng

(170)

lần lượt trung điểm

,

AB CD (định lý đường kính

vng góc dây cung) AHCK Xét OHM

OHM 900

OM (cạnh chung)

OHOK, OHM  OKM (cạnh huyền, cạnh góc vng) MHMK Ta có

MHAHMK CK MAMC

Câu Giải:

COD 900 suy tam giác

COD vuông cân O nên

2

CDR Gọi H trung điểm CD Vì HOM vuông H ,

1 , 2

2

OHCDR OMR Trong tam giác vng OMH ta có:

2

2 2 4 14

2 2

R R

MHOMOHR   MHR suy

2 7 1

2

R

MDMHAH   , 2

1

R

MC  

Câu

Gọi H giao điểm OA DE

Ta có OADEADAE Chỉ cần chứng minh AD AE có độ dài

khơng đổi Các đoạn thẳng AB AC,

có độdài khơng đổi, DEOA từđó

gợi cho ta vẽđường phụlà đường kính AF để suy ra: AD2 AH AF AC AB. , . AH AF.

Câu Giải:

OAB

 cân đỉnh O, ACBD, điều giúp ta nghỉđến chứng minh OM đường phân giác

góc OOAB.VẽOIAC ,

(171)

,

OKBD IAC KBD

thì ta có OIOK suy lời giải toán

Câu Giải:

VẽOHBC H, BC, suy BHHC (định lý

đường kính vng góc dây cung)

Ta có ABAC

AHBH

 

AHHC

2AHMAOAMO 900, theo định lý

Pitago có AM2 OM2 OA2; HAOAHO 900 nên AH2 OH2 OA2 mà

OBOMR, OHOB nên OHOM Do OH2 OM2, suy AHAM Từđó ta có:

2

ABACAM

Câu Giải:

Vẽ MHCD H, CD

Gọi N trung điểm CD

thì MN đường trung bình

hình thang tam giác MNC cân

tại N nên NMC ACMMCN

Suy CM tia phân giác ACH nên MAMH , Từđó ta có điều phải chứng minh

Câu Gợi ý:

Dễ thấy PB/ /AH, gọi D giao điểm CA BP tam giác BAD vuông A Do

PAPBPAPBPD (Do PDA DAP phụ với DBA PAB) Áp dụng định lý Thales ta có:

IA IH AH

PDPBBD

(172)

Câu 10 Giải:

Điều cần chứng minh làm ta nghĩ đến định lý Thales ta làm xuất “hai đường thẳng song song”

+ VẽCK / /AB K, DE

Ta có IM DM

ICCK (*)

+ Vì CEK AED ADE EKC

Suy tam giác CEK cân CCECK.Thay vào (*) ta có: IM DM

ICCE

Câu 11 Giải:

Vẽ tiếp tuyến E

đường tròn

 

O cắt AB AC,

tại H K, Ta có

, / /

EDHK EDBCHK BC

Gọi N tiếp điểm đường tròn

 

O tiếp xúc với AC

,

OK OC hai tia phân giác hai góc kề bù EON NOD (tính chất trung tuyến)

 900 KOC

  + Xét OEKCDO

OEC CDO

90 ,0

OKE COD (cùng phụ với EOK).Do

EK OE

OEK CDO

OD CD

    hay EK r

rCD Tương tựcũng có

HE r

rBD Do

EK BD EK BD

HECDEKHEBDCD hay

EK BD

HKBC (1)

+ Trong ABMHE / /BM, áp dụng hệ định lý Thales tam giác ta có

HE AE

BMAM Tương tự có

EK AE

CMAM Do

HE EK EK EK HE

BM CM CM CM BM

  

 hay

EK HK EK CM

CMBCHKBC (2)

Từ (1) (2) cho ta BDCM

(173)

thẳng hàng (vì O I, nằm tia phân góc A)

+ Gọi M N, tiếp điểm

 

O ;

 

I với AB, ta có OM / /IN

nên AO OM

AIIN (hệ định lý Thales).Mà OMOE IN, IF nên có

AO OE AIIF

Mặt khác EDBC IF, BCOD/ /IFAOE AIF + Xét OAE

IAF

 có AO OE;AOEAIF

AIIF  ,

 

OAE IAF OAE IAF

    Vậy A E F, , thẳng

hàng

Câu 13 Giải

+ Vì đường trịn ( )I tiếp xúc với

các cạnh D E F, , nên suy , ,

AEAF BEBD CDCF

+ Dựng AK / /BD K

DF

ta có:

MN MD

AKDA ,

EM AM

BDAD Ta cần

chứng minh: MD.AK AM.BD MD BD

DAADAMAK Nhưng AKAFAE, BDBE nên ta

cần chứng minh: MD BE

AMAE (điều hiển nhiên)

I F K

O

D M

C B

A

H E

(174)

Câu 14 Giải:

,

AM AN tiếp tuyến đường

tròn

 

O ,gọi H giao điểm AO

MN

Ta có tam giác AHE đồng dạng với

Tam giác ADO nên AE ADAH AO

Cũng theo tính chất tiếp tuyến ta có: AH AO. AM2.Từđó suy điều phải chứng minh

-/// -

D O C

B

H

E N

M

(175)

CHĐỀ 3- GĨC VỚI ĐƯỜNG TRỊN, TỨ GIÁC NỘI TIẾP

A.BÀI TẬP MINH HỌA

Câu Cho tứ giác ABCD có đường trịn đường kính AD tiếp xúc với BC đường trịn

đường kính BC tiếp xúc với AD Chứng minh AB/ /CD

Câu Cho tam giác ABC Trên nửa mặt phẳng bờ BC không chứa điểm A vẽ nửa đường trịn đường kính BC , D điểm nủa đường tròn cho sđCD 600 Gọi M là giao điểm AD với BC Chứng minh BM 2MC

Câu Cho đường tròn

O R;

O R'; '

tiếp xúc A

RR'

Tiếp tuyến điểm M

O R'; '

cắt

O R;

B C Chứng minh BAM MAC

Câu Cho tam giác ABC nội tiếp đường tròn

O R;

, AH đường cao

HBC

Chứng minh rằng: AB AC 2 R AH

Câu Cho tam giác ABCA nhọn nội tiếp đường tròn

O R;

Chứng minh rằng: 

2 sin

BCR BAC

Câu Cho hai đường tròn

 

O

 

O' cắt A B Qua A vẽ hai cát tuyến CAD EAF (C E nằm đường tròn

 

O , D F nằm đường tròn

 

O' ) cho

 

CABBAF Chứng minh CDEF

Câu Cho đường trịn

 

O đường kính AB C điểm cung AB (C khác A B ) Vẽ

CHAB HAB Vẽđường tròn

C CH;

cắt đường tròn

 

O D E DE cắt CH

M Chứng minh MHMC

Câu Cho tam giác ABC nội tiếp đường tròn

O R;

Vẽ AD đường cao tam giác ABC Chứng minh BAD OAC

Câu Cho hình bình hành ABCD Đường trịn ngoại tiếp tam giác BCD cắt đường thẳng AC E Chứng minh đường tròn ngoại tiếp tam giác ABE tiếp xúc với BD

Câu 10 Cho đoạn thẳng AB M điểm di động đoạn thẳng AB (M khác A B ) Vẽ đường thẳng xMy vng góc với AB M Trên tia Mx lấy C D cho

,

MCMA MDMB Đường trịn đường kính AC cắt đường trịn đường kính BD N (N khác A) Chứng minh đường thẳng MN luôn qua điểm cốđịnh

Câu 11 Cho tam giác ABC nhọn nội tiếp đường trịn

O R;

có đỉnh A cốđịnh, đỉnh B C, di

động.Dựng hình bình hành ABDC Chứng minh trực tâm H tam giác BDC điểm

(176)

Câu 12 Cho tam giác nhọn ABC Vẽđường trịn

 

O đường kính BC Vẽ AD đường cao tam giác ABC, tiếp tuyến AM AN, với đường tròn

 

O (M N, tiếp điểm) MN cắt AD E Chứng minh E trực tâm tam giác ABC

Câu 13 Cho tam giác nhọn ABC, trực tâm H Từ A vẽ tiếp tuyến AM AN, với đường trịn

 

O đường kính BC (M N, tiếp điểm) Chứng minh M H N, , thẳng hàng

Câu 14 Cho tam giác ABC cân đỉnh A, đường trung trực AB cắt BC D Chứng minh AB tiếp tuyến đường tròn ngoại tiếp tam giác ACD

Câu 15 Cho tam giác ABC

A 900

ABAC Vẽđường trịn tâm A bán kính AB cắt BC D, cắt AC E Chứng minh DB CB. EB2

Câu 16 Cho tam giác vng ABC nội tiếp đường trịn

O R AB;

AC A, 900

Đường tròn

 

I qua B C, tiếp xúc với AB B, cắt đường thẳng AC D Chứng minh OABD Câu 17 Cho đoạn thẳng AB 2a có trung điểm O Trên nửa mặt phẳng bờ AB dựng nửa đường trịn

 

O đường kính AB nửa đường trịn

 

O' đường kính AO Trên

 

O' lấy điểm M (khác A O), tia OM cắt

 

O C , gọi D giao điểm thứ hai CA với

 

O'

a) Chứng minh tam giác ADM cân

b) Tiếp tuyến C

 

O cắt tia OD E, xác định vịtrí tương đối đường thẳng EA

 

O

 

O'

Câu 18 Cho đường trịn tâm O có đường kính AB 2R Gọi M điểm di động đường

tròn

 

O Điểm M khác A B, ; dựng đường tròn tâm M tiếp xúc với AB H Từ A B kẻ hai tiếp tuyến AC BD với đường tròn tâm M vừa dựng

a) Chứng minh BM AM, tia phân giác góc ABDBAC

b) Chứng minh ba điểm C M D, , nằm tiếp tuyến đường tròn tâm O điểm M c) Chứng minh ACBD khơng đổi, từđó tính tích AC BD theo CD

d) Giả sử A B, nửa đường trịn đường kính AB khơng chứa M có điểm N cố định gọi I trung điểm MN , kẻ IP vng góc với MB Khi M chuyển động P chuyển động đường cốđịnh

(177)

a) Chứng minh EKAB

b) Gọi F điểm đối xứng với K qua I Chứng minh AF tiếp tuyến

 

O c) Chứng minh AK AC. BK BI. AB2

d) Nếu sin

3

BAC  Gọi H giao điểm EK AB Chứng minh

KH KHHEHE KE

Câu 20 Cho đường trịn

 

O đường kính AB 2A, điểm C thuộc đường tròn

CA C, B

Trên nửa mặt phẳng bờ AB chứa điểm C , kẻ tia Ax tiếp xúc với đường tròn

 

O Gọi M điểm cung nhỏ AC Tia BC cắt Ax Q, tia AM cắt BC N

a) Chứng minh tam giác BAN MCN cân

b) Khi MBMQ, tính BC theo R

Câu 21 Cho đường trịn

O R;

đường kính AC Trên đoạn thẳng OC lấy điểm B vẽđường trịn

 

O' có đường kính BC Gọi M trung điểm AB, qua M kẻ dây cung vng góc với

ABcắt đường tròn

 

O D E Nối CD cắt đường tròn

 

O' I a) Tứ giác DAEB hình có đặc tính gì? Vì sao?

b) Chứng minh MDMI MI tiếp tuyến đường tròn

 

O'

c) Gọi H hình chiếu vng góc I BC Chứng minh CH MBBH MC

Câu 22 Cho tam giác ABC đều, dựng nửa đường tròn tâm D đường kính BC tiếp xúc với ,

AB AC K L, Lấy điểm P thuộc cung nhỏ KL, dựng tiếp tuyến với nửa đường tròn P cắt cạnh AB AC, M N,

a) Chứng minh BMD CDN suy

4 BC

BM CN

b) Chứng minh

2 MDN

ABC

S MN

SBC

c) Gọi E F, nằm cạnh AB AC, cho chu vi AEF nửa chu vi ABC

 Chứng minh EDF 600

(178)

a) MA AD

MBAB b) AD BCAB CD

c) AB CDAD BCAC BD d) CBD cân

Câu 24 Trên nửa đường trịn tâm

O R;

, đường kính AB lấy hai điểm M E, theo thứ tự , , ,

A M E B Hai đường thẳng AM BE cắt C , AE BM cắt D a) Chứng minh tứ giác MCED nội tiếp CD vng góc với AB

b) Gọi H giao điểm CD AB Chứng minh BE BCBH BA

c) Chứng minh tiếp tuyến M E đường tròn

 

O cắt điểm I thuộc CD

d) Cho BAM 45 ,0 BAE  300 Tính diện tích tam giác ABC theo R

Câu 25 Cho tam giác ABC đều, gọi O trung điểm cạnh BC Các điểm D E, di động cạnh AB AC, cho DOE 600

a) Chứng minh BD CE không đổi,

b) Chứng minh tia DO tia phân giác BDE

c) Dựng đường tròn tâm O tiếp xúc với AB Chứng minh đường trịn ln tiếp xúc với DE AC

d) Gọi P Q, tiếp điểm

 

O với AB AC, I N giao điểm PQ

với OD OE Chứng minh DE 2IN Câu 26 Cho đường tròn

O R;

điểm A ởbên ngồi đường trịn Vẽ hai tiếp tuyến AB AC,

với đường tròn

 

O (B C, tiếp điểm) Gọi M trung điểm AB

a) Chứng minh tứ giác ABOC nội tiếp xác định tâm I đường tròn b) Chứng minh AM AOAB AI

c) Gọi G trọng tâm tam giác ACM Chứng minh MG/ /BC d) Chứng minh IG vng góc với CM

Câu 27 Cho đường tròn

O R;

nội tiếp ABC , tiếp xúc với cạnh AB AC, D vàE a) Gọi O' tâm đường trịn nội tiếp ADE, tính OO' theo R

(179)

c) Chứng minh MN DM EN

BCACAB

B.HƯỚNG DẪN GIẢI

Câu Giải:

Gọi O trung điểm BC

thì tam giác OCD nên OCD 600

/ / AB CD

 Để chứng minh:BM 2MC

Ta cần chứng minh AB 2CD

Xét tam giác vng BDC ta có:

0

.sin 30

CDBCBC suy BCAB 2CD

Câu Giải:

Ta gọi giao điểm AM cung BC D.Ta có BAM MAC BD DC

' / / OD BC O M OD

   AMO' ADO Để chứng minh: AMO'ADO ta

dựa vào tam giác cân O AM' OAD Câu Giải:

Vẽđường kính AD đường

D O

M C

B

A

(180)

tròn

 

O , suy ACD 900

(góc nội tiếp chắn nửa đường trịn) Xét HBACDA có:

 

90 ;0

 

AHBACDHBACDA (góc nội tiếp chắn AC), Do

AH AB

HBA CDA AB AC AD AH

AC AD

      Mà AD 2R Do AB AC 2 R AH

Câu Giải:

Vẽđường kính BD đường trịn

O R;

BCD 900 (góc nội tiếp chắn nửa đường trịn)

BCD

 có C 900 nên BCBDsinBDC Ta lại có BD 2 ;R BDC BAC (góc nội tiếp chắn BC) nên BC 2 sinR BAC

T toán ta cn ghi nh kết qu quan trng: Trong tam giác ABC ta có:

sin sin sin

a b c R

ABC

Câu Giải:

Ta có: AB tia phân giác CAF, Vẽ BHCD BK, EF

Thì suy BHBK

Ta có: CBD$EBF suy

1 CD BH

CD EF

EFBK    Đó điều phải chứng minh Câu Giải:

Dựng đường kính HN đường trịn

 

C cắt đường tròn

 

O K ta có

CNCHHK

(181)

 

MC MK HC MC HC MC

   

2

MC MK HC MC

   MC MC( MK)HC2

Hay MC MC( MK)HC2 MC HC.2 HC2 HC 2MC là điều phải chứng minh Câu Giải:

Dựng đường kínhAE đường

trịn

O R;

.Ta có AEC ABD (cùng chắn

cung AC )

suy DBACEA, từđó suy

 

BADOAC Câu

Ta có: BEC BDC (cùng chắn cung ) BC ABD BDC(so le trong) suy BEC ABD

Vì tia BD tia tiếp tuyến đường tròn ngoại tiếp tam giác ABE Câu Giải:

+ Vẽđường trịn đường kính AB MBD

 vng M có MB MD

(gt) nên tam giác vuông cân  450

ACM

  Từđó ta có

  450

ANMACM  (hai góc nội

tiếp chắn AM)

   900

ANBANMMNB  ; N thuộc đường trịn đường kính AB + Gọi E giao điểm MN AB (E khác N ) Ta có

  450  

ANMMNB  AEEBE cốđịnh Vậy MN qua điểm cốđịnh E

Câu 10 Giải:

(182)

Dựng đường kính AH

 

O Ta chứng minh H trực tâm

BDC

 Thật ta có: ACH 900

CH AC CH BD

    Tương tựta có: BHABBHCD Như H

là trực tâm BDC Suy trực tâm H điểm cốđịnh Câu 11 Giải:

AB cắt

 

O B F Vì AEH$ADO

suy AE AD. AH AO. AM2

Để chứng minh E trực tâm tam giác ABC, ta cần chứng

minh AFE 900, nghĩa cần có AF AB. AE AD.

Nhưng ta có: AF AB. AM2(Tính chất tiếp tuyến, cát tuyến) hoặc có thểdùng tam giác đồng dạng

Câu 12 Giải:

Gọi D E, giao điểm đường tròn

 

O với cạnh AC AB, H giao điểm BD CE,

Chứng minh AMH AMN, từđó có M H N, , thẳng hàng Câu 13 Giải:

Hai tam giác cân ABC DAB,

có chung góc ởđáy ABC,

do BAC ADC Suy BA tiếp tuyến đường tròn ngoại tiếp

tam giác ACD

(183)

Câu 14 Giải:

Vẽ tiếp tuyến Ax đường tròn

 

O

xAB ACB góc tạo tia tiếp tuyến dây cung góc nội tiếp chắn cung AB

 

O nên xAB ACB

ABD ACB góc tạo tia tiếp tuyến dây cung góc nội tiếp chắn cung BD

 

I nên ABD ACB

Do xAB ABD Ax/ /BDOAAx OA, BD suy OABD Câu 15 Giải:

Giả sử CA cắt

 

O Fthì EF

đường kính

A AB;

, ta có BF BE (vì BAEF) Ta có: BED BFD,

  1s  

2

BCFBCE  đBFDE

   

1s 1s

2 BE DE BD BFD  

   

 

 

đ đ

Từđó suy BED ECB Xét tam giác ,

BCE BED

  có B chung, BED ECBBCE BED BC BE DB CB. EB2

BE BD

  $     Câu 16 Giải:

a) Ta có OAOC   a OAC cân O Mà ADO 900 (góc nội tiếp chắn nửa đường tròn

 

O' ) ODACOD đường phân giác AOC, nghĩa AOD DOM

 

AD DM

  (hai góc tâm

nhau nên cung chắn nhau)

AD DM ADM

    cân D

(184)

 

AOECOE (cmt); OAOCa, AOE  COE (c.g.c) EAO ECO 900 hay

EAAB A, OAa bán kính

 

OEA tiếp tuyến

 

O

 

O' Câu 17 Giải:

a) Do BD BH, hai tiếp tuyến cắt đường tròn

 

M

BM

 tia phân giác ABD 1 2 

2

HBD

B B

   Lý luận tương

tự AM tia phân giác BAC 1 2 

2

BAC

A A

  

b) AMB 900 (góc nội tiếp chắn nửa đường tròn)  

1 90

A B

  

   

0

90 180

2

HBD BAC

HBD BAC

     Vậy AC / /BD, mà MDBD MC, AC (gt)

nên M C D, , thẳng hàng Ta có OM đường trung bình hình thang vng ABDC nên / /

OM AC mà CD AC (gt) OMCD M , CM bán kính

 

MCD tiếp

tuyến đường tròn

 

O M

c) Áp dụng tính chất hai tiếp tuyến cắt đường trịn, có:

2

AC AH

AC BD AH BH AB R const

BD BH

 

      

 

 Áp dụng hệ thức lượng tam

giác vuông: . . 2

4 CD

AC BDAH BHMH  (do CHD vng có HM trung tuyến ứng với cạnh huyền)

(185)

Câu 18 Giải:

a) Ta có AIB 900 (góc nội tiếp chắn nủa đường trịn) BIAE Tương tự ACBE  AEB có hai đường cao AC BI, cắt

KK trực tâm AEBEKAB (tính chất ba đường cao)

b) Do I điểm AC IA IC IBA IBC (hai góc nội tiếp chắn hai cung

bằng nhau) Mà IAC IBC (hai góc nội tiếp chắn IC) IAC IBAFAK

 có AI đường cao

AIBI

đồng thời đường trung tuyến (F K đối xứng qua I )

FAK

  cân AFAI IAK.Ta có

       900

FABFAIIABIAKIABIBA IAB  AFAB AAF tiếp tuyến

của

 

O c) sinKAHKH AK

 mà

 2

sin

3

KH

BAC AK HK

AK

     ABE có BI vừa đường cao vừa đường

phân giác  ABE cân B nên BI đường trung trực KAKE K

BI

3 1

2

EHEKKH   KH

  Ta có

2

2 1

3 6

2

KH KHHEKH KH    KH   KH

   

 

3

2

2

HE KE    HK HK   HK

  Suy KH KH

2HE

2HE KE

Câu 19 Giải:

a) Do M điểm AC

 

MA MC

  NBM ABM

(hai góc nội tiếp chắn hai cung

bằng nhau) BM đường phân

(186)

giác ABN ABM.Mặt khác BMA 900 (góc nội tiếp chắn nửa đường trịn)

BAN

 có BM vừa đường cao vừa đường phân giác  BAN cân B

 

BAN BNA

  Ta lại có BAN MCN (vì bù BCM) Do BNA MCN  CMN

cân M

b) Do MBMQ (gt)  BMQ cân MMBQ MQBMCB MNQ (vì bù với hai

góc nhau)  BCM QNM (g.g) BC CM

QN MN

   (do CMN cân M nên

CMMN )QNBC BCA 900 (góc nội tiếp chắn nửa đường trịn) Xét BAQ vng A, ACBQ có:

2 .

ABBC BQBC BNNQBC ABBC (1) Đặt BCx x, 0, biết AB 2R, từ (1) cho 4R2 x R

2 x

x2 2Rx4R2 0 ' R2 4R2 5R2   ' R 5,

1

x   R R x2   R R 0 (loại) Vậy BC

51

R Câu 20 Giải:

a) Đường kính AC vng góc

với dây DE M MDME

Tứ giác ADBEMDME,

MAMB (gt), ABDE

ADBE

 hình thoi (hình bình

hành có hai đường chéo vng góc nhau)

b) Ta có BIC 900 (góc nội tiếp chắn nủa đường tròn

 

O' )

 900

ADC  (góc nội tiếp chắn nửa đường trịn

 

O )BICD ADDC nên AD/ /BI , mà BE / /ADE B I, , thẳng hàng (tiên đềƠclit) DIEIM đường trung tuyến ứng với

cạnh huyền MIMD Do MIMD (cmt)  MDI cân MMID MDI + O I' O C' R  O IC' cân O' O IC' O CI' Suy

 '  ' 900

(187)

c) BCI BIM (góc nội tiếp, góc tạo tia tiếp tuyến dây cung chắn BI) BCI BIH (cùng phụ HIC) BIM BIH IBlà phân giác MIH MIH Ta lại có BICIIC phân giác đỉnh I MIH Áp dụng tính chất phân giác MIH có:

BH IH CH CH MB BH MC

MBMICM  

Câu 21 Giải:

Xét tứ giác AKDLKDLKAL 1800 (vì K L 900)KDL 1800 600 1200 Theo tính chất hai tiếp tuyến cắt

ta có DM DN, tia phân giác KDPPDL

    1200 600

2 2

KDP PDL KDL

MDN

     Ta có: MDC MDNNDC 600 NDC;

   600 

MDCBBMD  NDC (góc ngồi BMD)

 

NDC BMD

  , mà MBD DCN 600 (ABC đều) BMD CDN (g.g)

2

4 BM BD BM CN BD CD BC CD CN

     b) Ta có

1 .

2 . .

1 .

2 MDN ABC

MN PD

S MN PD MN KD MN

SAD BCBC ADBC ADBCDMD tia

phân giác BMN DKDP , AKD có  90 ,0  300

2

AD KD K KAD KD

AD

     

c) Dựng đường trịn bàng tiếp góc A có tâm O AEF Do AD đường trung tuyến ABC nên AD tia phân giác BAC Suy OAC Gọi P K L', ', ' tiếp điểm

 

O với EF AB AC, , Ta có AK'AL P E'; ' EK P F'; ' FL' (tính chất hai tiếp

tuyến cắt nhau) PAEFAEEFFAAEEP'P F' FA

' ' ' ' '

AE EK FL FA AK AL AK

       Mà

2

AEF ABC

PP (gt)

1

2 '

2 ABC AK P AB

   (ABC đều) ' '

4

AB AK AB BK

    (vì AK'K B' AB)

2 '

4 AB BK AB

  Mặt khác

2 2

2

2

BD BC

BD   

  (D trung điểm BC ); ABBC ( ABC

(188)

' 900

OK B  OD (vì O D, AD) Mà K AL' 'K DL' '1800 (vì AK DL' ' tứ giác nội tiếp) mà K AL' '600 K DL' '1200 EDF 600 (tia phân giác của hai góc kề)

Câu 22 Giải:

a) Xét MADMBAAMB chung;

 

MADMBA (góc nội tiếp, góc tạo tia tiếp tuyến dây chắn AD)

MAD MBA

  $ (g.g) MA AD MD

MB AB MA

  

b) Ta có MAMC (tính chất hai tiếp

tuyến cắt đường tròn) MD MD

MA MC

  Lập luận tương tự, ta có MD CD

MCBC Suy

ra AD CD AD BC AB CD

ABBC  

c) Dựng điểm EAC cho EDC ADBDAB

 DECADB EDC (cách dựng), ABD ECD (hai góc nội tiếp chắn AD

) DAB$DEC (g.g) AB BD AB DC EC BD

EC DC

    (1) Do

   

EDCADBBDCADE, nên DAE$DBC (g.g)AD BCBD AE (2) Từ (1) (2) ta có AB CDAD BCBD AE

EC

BD AC

c) Ta có

AD BC AB CD

AB CD AC BD

AD BC AB CD AC BD

 

  

  



AC 2AB (gt) 2AB CD 2AB BDCDBD Suy tam giác BCD cân D Câu 23 Giải:

a) Áp dụng tính chất góc nội tiếp chắn nửa đường trịn ta có:

  900

AEBAMB  ,

  900

BMCAEC

  1800

AEC BMC

    Tứ giác MCED nội tiếp đường trịn ABC có hai đường cao

,

(189)

b) cosABCBE BH BE BC BH AB

AB BC

    c) + Gọi I giao điểm tiếp tuyến M đường tròn

 

O với CD Trong đường tròn

 

O

 

IMDMAB (góc nội tiếp, góc tạo tia tiếp tuyến dây chắn MB), MAB MDI (cùng phụ với ACH)IMD MDI  IMD cân IIMID Ta lại có IMC ICM (cùng phụ với hai góc nhau)  MIC cân IIMIC Vậy IMIDICI trung điểm CD

+ CEDEI trung tuyến ứng với cạnh huyền nên IEICIDIM, CEDIED

IMIE (cmt), OI chung, OMOER  IMO  IEO (c.c.c)

  900 ,

IEO IMO IE OE OE R

      nên IE tiếp tuyến đường tròn

 

O E

Nghĩa tiếp tuyến M E, đường tròn

 

O cắt điểm I thuộc CD

d) AHCH 900, CAH 450  AHC vuông cân tại HCHAHx

 300  600

EAB  EBA ; cotEBAHB cot600 3

HC

   3

3

HB HC x

   Ta có

3

2 3

3 3 3

R

ABAHHBR  x  xR

 Vậy

.2 3

2

ABC

AB CH

S   R RR (đvdt) Câu 24 Giải:

a) Ta có     

0

0

180 120

180 120

BDO BOD B

BDO COE

BOD COE DOE

    

  

    

 ,

DOE B 600

BDO COE

   (g.g) BD OB

OC CE

 

4 BC BD CE OB OC

   (không đổi)

(190)

BDO COE  

OD BD BD

OE OC OB

   mặt khác

  600

DBODOE   BDO ODE (c.g.c) BDO ODE, mà tia DO nằm hai tia

,

DB DEDO tia phân giác BDE

c) ABC nên đường trung tuyến AO đường phân giác BAC, mà DO phân giác đỉnh DO tâm đường trịn bàng tiếp góc A ADE

ĐƯờng trịn

 

O ln tiếp xúc DE AC,

d) APAQ (tính chất hai tiếp tuyến cắt nhau), ABAC

 

/ / 60

AP AQ PQ BC IQA ACB

AB AC

      , mà DOE 600 IQE IOE 60 ; ,0 O Q

hai đỉnh liên tiếp tứ giác IOQE Tứ giác IOQE nội tiếp (cùng thuộc cung chứa góc) Suy EIO EQO 900 Lý luận tương tự DNE 900 Vậy tứ giác DINE (DIEDNE

cùng nhìn DE góc vuông) ONI ODE Vậy ONI ODE (g.g)

0

cos 60

2

IN ON

DE NI

DE OD

     

Câu 25 Giải:

a) Do AB AC, hai tiếp tuyến cắt đường tròn

 

O nên ABO ACO 900 B C,

thuộc đường trịn đường kính OA có tâm I trung điểm OA

b) Ta có

2 AB

AM AOAIAB AI c) Gọi E trung điểm MA, G trọng tâm CMA nên G CE

3 GE

CE  Mặt khác

1 ME

BE  (vì

2 MA MB

ME   nên

3 BE

ME  ) GE ME CE BE

  , theo định lý Ta-lét đảo MG/ /BC d) Gọi G' giao điểm OA CMG' trọng tâm ABC Nên '

3 ' G M GE

CM  CE , theo

định lý Ta-lét đảo GG'/ /ME (1)

MI đường trung bình OABMI / /OB, mà ABOB (cmt) MIAB, nghĩa MIME (2) Từ (1) (2) cho MIGG', ta lại có GI'MK (vì OAMK) nên I trực

(191)

Câu 26 Giải:

a) Gọi O' giao điểm AO với cung nhỏ DE đường tròn

 

OO' thuộc đường phân giác A ADE Ta có

 

DOAEOA (tính chất hai tiếp tuyến cắt nhau) ' '

DO O E

  Mà

đ  đ

' s '; ' s '

2

ADODO EDOO EADO'EDO' DO' phân giác D O' tâm

đường trịn nội tiếp ADE Do OO'R

b) Do ABAC (tính chất hai tiếp tuyến cắt nhau)  ADE cân A nên  1800  900 

2

BAC BAC

ADE     Mà     

2

ABC

ADEABMNMB  NMB (do BO phân

giác ABC nên  

2

ABC

ABM  )    900   

2 2

B BAC ABC ACB

NMB ADE

      Mặt khác

 

2

ACB

NCB  (do CO tia phân giác ACB) Suy NMB NCB, mà M C, hai đỉnh liên

tiếp tứ giác BCMN Tứ giác BCMN nội tiếp (vì thuộc cung chứa góc) c) NMOBCONOM BOC (đối đỉnh); NMO BCO (cmt) NMO$BCO

(g.g) OM ON MN

OC OB BC

   Tương tự DMO$ACO (g.g) DM OM

AC OC

  ; NEO$BAO

(g.g)

(192)

a

b= a

c c

d

b Đỉnhcây

Gốccây

CHUYÊN ĐỀ:CÁC BÀI TỐN THỰC TẾ HÌNH HỌC §1 ĐỊNH LÍ PYTHAGORE VÀ NHỮNG ỨNG DỤNG

TRONG CÁC BÀI TOÁN THỰC TẾ A. KIẾN THỨC LIÊN QUAN

Định lí Pythagore định lí quan trọng tất định lí khoa học nói chung hình học nói riêng Định lý Pythagore đơn giản lí thú Nhiều nhà khoa học cịn cam đoan có người sống hành tinh khác định lí hình học có giá trị mà họ tìm định lí Pythagore Đã có dự án đề nghị xây dựng cơng trình tường xanh tạo thành tam giác vng có ba cạnh 3,

khổng lồ cánh đồng lớn để liên lạc với người Trái Đất

Ngày 08 tháng 09 năm 1977, hai tàu thăm dò Voyager Mỹ phóng lên vũ trụ mang theo hình vẽ biểu diễn định lí Pythagore

Pythagore nhà hiền triết người Hy Lạp sống khoảng 500 năm trước công nguyên Sau

này người ta phát định lí Pythagore biết đến trước từ lâu văn minh cổ đại giới Điển hình số nhà khảo cổ tìm thấy bảng đất sét nung văn minh Babilon nghìn năm trước Pythagore có hình vẽ khác tam giác vng có cạnh thể định lí

Trong văn tự Ấn Độ cổ đại khoảng 1500 năm trước Công

nguyên có phần quan trọng gọi Sulbasutras nói việc đo đạc thiết kế đền thờ Ở phần tìm thấy định lí Pythagore dạng: Diện tích hình vng có cạnh cạnh

huyền tam giác vuông tổng diện tích hai hình vng bằng tổng diện tích hai hình vng có cạnh hai cạnh bên

của tam giác vng đó.

B. VÍ DỤ MINH HOẠ ỨNG DỤNG TOÁN HỌC VÀO BÀI TOÁN THỰC TẾ PHƯƠNG PHÁP GIẢI

- Sử dụng công thức Pythagore để tìm cạnh góc vng cạnh huyền từ hai cạnh lại: c2 a2 b2 (c cạnh huyền, a b, cạnh góc vng)

- Rút kết luận tốn

Ví dụ 1

Từ đỉnh có treo cáidây thả xuốngđất thừa một đoạn có độ dài d Nếu kéo căng dây đầu dây chạm đất khoảng cách b so với gốc Hãy tìm độ dài dây. Nếu có độ dài a có tốn tính độ dài c

(193)

a

b c

d c

Ngọnt re

Gốct re Chỗgãy

a

b c

c - b

C

E D

B A a  c d b Theo định lí Pythagore ta có:

2 2

(c d ) bc .

Từ suy ra: 2

2 b d c d  

Ví dụ 2

Có tre có độ cao a Khi gãy tre chạmđất khoảng cách b so với gốc tre Hãy tìm độ cao chỗ tre. Ta phải tính cạnh a tam giác vng có cạnh bên b cạnh huyền c d a

Theo định lí Pythagore ta có: a2b2 (da)2 Từ suy ra: 2

2 d b a d  

Ví dụ 3

Có ao hình vng, cạnh dài 3, 33m, ao có sậy nhơ lên khỏi mặt

nước vừa 0, 33m, kéo sậy vào bờ chọn vừa chạm mặt nước Hỏi độ sau

nước sậy cao bao nhiêu?

Giả sử chiều rộng ao ED2a 3, 33 (m),

C trung điểm ED nên:

1, 665

DC  a (m)

Chiều cao sậy mặt ao AB, phần nhô khỏi mặt nước

0, 33

AC  (m)

ABBD, giả sử BDc, độ sâu nước BCb, tam giác BCD tam giác vuông Rõ ràng ACABBC   c b 0, 33 (m)

Độ dài AC hiệu đường huyền với cạnh dài góc vng Vậy tốn quy việc tính chiều dài cạnh huyền cạnh góc vng lớn

tam giác vng biết cạnh góc vng bé hiệu cạnh huyền cạnh góc vng lớn

Từ định lí Pythagore, ta có:

2 2

acb

2 ( )2 2 ( )2 a  c bc   b c b

2 ( 2 2)

c b c bc b

    

2bc2b2

2 (b c b )

2 ( )2

2( )

a c b

b c b     (1) ( )

(194)

A

D

C O

B

H

D C

M

J A

F E

B W

Đem giá trị a c b,  thay vào hai công thức (1) (2) dễ dàng tính độ sâu nước là:

2

1, 665 0, 33 2, 772225 0,1089 4, 035

2.0, 33 0, 66

b     (m)

Độ cao sậy là: c4, 0350, 334, 365 (m)

C. LỜI BÌNH

Định lí Pythagore định lí hình học nói riêng khoa học nói chung có nhiều cách chứng minh Theo thống kê, đến có 385 cách giải Nhiều trị gia

lỗi lạc Tổng thống Hoa kỳ James Garfiel tham gia tìm cách chứng minh định lí Ở bậc học cao hơn, người ta dùng Vật lí học để chứng minh định lí Pythagore Định lí Pythagore cịn xuất mơn phi-Euclide, hình học giả Euclide, phương trình vi phân, Đại số tuyến tính, … Hầu lĩnh vực quan trọng người ta thấy bóng dáng định lí Pythagore Qua minh chứng tầm quan trọng định lí Pythagore lĩnh vực khoa học đời sống

D. BÀI TẬP RÈN LUYỆN

Bài tốn

Tính bán kính đường trịn ngoại tiếp tam giác có cạnh 50, 50, 60

Bài toán 2

Dựng hình vng có diện tích diện tích hình chữ nhật cho trước.

E. ĐÁP SỐ VÀ HƯỚNG DẪN GIẢI

Bài toán

Theo định lí Pythagore, ta có: AD2 AC2DC2

Do DCBC : 230, nên:

AD 502302 40 Ta lại có:

OC2 DC2 (AD OA )2

DC2AD22AD OC. OC2 Do đó: 2 302 402 125

2 2.40

DC AD

OC

AD

 

  

Bài tốn 2

Cho hình chữ nhật ABCD Ta vẽ hình chữ vng ABKH hình chữ nhật ABCD Sau xác định trung điểm E M

DH CK

Dựng hình vng AEFJ qua M Lấy J làm tâm vẽ đường trịn có bán kính JF cắt BM W Hình vng có cạnh BW có diện tích diện tích ABCD theo định lí Pythagore ta có: BW2 JW2BJ2

(195)

a

c b

A

B C

(JFBJ JF)( KM)

AB BC

§2 HỆ THỨC GIỮA CÁC CẠNH VÀ CÁC GĨC CỦA MỘT TAM GIÁC VUÔNG

A. KIẾN THỨC LIÊN QUAN

Trong tam giác vuông, cạnh góc vng bằng: - Cạnh huyền nhân với sin góc đối hay nhân với cơsin góc kề - Cạnh góc vng nhân với tang góc đối hay nhân với cơtang góc kề

B. VÍ DỤ MINH HOẠ ỨNG DỤNG TOÁN HỌC VÀO BÀI TOÁN THỰC TẾ PHƯƠNG PHÁP GIẢI

- Sử dụng hệ thức lượng tam giác vng cách thích hợp như:

sin canhdoi ; cos canhke

canhhuyen canhhuyen

tan canhdoi; cotan canhke

canhke canhdoi

( góc nhọn tam giác vuông)

- Từ rút kết luận tốn

Ví dụ 1

Một cột điện có bóng mặt đất dài 7, 5m, tia sáng mặt trời tạo với mặt đất góc xấp xỉ

0

42 Tính chiều cao cột đèn.

(196)

6 10

H A

B

12

H A

C B

Gọi chiều cao cột đèn AB, bóng mặt đất AC Ta có BAC 900

Theo giả thiết, ta có BCA 420

Áp dụng tỉ số lượng giác tam giác ABC vuông A, ta có:

tanBCAAB AB AC tanBCA 7, tan 420 6, 75

AC

     (cm)

Vậy chiều cao cột đèn 6, 75 (cm)

Ví dụ 2

Ở độ cao 920m, từ máy bay trực thăng ngườita nhìn hai

điểm D C, của hai đầu cầu góc so với đường vng góc với

mặt đất góc 37 ,0 310 Tính chiều dài CD của cầu (hình vẽ).

Gọi A vị trí trực thăng, B chân đường vng góc hạ từ A xuống mặt đất C D hai điểm đầu cầu

Ta có

tanBADBD AB

BDAB tanBAD 920 tan 370 920.0, 754693, 68 (m)

Mặt khác

tanBACBC AB

BCAB tanBAC 920 tan 310 920.0, 6552 (m) Vậy chiều dài cầu là:

CDBDBC 693, 68 552 141, 68 (m)

C. LỜI BÌNH

Hệ thức lượng tam giác vuông chủ đề hay quan trọng chương trình tốn phổ thơng Nó có nhiều ứng dụng thực tế Bài viết cần trao đổi thêm? Mong chia sẻ bạn

D. BÀI TẬP LUYỆN TẬP

Bài toán

Cho tam giác ABC vuông A, đường cao AH Tính sin , sinB C ứng với trường hợp sau:

(197)

H

A

B

Bài tốn 2

Cho tam giác vng ABC vng A Tính sin , tanB B trong trường hợp sau: a) 12

13

AB

BC; b)

15

AB

AC.

E. ĐÁP ÁN VÀ HƯỚNG DẪN GIẢI

Bài toán

a) Ta có: AH2 AB2BH2 10262 100 36 64 Vậy AH  64 8 (cm)

Do sin

10

AH B

AB

  

Ta có: 2 12 2

AHABAC

2 2 2 2 2 2

1 1 10 36

6400

10

AB AH

AC AH AB AH AB

 

     

Vậy 6400 80 13,

36

AC    (cm) Theo định lí Pythagore, ta có:

BC2 AB2 AC2 10213, 32 276, 89 Suy BC  276, 89 16, 64 (cm)

Vì vậy: sin 10 0, 16, 64

AB C

BC

  

b) Áp dụng định lí Pythagore tam giác vng AHB vng H, ta có: 2 122 52 144 25 169

ABAHBH     

Do AB 16913 (cm)

Suy ra: sin 12 13

AH B

AB

 

Ta có 2 12 2

AHABAC

2 2 2 2 2 2

1 1 13 12 25

24336

13 12

AB AH

AC AH AB AH AB

 

    

Vậy 24336 31,

25

AC   (cm)

Áp dụng định lí Pythagore tam giác AHC vng H: HC2 AC2AH2 31, 22122 829, 44

Vậy HC  829, 44 28, (cm)

Ta có: BCBHHC  5 28, 833, (cm)

Vậy: sin 13 33,

AB C

BC

(198)

H B A

C

Bài tốn 2

a) Ta có: cos 12 13

AB B

BC

 

Áp dụng công thức 2

sin Bcos B1, ta được:

2

2 12 144 25

sin cos 1

13 169 169

B  B        

 

Từ đó, ta có: sin 25

169 13

B   (do sinB0)

Mặt khác,

5

sin 13

tan

cos 12 13

13

B B

B

  

b) 15

8

AB

AC

Ta có: cotan 15 tan 1

8 cotan 15 15

8

AB

B B

AC B

     

Theo công thức lượng giác, ta được:

2

2

1 15 225 289

cotan 1

8 64 64

sin B B

   

          

Từ đây, suy ra: sin 64

289 17

B   (do sinB0)

§3 ĐỊNH LÍ THALES TRONG CÁC BÀI TỐN THỰC TẾ A. KIẾN THỨC LIÊN QUAN

(199)

nhiều khám phá địa lý thực Thế giới sẵn sàng cho kiểu văn minh

Nền văn minh xuất thành phố thương mại chạy dài dọc theo bờ biển Tiểu Á sau lãnh thổ Hy Lạp, vùng biển Italia Cái nhìn tĩnh phương đơng cổ đại trở nên phủ nhận bầu khơng khí phát triển chủ nghĩa lý, người ta bắt đầu hỏi

Ở thời gian đầu, toán học lĩnh vực khác, người ta bắt đầu đặt câu hỏi có tính chất “Tại các góc đáy tam giác cân lại nhau?” “Tại đường kính lại chia đơi đường trịn?” Những q trình thực nghiệm phương đơng cổ đại hồn tồn đủ để trả lời câu hỏi làm không đủ để trả lời câu hỏi có tính chất khoa học từ Ít nhiều cố gắng phương pháp chứng minh để tự khẳng định khía cạnh suy diễn mà học giả ngày coi đặc trưng toán học thấy xuất Có thể tốn học có ý nghĩa từ này, đời khơng khí chủ nghĩa lý đô thị thương mại nằm vùng bờ biển phía tây Tiểu Á Theo lời truyền lại hình học

chứng minh bắt đầu với Thales vùng Miletus, “bảy nhà thông thái” thời đại khoảng thời gian nửa đầu kỷ XV trước Công nguyên

Theo nhà nghiên cứu lịch sử, phần đầu đời Thales nhà buôn trở nên giàu có để quãng đời sau đời dành cho việc nghiên cứu học tập du lịch Ông thiên tài nhiều mặt khách, người cố vấn, kỹ sư, doanh nghiệp, nhà triết học, toán học thiên văn học Thales người biết đến với khám phá toán học Trong hình học ơng cơng nhận đưa kết sau đây:

- Một đường trịn chia đơi đường kính - Hai góc đáy tam giác cân - Các góc đối đỉnh

- Hai tam giác theo trường hợp góc cạnh góc

Thales coi người đoán tượng nhật thực vào năm 585

B. VÍ DỤ MINH HOẠ ỨNG DỤNG TOÁN HỌC VÀO BÀI TOÁN THỰC TẾ PHƯƠNG PHÁP GIẢI

- Sử dụng tỉ số hai tam giác đồng dạng

AB A B

ABC A B C

AC A C

    

   

 

- Rút kết luận tốn

Ví dụ

(200)

Kim tự tháp cơng trình kiến trúc cổ hùng vĩ phần mộ vua chúa Ai Cập cổ đại Hơn 2600 năm trước, có vương quốc Ai Cập muốn biết độ cao thực kim tự

tháp bao nhiêu, chẳng đo

Cho người trèo lên đỉnh tháp? Rõ ràng khơng thể tháp nghiêng, có trèo lên chẳng biết dùng cách để đo

Thales cho người giúp Quốc Vương đo chiều cao kim tự tháp Thales chọn ngày đẹp trời, mời Quốc Vương quan trọng triều hành lễ đo tháp Người đến xem đông, chen chúc nhau, bàn tán vào sôi Nhưng thời gian trôi đi, mặt trời chiếu xuống Kim tự tháp đám người mà chưa thấy Thales có động tĩnh Mãi thấy bóng người chiều cao ông, ông phát lệnh đo tháp Lúc đó, người giúp việc đo độ dài bóng Kim tự tháp DB (hình vẽ trên) Sau đó, ơng đưa chiều cao Kim tự tháp cách chuẩn xác

Thales làm để đo chiều cao Kim tự tháp? Ơng phải chờ tới độ dài bóng người ơng độ cao ơng đo, lúc tia nắng mặt trời người ơng tạo thành góc 450 Tức CBA 450 ACB 90 ,0 BAC 450 Lúc ấy, điểm đỉnh Kim tự tháp với điểm trung tâm Kim tự tháp điểm cuối bóng Kim tự tháp tạo thành tam giác vuông cân, đương nhiên hai cạnh bên ACCB Nửa độ dài Kim tự tháp đoạn CD (đã ơng đo trước, cịn độ dài đoạn bóng Kim tự tháp DB ơng nhờ trợ lý đo Cuối việc cộng lại hai đoạn CD DB lại chiều cao Kim tự tháp

Ví dụ 2

Làm để đo chiều cao cây?

Ngày đăng: 24/02/2021, 05:36

TỪ KHÓA LIÊN QUAN

TÀI LIỆU CÙNG NGƯỜI DÙNG

TÀI LIỆU LIÊN QUAN

w